Vous êtes sur la page 1sur 98

Contents

CRIMINAL PROCEDURE .................................................................................................................................. 4 SERANA VS. SANDIGANBAYAN G.R. NO. 162059, JANUARY 22, 2008 .............................................................. 4 LACSON VS. EXECUTIVE SECRETARY G.R. NO. 128096 JANUARY 20, 1999 ....................................................... 5 GARCIA VS. SANDIGANBAYAN G.R. NO. 170122 OCTOBER 12, 2009 ............................................................... 7 SANCHEZ VS. DEMETRIOU G.R. NOS. 111771-77 NOVEMBER 9, 1993............................................................. 9 DE VERA VS. DE VERA G.R. NO. 172832 APRIL 7, 2009 .................................................................................... 9 PEOPLE AND PHOTOKINA VS. BENIPAYO (G.R. NO. 155573 APRIL 24, 2009) ................................................. 10 FOZ, JR. VS. PEOPLE G.R. NO. 167764 OCTOBER 9, 2009 .............................................................................. 12 PEOPLE VS. SANDIGANBAYAN G.R. NO. 173396, SEPTEMBER 22, 2010 ........................................................ 13 RAMISCAL, JR. VS.SANDIGANBAYAN G.R. NOS. 172476-99 SEPTEMBER 15, 2010 ......................................... 13 PEOPLE VS. JACK RACHO G.R. NO. 186529 AUGUST 3, 2010 ........................................................................ 15 MIAQUE VS. PATAG G.R. NOS. 170609-13 JANUARY 30, 2009 ...................................................................... 16 BALTAZAR VS. CHUA G.R. NO. 177583 ......................................................................................................... 17 PEOPLE VS. TEODORO G.R. NO. 172372 DECEMBER 4, 2009 ........................................................................ 18 LEVISTE VS. ALAMEDA G.R. NO. 182677 AUGUST 3, 2010 ............................................................................ 19 CAPISTRANO VS. LIMCUANDO G.R. NO. 152413 FEBRUARY 13, 2009 ........................................................... 20 ALBERT VS SANDIGANBAYAN 580 SCRA 279 ............................................................................................... 20 YAP VS. CABALES G.R. NO. 159186 JUNE 5, 2009 .......................................................................................... 21 PIMENTEL V. PIMENTEL G.R. NO. 172060 .................................................................................................... 22 HEIRS OF SIMON V. CHAN G.R. NO. 157547 ................................................................................................. 23 METROBANK V. YAO G.R. NO. 180165 ......................................................................................................... 24 ALAWIYA VS. DATUMANONG G.R. NO. 164170 APRIL 16, 2009 .................................................................... 25 ENRIQUE V. VIUDEZ II vs.THE COURT OF APPEALS G.R. No. 152889 June 5, 2009 ..................................... 27 PEOPLE VS. CASTILLO G.R. No. 171188 June 19, 2009 .................................................................................. 28 HEIRS OF THE LATE NESTOR TRIA V. ATTY. EPIFANIA OBIAS G.R. NO. 175887 NOV. 24, 2010 ....................... 29 BORLONGAN, JR VS PENA GR NO. 143591.................................................................................................... 31 PEOPLE VS. GREY G.R. NO. 180109 JULY 26, 2010 ........................................................................................ 33 PEOPLE OF THE PHILIPPINES VS. GADIANA G.R. NO. 184761 SEPTEMBER 8, 2010 ....................................... 33 PEOPLE VS. NG YIK BUN 639 SCRA 639 SCRA ................................................................................................ 34 PICO VS. COMBONG 215 SCRA 421 .............................................................................................................. 37 GOVT. OF THE USA VS. PURGANAN 389 SCRA 623 ...................................................................................... 38 ZUO VS. CABEBE 444 SCRA 382.................................................................................................................. 39 LEVISTE VS. CA G.R. NO. 189122 MARCH 17, 2010 ....................................................................................... 40 1

IVLER VS. MODESTO-SAN PEDRO G.R. NO. 172716 NOVEMBER 17, 2010 .................................................. 41 GACAL VS. INFANTE [FORMERLY A.M. NO. IPI NO. 03-1831-RTJ] A.M. NO. RTJ- 04-1845 OCTOBER 5, 2011 . 41 OKABE VS. GUTIERREZ G.R. NO. 150185 MAY 27, 2004 ................................................................................ 42 RE: ANONYMOUS LETTER-COMPLAINT AGAINST HON. MARILOU RUNES-TAMANG, PRESIDING JUDGE A.M. MTJ-04-1558 APRIL 7, 2010(FORMERLY OCA IPI NO. 04-1594-MTJ) ...................................................... 43 PEOPLE vs. HON. JUDGE AYSON G.R. NO. 85215 .......................................................................................... 44 PEOPLE VS. MORIAL, ET. AL. G.R. NO. 129295 .............................................................................................. 44 PEOPLE VS. BASADA, ET. AL. G.R. NO. 185840 .............................................................................................. 45 PEOPLE V. SIONGCO G.R. NO. 186472 JULY 5, 2010...................................................................................... 45 PEOPLE VS. FRANCISCO G.R. NO. 192818 NOVEMBER 17, 2010.................................................................... 46 IMPERIAL VS JOSON ..................................................................................................................................... 47 PEOPLE VS ESTOMACA................................................................................................................................. 48 SAMSON VS DAWAY .................................................................................................................................... 49 PEOPLE VS PANGILINAN............................................................................................................................... 50 PEOPLE VS AGUILAR .................................................................................................................................... 50 DAAN VS. SANDIGANBAYAN G.R. NOS. 163972-77 ...................................................................................... 51 BIENVENIDO DIO VS. PABLO OLIVAREZ G.R. NO. 170447 ........................................................................... 52 PEOPLE VS. JANJALANI G.R. NO. 188314 JANUARY 10, 2011 ............................................................ 53 PEOPLE VS. ABAD G.R. NO. L-55132 AUGUST 30, 1988 ................................................................................. 54 PEOPLE VS. LACSON G.R. NO. 149453 MAY 28, 2002 .................................................................................... 55 PEOPLE VS NAVARRO 414 SCRA 395 G. R. NO. 137597, OCTOBER 24, 2003 ............................................... 57 PANAGUITON VS DOJ - 571 SCRA 549 G.R. NO. 167571, NOVEMBER 25, 2008 ............................................. 58 JUMAQUIO VS VILLAROSA - 576 SCRA 204 G.R. NO. 165924, JANUARY 19, 2009 .......................................... 60 PEOPLE VS DUMLAO - 580 SCRA 409 G.R. NO. 168918, MARCH 2, 2009 ....................................................... 60 SORIANO VS PEOPLE - 591 SCRA 244 G.R. NO. 159517-18, JUNE 30, 2009 .................................................... 62 PEOPLE VS. TAN G.R. NO. 167526 JULY 26, 2010 ......................................................................................... 63 JOSEPH C. CEREZO VS. PEOPLE G.R. NO. 185230, JUNE 01, 2011 .................................................................. 64 LUGTU VS. COURT OF APPEALS .................................................................................................................... 65 SALVANERA VS.PEOPLE G.R. NO. 143093 MAY 21, 2007 ............................................................................. 66 MANGUERRA VS. RISOS G.R. NO. 152643, AUGUST 28, 2008 ....................................................................... 67 CABADOR VS. PEOPLE G.R. NO. 186001 OCTOBER 2, 2009 ........................................................................... 69 PEOPLE VS. DE GRANO G.R. NO. 167710 JUNE 5, 2009 ................................................................................. 70 IMPERIAL VS. JOSON G.R. NO. 160067 AND G.R. NO. 171622 NOVEMBER 17, 2010 ..................................... 71 PP VS. SANDIGANBAYAN G.R. NO. 174504 MARCH 21, 2011 ........................................................................ 71 PEOPLE VS. TAN G.R. NO. 167526 JULY 26, 2010 .......................................................................................... 72 2

BANGAYAN JR. V. BANGAYAN ...................................................................................................................... 73 PEOPLE VS. BARTOLOME TAMPUS AND IDA MONTESCLAROS G.R. NO. 181084 JUNE 16, 2009 .................... 75 HIPOS SR. V. BAY.......................................................................................................................................... 76 PEOPLE OF THE PHILIPPINES VS. PATERNO LORENZO G.R. NO. 184760 APRIL 23, 2010 ................................ 77 PEOPLE VS. BARON G.R. NO. 185209, JUNE 28, 2010 ................................................................................... 78 ABELLANA VS. PEOPLE G.R. NO. 174654, AUGUST 17, 2011 ......................................................................... 79 PEOPLE VS. ASIS G.R. NO. 142531 OCTOBER 15, 2002 .................................................................................. 80 ESTINO VS. PEOPLE G.R. NOS. 163957-58 APRIL 7, 2009............................................................................... 81 BRIONES V. PEOPLE ..................................................................................................................................... 82 SALUDAGA VS. GENIO G.R. NO. 184537 APRIL 23, 2010 ........................................................................... 83 LUMANOG VS. PEOPLE G.R. NO. 182555 FEBRUARY 8, 2011 ........................................................................ 84 PAYUMO VS. SANDIGANBAYAN G.R. NO. 151911, JULY 25, 2011 ................................................................. 85 PEOPLE V. MORALES G.R. NO. 172873 MARCH 19, 2010 ........................................................................... 86 QUIDET VS. PEOPLE G.R. NO. 170289 APRIL 8, 2010 ..................................................................................... 87 BALABALA V. PEOPLE GR NO.169519; JULY 17, 2009 .................................................................................... 87 GUASCH V. DELA CRUZ GR NO.176015; JUNE 16, 2009 ................................................................................ 88 PEOPLE V. OLIVO, DANDA AND REYES GR NO.177768; JULY 27, 2009 ........................................................... 89 PEOPLE VS. TARUC GR NO. 185202; FEBRUARY 18, 2009.............................................................................. 90 TIU VS. COURT OF APPEALS AND EDGARDO POSTANES GR NO. 16273; APRIL 21, 2009 ................................ 91 MICLAT, JR. VS. PEOPLE G.R. NO. 176077, 31 AUGUST 2011 ........................................................................ 92 PEOPLE VS. MARIACOS G.R. NO. 188611, 16 JUNE 2010 ............................................................................... 93 PEOPLE VS. TUAN G.R. NO. 176066, 11 AUGUST 2010.................................................................................. 95 ESQUILLO VS. PEOPLE G.R. NO. 182010, AUGUST 25, 2010 .......................................................................... 97

CRIMINAL PROCEDURE SERANA VS. SANDIGANBAYAN G.R. NO. 162059, JANUARY 22, 2008

Facts: Petitioner Hannah Eunice D. Serana, a senior student, was appointed by then President Estrada as a student regent of UP, to serve a one-year term. Petitioner, with her siblings and relatives, registered with the Securities and Exchange Commission the Office of the Student Regent Foundation, Inc. (OSRFI). One of the projects of OSRFI was the renovation of the Vinzons Hall Annex in UP Diliman. Pres. Estrada gave P15 million to the OSRFI as financial assistance for the proposed renovation, from the Office of the President. The renovation of Vinzons Hall Annex failed to materialize. The succeeding student regent, Kristine Clare Bugayong, and Christine Jill De Guzman, Secretary General of the KASAMA sa U.P., a system-wide alliance of student councils within the state university, consequently filed a complaint for Malversation of Public Funds and Property with the Office of the Ombudsman. Serana and her brother were indicted for Estafa. Petitioner moved to quash the information on the ground that the Sandiganbayan does not have any jurisdiction over the offense charged or over her person, in her capacity as UP student regent, as she was not a public officer since she merely represented her peers, in contrast to the other regents who held the position in an ex officio capacity. Petitioner also claimed that she had no power or authority to receive monies or funds. She claimed such power was vested with the Board of Regents (BOR) as a whole. Hence, since it was not alleged in the information that it was among her functions or duties to receive funds, or that the crime was committed in connection with her official functions, the same is beyond the jurisdiction of the Sandiganbayan. Petitioner also posited that Republic Act (R.A.) No. 3019, as amended by R.A. No. 8249, enumerates the crimes or offenses over which the Sandiganbayan has jurisdiction. It has no jurisdiction over the crime of estafa. It only has jurisdiction over crimes Crimes Committed by Public Officer; Estafa falling under Title X, Chapter VI (Crimes Against Property), Book II of the RPC is not within the Sandiganbayans jurisdiction. The Ombudsman opposed the motion. It disputed petitioners interpretation of the law. Section 4(b) of Presidential Decree (P.D.) No. 1606 clearly contains the catch -all phrase in relation to office, thus, the Sandiganbayan has jurisdiction over the charges against petitioner. The Sandiganbayan dismissed the Motion filed by Serana. Issue: 1. Whether or not the Sandiganbayan has jurisdiction over the crime charged; and 2. Whether the Sandiganbayan has jurisdiction over the person of Petitioner. Held: 1. YES, Contrary to Petitioners claims, it is P.D. 1606 which defines the jurisdiction of the Sandiganbayan. Section 4(B) of P.D. No. 1606 reads: B. Other offenses or felonies whether simple or complexed with other crimes committed by the public officials and employees mentioned in subsection a of this section in relation to their office. Evidently, the Sandiganbayan has jurisdiction over other felonies committed by public officials in relation to their office. There is no plausible or sensible reason to exclude estafa as one of the offenses 4

included in Section 4(bB) of P.D. No. 1606. Plainly,estafa is one of those other felonies. The jurisdiction is simply subject to the twin requirements that (a) the offense is committed by public officials and employees mentioned in Section 4(A) of P.D. No. 1606, as amended, and that (b) the offense is committed in relation to their office. In fine, the two statutes differ in that P.D. No. 1606, as amended, defines the jurisdiction of the Sandiganbayan while R.A. No. 3019, as amended, defines graft and corrupt practices and provides for their penalties. 2. YES, Petitioner claims that she is not a public officer with Salary Grade 27; she is, in fact, a regular tuition fee-paying student. This is likewise bereft of merit. It is not only the salary grade that determines the jurisdiction of the Sandiganbayan. The Sandiganbayan also has jurisdiction over other officers enumerated in P.D. No. 1606. While the first part of Section 4(A) covers only officials with Salary Grade 27 and higher, its second part specifically includes other executive officials whose positions may not be of Salary Grade 27 and higher but who are by express provision of law placed under the jurisdiction of the said court. Petitioner falls under the jurisdiction of the Sandiganbayan as she is placed there by express provision of law. Section 4(A)(1)(g) of P.D. No. 1606 explictly vested the Sandiganbayan with jurisdiction over Presidents, directors or trustees, or managers of government-owned or controlled corporations, state universities or educational institutions or foundations. Petitioner falls under this category. As the Sandiganbayan pointed out, the BOR performs functions similar to those of a board of trustees of a non-stock corporation. By express mandate of law, We find that petitioner is, indeed, a public officer as contemplated by P.D. No. 1606 the statute defining the jurisdiction of the Sandiganbayan.
LACSON VS. EXECUTIVE SECRETARY G.R. NO. 128096 JANUARY 20, 1999

FACTS: In the early morning of May 18, 1995, eleven (11) persons believed to be members of the Kuratong Baleleng gang, reportedly an organized crime syndicate which had been involved in a spate of bank robberies in Metro Manila, where slain along Commonwealth Avenue in Quezon City by elements of the Anti-Bank Robbery and Intelligence Task Group (ABRITG) headed by Chieff Superintendent Jewel Canson of the Philippine National Police (PNP). The ABRITG was composed of police officers from the Traffic Management Command (TMC) led by petitioner-intervenor Senior Superintendent Francisco Zubia, Jr.; Presidential Anti-Crime Commission Task Force Habagat (PACC-TFH) headed by petitioner Chief Superintendent Panfilo M. Lacson; Central Police District Command (CPDC) led by Chief Superintendent Ricardo de Leon; and the Criminal Investigation Command (CIC) headed by petitioner-intervenor Chief Superintendent Romeo Acop. Acting on a media expose of SPO2 Eduardo delos Reyes, a member of the CIC, that what actually transpired at dawn of May 18, 1995 was a summary execution (or a rub out) and not a shoot-out between the Kuratong Baleleng gang members and the ABRITG, Ombudsman Aniano Desierto formed a panel of investigators headed by the Deputy Ombudsman for Military Affairs, Bienvenido Blancaflor, to investigate the incident. This panel later absolved from any criminal liability all the PNP officers and personal allegedly involved in May 18, 1995 incident, with a finding that the said incident was a legitimate police operation.

However, a review board led by Overall Deputy Ombudsman Francisco Villa modified the Blancaflor panel's finding and recommended the indictment for multiple murder against twenty-six (26) respondents, including herein petitioner and intervenors. Petitioner Panfilo Lacson was among those charged as principal in eleven (11) information for murder before the Sandiganbayan's Second Division, while intervenors Romeo Acop and Francisco Zubia, Jr. were among those charged in the same informations as accessories after-in-the-fact. On March 5-6, 1996, all the accused filed separate motions questioning the jurisdiction of the Sandiganbayan, asserting that under the amended informations, the cases fall within the jurisdiction of the Regional Trial Court. They contend that the said law limited the jurisdiction of the Sandiganbayan to cases where one or more of the "principal accused" are government officials with Salary Grade (SG) 27 or higher, or PNP officials with the rank of Chief Superintendent (Brigadier General) or higher. The highest ranking principal accused in the amended informations has the rank of only a Chief Inspector, and none has the equivalent of at least SG 27. The Sandiganbayan admitted the amended information and ordered the cases transferred to the Quezon City Regional Trial Court which has original and exclusive jurisdiction under R.A. 7975, as none of the principal accused has the rank of Chief Superintendent or higher. The Office of the Special Prosecutor moved for a reconsideration, insisting that the cases should remain with the Sandiganbayan. While these motions for reconsideration were pending resolution, and even before the issue of jurisdiction cropped up with the filing of the amended informations on March 1, 1996, House Bill No. 2299 and No. 1094 (sponsored by Representatives Edcel C. Lagman and Lagman and Neptali M. Gonzales II, respectively), as well as Senate Bill No. 844 (sponsored by Senator Neptali Gonzales), were introduced in Congress, defining expanding the jurisdiction of the Sandiganbayan. Specifically, the said bills sought, among others, to amend the jurisdiction of the Sandiganbayan by deleting the word "principal" from the phrase "principal accused" in Section 2 (paragraphs a and c) of R.A. No. 7975. These bills were consolidated and later approved into law as R.A. No. 8249 13 by the President of the Philippines on February 5, 1997. Subsequently, on March 5, 1997, the Sandiganbayan promulgated a Resolution denying the motion for reconsideration of the Special Prosecutor, ruling that it "stands pat in its resolution dated May 8, 1996." Petitioner and entervenors' posture that Section 4 and 7 of R.A. 8249 violate their right to equal protection of the law 33 because its enactment was particularly directed only to the Kuratong Baleleng cases in the Sandiganbayan, is a contention too shallow to deserve merit. No concrete evidence and convincing argument were presented to warrant a declaration of an act of the entire Congress and signed into law by the highest officer of the co-equal executive department as unconstitutional. Every classification made by law is presumed reasonable. Thus, the party who challenges the law must present proof of arbitrariness. As stated earlier, the multiple murder charge against petitioner and intervenors falls under Section 4 [paragraph b] of R.A. 8249. Section 4 requires that the offense charged must be committed by the offender in relation to his office in order for the Sandiganbayan to have jurisdiction over it. 63 This jurisdictional requirement is in accordance with Section 5, Article XIII of the 1973 Constitution which mandated that the Sandiganbayan shall have jurisdiction over criminal cases committed by the public officers and employees, including those in goverment-owned or controlled corporations, "in relation to 6

their office as may be determined by law." This constitutional mandate was reiterated in the new (1987) Constitution when it declared in Section 4 thereof that the Sandiganbayan shall continue to function and exercise its jurisdiction as now or hereafter may be provided by law. ISSUE: Whether Sandiganbayan has a jurisdiction over the case. HELD: None. An offense is said to have been committed in relation to the office if it (the offense) is "intimately connected" with the office of the offender and perpetrated while he was in the performance of his official functions. This intimate relation between the offense charged and the discharge of official duties "must be alleged in the informations. The jurisdiction of a court is defined by the Constitution or statute. The elements of that definition must appear in the complaint or information so as to ascertain which court has jurisdiction over a case. Hence the elementary rule that the jurisdiction of a court is determined by the allegations in the complaint or informations, and not by the evidence presented by the parties at the trial. Consequently, for failure to show in the amended informations that the charge of murder was intimately connected with the discharge of official functions of the accused PNP officers, the offense charged in the subject criminal cases is plain murder and, therefore, within the exclusive original jurisdiction of the Regional Trial Court, not the Sandiganbayan.
GARCIA VS. SANDIGANBAYAN G.R. NO. 170122 OCTOBER 12, 2009

Facts: The Office of the Ombudsman filed for a petition for the forfeiture of the properties amounting to PhP 143,052,015.29 allegedly amassed by then Maj. Gen. Carlos Garcia, his wife Clarita and two children, docketed as Civil Case No. 0193 (Forfeiture Case 1). Another forfeiture case was subsequently filed to recover funds amounting to PhP 202,005,980.55 docketed as Civil Case No. 0196 (Forfeiture Case 2), raffled to the 4th Division. Prior to the filing of Forfeiture II, but subsequent to the filing of Forfeiture I, the OMB charged the Garcias and three others with violation of RA 7080 (plunder) which placed the value of the property and funds plundered at PhP 303,272,005.99, docketed as Crim. Case No. 28107, raffled to the Second Division of the Sandiganbayan. As per the Sheriffs return, the corresponding summons involving Forfeiture 1 were issued and all served on Gen. Garcia at his place of detention. The SB subsequently issued a writ of attachment in favor of the Republic. The Garcias filed a motion to dismiss on the ground of SBs lack of jurisdiction over separate civil actions for forfeiture. The SB denied the Motion to Dismiss and declared the Garcias in default. Despite the standing default order, the Garcias moved for the transfer and consolidation of Forfeiture I with the plunder case which were respectively pending in different divisions of the SB, contending that such consolidation is mandatory under RA 8249. This motion was denied by the SB. The Garcias filed another motion to dismiss and/or to quash Forfeiture I on, inter alia, the following grounds: (a) the filing of the plunder case ousted the SB 4th Division of jurisdiction over the forfeiture case; and (b) that the consolidation is imperative in order to avoid possible double jeopardy entanglements. The SB merely noted the motion.

As regards Forfeiture 2, the SB sheriff served the corresponding summons. In his return, the sheriff stated giving the copies of the summons to the OIC/Custodian of the PNPDetention Center who in turn handed them to Gen. Garcia. The general signed his receipt of the summons, but as to those pertaining to the other respondents, Gen. Garcia acknowledged receiving the same, but with the following qualifying note: Im receiving the copies of Clarita, Ian Carl, Juan Paolo & Timothy but these copies will not guarantee it being served to the above-named(sic). Issue: 1. Whether the SB has jurisdiction over petitioner despite improper service of summons. 2. Whether the SB has jurisdiction over the forfeiture case despite the filing of the plunder case. Held: 1. NO. It is basic that a court must acquire jurisdiction over a party for the latter to be bound by its decision or orders. Valid service of summons, by whatever mode authorized by and proper under the Rules, is the means by which a court acquires jurisdiction over a person. It is undisputed that summons for Forfeitures I and II were served personally on Maj. Gen. Carlos Flores Garcia, who is detained at the PNP DetentionCenter, who acknowledged receipt thereof by affixing his signature. It is also undisputed that substituted service of summons for both Forfeitures I and II were made on petitioner and her children through Maj. Gen. Garcia at the PNP Detention Center. However, such substituted services of summons were invalid for being irregular and defective. The requirements for a valid substituted service of summons are: (1) Impossibility of prompt personal service (2) Specific details in the return (3) Substituted service effected on a person of suitable age and discretion residing at defendants house or residence; or on a competent person in charge of defendants office or regular place of business. From the foregoing requisites, it is apparent that no valid substituted service of summons was made on petitioner and her children, as the service made through Maj. Gen. Garcia did not comply with the first two (2) requirements mentioned above for a valid substituted service of summons. Also, petitioners special appearance to question the courts jurisdiction is not voluntary appearance. Thus, a defendant who files a motion to dismiss, assailing the jurisdiction of the court over his person, together with other grounds raised therein, is not deemed to have appeared voluntarily before the court. 2. YES, Petitioners posture respecting Forfeitures I and II being absorbed by the plunder case, thus depriving the 4th Division of the SB of jurisdiction over the civil cases, is flawed by the assumptions holding it together, the first assumption being that the forfeiture cases are the corresponding civil action for recovery of civil liability ex delicto. As correctly ruled by the SB 4th Division in its May 20, 2005 Resolution, the civil liability for forfeiture cases does not arise from the commission of a criminal offense as such liability is based on a statute that safeguards the right of the State to recover unlawfully acquired properties. Secondly, a forfeiture case under RA 1379 arises out of a cause of action separate and different from a plunder case, thus negating the notion that the crime of plunder charged in Crim. Case No. 28107 absorbs the forfeiture cases. In a prosecution for plunder, what is sought to be established is the commission of the criminal acts in furtherance of the acquisition of ill-gotten wealth. On the other hand, all that the court needs to determine, by preponderance of evidence, under RA 1379

is the disproportion of respondents properties to his legitimate income, it being unnecessary to prove how he acquired said properties.
SANCHEZ VS. DEMETRIOU G.R. NOS. 111771-77 NOVEMBER 9, 1993

Facts: Mayor Antonio Sanchez was charged before the RTC of Calamba, Laguna of seven informations of homicide, in connection with the rape-slay of Mary Eileen Sarmenta and the killing of Allan Gomez. Sanchez moved to quash the information on the ground, among others that as a public officer, he can be tried for the offense only by the Sandiganbayan, among others. Issue: Whether it is the Sandiganbayan who has jurisdiction over petitioner. Held: NO. The petitioners contention that since most of the accused were incumbent public officials or employees at the time of the alleged commission of the crimes, the cases against them should come under the jurisdiction of the Sandiganbayan and not of the regular courts, is untenable. Section 4, paragraph (a) of P.D. No, 1606, as amended by P.D. No.1861, provides: The Sandiganbayan shall exercise: a) Exclusive original jurisdiction in all cases involving: (1) Violations of Republic Act No. 3019, as amended, otherwise known as the Anti-Graft and Corrupt Practices Act, Republic Act No. 1379, and Chapter II, Section 2, Title VII of the Revised Penal Code: (2) Other offenses or felonies committed by public officers and employees in relation to their office, including those employed in government-owned or controlled corporations, whether simple or complexed with other crimes, where the penalty prescribed by law is higher than prision correccional or imprisonment for six (6) years, or a fine of P6,000.00. . . . (Emphasis supplied) The crime of rape with homicide with which the petitioner stands charged obviously does not fall under paragraph (1), which deals with graft and corruption cases. Neither is it covered by paragraph (2) because it is not an offense committed in relation to the office of the petitioner. There is no direct relation between the commission of the crime of rape with homicide and the petitioner's office as municipal mayor because public office is not an essential element of the crime charged. The offense can stand independently of the office. Moreover, it is not even alleged in the information that the commission of the crime charged was intimately connected with the performance of the petitioner's official functions.
DE VERA VS. DE VERA G.R. NO. 172832 APRIL 7, 2009

Facts: Petitioner Rosario T. de Vera accused her spouse Geren A. de Vera and Josephine F. Juliano of Bigamy. Upon arraignment, Geren pleaded "Guilty." Subsequently, he prayed that he be allowed to withdraw his plea in the meantime in order to prove the mitigating circumstance of voluntary surrender. The motion was opposed by petitioner on the ground that not all the elements of the mitigating circumstance of "voluntary surrender" were present. She added that "voluntary surrender" was raised only as an afterthought, as Geren had earlier invoked a "voluntary plea of guilty" without raising the former. 9

Finally, she posited that since the case was ready for promulgation, Gerens motion should no longer be entertained. The RTC granted the motion and appreciated the mitigating circumstance of voluntary surrender. Petitioner moved for the reconsideration of the decision but was denied. For failure to obtain a favourable relief from the RTC, she instituted a special civil action of certiorari before the CA. However, the CA likewise maintained that all the requisites of voluntary surrender were present. Hence, the instant petition. Issue: Whether the lower courts erred in appreciating the mitigating circumstance of voluntary surrender as to merit the exercise of the SCs appellate judicial discretion. Held: NO. Section 7, Rule 120 of the Revised Rules of Criminal Procedure provides that a judgment of conviction may, upon motion of the accused, be modified or set aside before it becomes final or before appeal is perfected. Except where the death penalty is imposed, a judgment becomes final after the lapse of the period for perfecting an appeal, or when the sentence has been partially or totally satisfied or served, or when the accused has waived in writing his right to appeal, or has applied for probation. Simply stated, in judgments of conviction, errors in the decision cannot be corrected unless the accused consents thereto; or he, himself, moves for reconsideration of, or appeals from, the decision. While a petition for certiorari may be resorted to on jurisdictional grounds, that review is allowed only in apparently void judgments where there is a patent showing of grave abuse of discretion amounting to lack or excess of jurisdiction. The aggrieved parties, in such cases, must clearly show that the public respondent acted without jurisdiction or with grave abuse of discretion amounting to lack of jurisdiction. Obviously, no grave abuse of discretion may be attributed to a court simply because of its alleged misappreciation of the mitigating circumstance of voluntary surrender. Consequently, the trial courts action cannot come within the ambit of the writs limiting requirement of excess or lack of jurisdiction. Thus, the trial courts action becomes an improper object of, and therefore non -reviewable by, certiorari. In this case, it appears that the Information was filed with the RTC on February 24, 2005. On March 1, 2005, the court issued an Order finding probable cause for the accused to stand trial for the crime of bigamy and for the issuance of a warrant of arrest. In the afternoon of the same day, Geren surrendered to the court and filed a motion for reduction of bail. After the accused posted bail, there was no more need for the court to issue the warrant of arrest. The foregoing circumstances clearly show the voluntariness of the surrender. As distinguished from the earlier cases, upon learning that the court had finally determined the presence of probable cause and even before the issuance and implementation of the warrant of arrest, Geren already gave himself up, acknowledging his culpability. This was bolstered by his eventual plea of guilt during the arraignment. Thus, the trial court was correct in appreciating the mitigating circumstance of "voluntary surrender."

PEOPLE AND PHOTOKINA VS. BENIPAYO (G.R. NO. 155573 APRIL 24, 2009)

Facts: In the first libel case filed against him, Alfredo L. Benipayo, then Chairman COMELEC, delivered a speech in the "Forum on Electoral Problems: Roots and Responses in the Philippines" held at UP Diliman which was subsequently published in the Manila Bulletin. Petitioner corporation, believing that it was the one 10

alluded to by the respondent when he stated in his speech that Even worse, the Commission came right up to the brink of signing a 6.5 billion contract for a registration solution that could have been bought for 350 million pesos, and an ID solution that isnt even a requirement for voting. But reason intervened and no contract was signed. Now, they are at it again, trying to hoodwink us into contract that is so grossly disadvantageous to the government that it offends common sense to say that it would be worth the 6.5 billion-peso price tag., filed, through its authorized representative, an AffidavitComplaint for libel. The case was dismissed based on lack of jurisdiction since the offense was committed in relation to his office, hence vesting the jurisdiction on the Sandiganbayan. G.R. No. 155573 Respondent as COMELEC Chair was a guest of the talk show "Point Blank," hosted by Ces Drilon and televised nationwide on the ANC-23 channel. The television shows episode that day was entitled "COMELEC Wars.", where Respondent against discussed that Photokinas funds are being used to campaign against him. Another information for libel was instituted against Respondent but also dismissed by the RTC rationating that being an impeachable officer, the jurisdiction must be with the Sandiganbayan. Issue: Whether the RTC has jurisdiction over the crime of libel filed against Benipayo. Held: YES. The jurisdiction of the court to hear and decide a case is conferred by the law in force at the time of the institution of the action, unless a latter statute provides for a retroactive application thereof. Article 360 of the RPC, as amended by Republic Act No. 4363, is explicit on which court has jurisdiction to try cases of written defamations in providing that the criminal and civil action for damages in cases of written defamations as provided for in this chapter, shall be filed simultaneously or separately with the court of first instance [now, the Regional Trial Court] of the province or city where the libelous article is printed and first published or where any of the offended parties actually resides at the time of the commission of the offense. RA 7691 also did not divest the RTC of jurisdiction over libel cases because although it was enacted to decongest the clogged dockets of the RTCs by expanding the jurisdiction of first level courts, said law is of a general character. Even if it is a later enactment, it does not alter the provision of Article 360 of the RPC, a law of a special nature. Laws vesting jurisdiction exclusively with a particular court, are special in character, and should prevail over the Judiciary Act defining the jurisdiction of other courts (such as the Court of First Instance) which is a general law. A later enactment like RA 7691 does not automatically override an existing law, because it is a well-settled principle of construction that, in case of conflict between a general law and a special law, the latter must prevail regardless of the dates of their enactment. Jurisdiction conferred by a special law on the RTC must therefore prevail over that granted by a general law on the MTC. Since jurisdiction over written defamations exclusively rests in the RTC without qualification, it is unnecessary and futile for the parties to argue on whether the crime is committed in relation to office. Thus, the conclusion reached by the trial court that the respondent committed the alleged libelous acts in relation to his office as former COMELEC chair, and deprives it of jurisdiction to try the case, is, following the above disquisition, gross error.

11

FOZ, JR. VS. PEOPLE G.R. NO. 167764 OCTOBER 9, 2009

Facts: Danny Fajardo, a columnist of Panay News, wrote an article entitled Meet Dr. Portigo, Company Physician about Dr. Edgar Portigo, a company physician of a local branch of San Miguel Office. The said article conveyed how Dr. Portigo failed to discharge his sworn duty to look after the health of the employees. It contained a detailed sad plight of one Lita Payunan, wife of employee Wilfredo Payunan, who had complications while under the care of Dr. Portigo and the other doctors the latter recommended for Payunans health services. Subsequently, an Information was filed against Vicente Foz and Danny Fajardo, to wit: That on or about the 5th day of July, 1994 in the City of Iloilo, Philippines and within the jurisdiction of this court, both the accused as columnist and Editor-Publisher, respectively, of Panay News, a daily publication with a considerable circulation in the City of Iloilo and throughout the region, did then and there willfully, unlawfully and feloniously with malicious intent of impeaching the virtue, honesty, integrity and reputation of Dr. Edgar Portigo, a physician and medical practitioner in Iloilo City, and with the malicious intent of injuring and exposing said Dr. Edgar Portigo to public hatred, contempt and ridicule, write and publish in the regular issue of said daily publication on July 5, 1994, a certain article entitled MEET DR. PORTIGO, COMPANY PHYSICIAN, The RTC convicted both accused of libel, which was affirmed by the CA. Petitioners raise for the first time the issue that the information charging them with libel did not contain allegations sufficient to vest jurisdiction in the RTC of Iloilo City. Issue: Whether the information was sufficient to vest jurisdiction in the RTC of Iloilo. Held: NO. Venue in criminal cases is an essential element of jurisdiction. It is a fundamental rule that for jurisdiction to be acquired by courts in criminal cases the offense should have been committed or any one of its essential ingredients took place within the territorial jurisdiction of the court. Territorial jurisdiction in criminal cases is the territory where the court has jurisdiction to take cognizance or to try the offense allegedly committed therein by the accused. Thus, it cannot take jurisdiction over a person charged with an offense allegedly committed outside of that limited territory. Furthermore, the jurisdiction of a court over the criminal case is determined by the allegations in the complaint or information. And once it is so shown, the court may validly take cognizance of the case. Article 360 of the Revised Penal Code, as amended by Republic Act No. 4363, provides that xxx The criminal action and civil action for damages in cases of written defamations, as provided for in this chapter shall be filed simultaneously or separately with the court of first instance of the province or city where the libelous article is printed and first published or where any of the offended parties actually resides at the time of the commission of the offense:xxx Since Dr. Portigo is a private individual at the time of the publication of the alleged libelous article, the venue of the libel case may be in the province or city where the libelous article was printed and first published, or in the province where Dr. Portigo actually resided at the time of the commission of the offense. The allegations in the Information that Panay News, a daily publication with a considerable circulation in the City of Iloilo and throughout the region only showed that Iloilo was the place where Panay News was in considerable circulation but did not establish that the said publication was printed and first published in Iloilo City. While the Information alleges that Dr. Edgar Portigo is a physician and 12

medical practitioner in Iloilo City, such allegation did not clearly and positively indicate that he was actually residing in Iloilo City at the time of the commission of the offense. It is possible that Dr. Portigo was actually residing in another place.

PEOPLE VS. SANDIGANBAYAN G.R. NO. 173396, SEPTEMBER 22, 2010

FACTS: Private respondents were charged in an Information for Violation of Section 3(e) of Republic Act No. 3019. Private respondents were duly arraigned and pleaded not guilty to the charge against them. Thereafter, trial on the merits ensued. The Sandiganbayan declared the accused not guilty for the said offense charged. The People, represented by the Office of the Ombudsman, through the Office of the Special Prosecutor, then filed the present petition for certiorari. The aim of the present petition is to overturn the Sandiganbayan's conclusion that there is no doubt that dredging work was performed along the Palto and Pakulayo Rivers and the project was actually undertaken and accomplished by the said contractor x x x [h]ence the payment made to the latter was justified. From such finding, the trial court held that the prosecution failed to prove the presence of all the elements of the offense charged, resulting in the acquittal of private respondents. Petitioner points out that the lower court erred in arriving at such conclusion, since prosecution evidence shows that as of September 2, 1991 to October 2, 1991, when the dredging works were supposedly conducted, there was as yet no approved plans and specifications as required by PD No. 1594 before bidding for construction contracts can proceed. Petitioner doubts that the proper procedure for bidding had been followed. Petitioner then asks how the project could have proceeded on September 2, 1991 when the required plan was only dated November 18, 1991. ISSUE: Whether or not the acquittal of private respondents can still be reviewed by the Court. HELD: It is essentially an issue involving an alleged error of judgment, not an error of jurisdiction. There being no mistrial in this case, the acquittal of private respondents can no longer be reviewed by the Court as this would constitute a violation of the constitutional right against double jeopardy. Moreover, since the alleged error is only one of judgment, petitioner is not entitled to the extraordinary writ of certiorari. The petition is DISMISSED for lack of merit. The Decision of the Sandiganbayan is AFFIRMED.

RAMISCAL, JR. VS.SANDIGANBAYAN G.R. NOS. 172476-99 SEPTEMBER 15, 2010

Facts: Petitioner Jose S. Ramiscal, Jr. was a retired officer of the Armed Forces of the Philippines (AFP), with the rank of Brigadier General, when he served as President of the AFP-Retirement and Separation Benefits System (AFP-RSBS). During petitioners term as president of AFP-RSBS, the Board of Trustees of AFP-RSBS approved the acquisition of land situated in General Santos City for development as housing projects. AFP-RSBS, represented by petitioner, and Atty. Nilo J. Flaviano, as attorney-in-fact of the 12 individual vendors, executed and signed bilateral deeds of sale over the subject property. Petitioner forthwith caused the payment to the individual vendors. Subsequently, Flaviano executed and signed unilateral 13

deeds of sale over the same property. The unilateral deeds of sale reflected a purchase price of only P3,000.00 per square meter instead of the actual purchase price of P10,500.00 per square meter. Flaviano presented the unilateral deeds of sale for registration. The unilateral deeds of sale became the basis of the transfer certificates of title issued by the Register of Deeds of General Santos City to AFPRSBS. Luwalhati R. Antonino, the Congresswoman representing the first district of South Cotabato, which includes General Santos City, filed in the Ombudsman a complaint-affidavit against petitioner, along with 27 other respondents, for (1) violation of Republic Act No. 3019, otherwise known as the Anti-Graft and Corrupt Practices Act; and (2) malversation of public funds or property through falsification of public documents. After preliminary investigation, the Ombudsman, found petitioner probably guilty of violation of Section 3(e) of RA 3019 and falsification of public documents The Ombudsman filed in the Sandiganbayan 12 informations for violation of Section 3(e) of RA 3019 and 12 informations for falsification of public documents against petitioner and several other co-accused. Petitioner filed his first motion for reconsiderati on with a supplemental motion of the Ombudsmans finding of probable cause against him. The Sandiganbayan disposed of petitioners first motion for reconsideration. The Office of the Special Prosecutor (OMB-OSP) recommended that petitioner be excluded from the informations. On review, the Office of Legal Affairs (OMB-OLA), recommended the contrary, stressing that petitioner participated in and affixed his signature on the contracts to sell, bilateral deeds of sale, and various agreements, vouchers, and checks for the purchase of the subject property. The memoranda of OMB-OSP and OMB-OLA were forwarded for comment to the Office of the Ombudsman for Military (OMB-Military).The OMB-Military adopted the memorandum of OMB-OSP recommending the dropping of petitioners name from the informations. Acting Ombudsman Margarito Gervacio approved the recommendation of the OMB-Military. However, the recommendation of the OMB-Military was not manifested before the Sandiganbayan as a final disposition of petitioners first motion for reconsideration. After thorough review, the panel of prosecutors found that petitioner indeed participated in and affixed his signature on the contracts to sell, bilateral deeds of sale, and various agreements, vouchers, and checks for the purchase of the property. Ombudsman Ma. Merceditas N. Gutierrez approved the recommendation of the panel of prosecutors. Upon receipt of the final findings of the Ombudsman, the Sandiganbayan scheduled the arraignment of petitioner. Meanwhile, petitioner filed his second motion for reconsideration of the Ombudsmans finding of probable cause against him. Petitioner was arraigned. For his refusal to enter a plea, the Sandiganbayan entered in his favor a plea of not guilty. Petitioner filed a motion to set aside his arraignment pending resolution of his second motion for reconsideration of the Ombudsmans finding of probable cause against him. ISSUE: Whether or not the Sandiganbayan commit grave abuse of discretion when it denied petitioners motion to set aside his arraignment pending resolution of his second motion for reconsideration of the Ombudsmans finding of probable cause against him. HELD: 14

Significantly, while it is the Ombudsman who has the full discretion to determine whether or not a criminal case should be filed in the Sandiganbayan, once the case has been filed with said court, it is the Sandiganbayan, and no longer the Ombudsman, which has full control of the case. In this case, petitioner failed to establish that the Sandiganbayan committed grave abuse of discretion amounting to lack or excess of jurisdiction when it denied petitioners motion to set aside his arraignment. There is grave abuse of discretion when power is exercised in an arbitrary, capricious, whimsical, or despotic manner by reason of passion or personal hostility so patent and gross as to amount to evasion of a positive duty or virtual refusal to perform a duty enjoined by law. Absent a showing of grave abuse of discretion, this Court will not interfere with the Sandiganbayans jurisdiction and control over a case properly filed before it. The Sandiganbayan is empowered to proceed with the trial of the case in the manner it determines best conducive to orderly proceedings and speedy termination of the case. There being no showing of grave abuse of discretion on its part, the Sandiganbayan should continue its proceedings with all deliberate dispatch.
PEOPLE VS. JACK RACHO G.R. NO. 186529 AUGUST 3, 2010

FACTS: A confidential agent of the police transacted through cellular phone with appellant for the purchase of shabu. The police authorities immediately formed a team to apprehend the appellant. The agent gave the police appellants name, together with his physical description. He also assured them that appellant would arrive in Baler, Aurora the following day. Appellant called up the agent and informed him that he was on board a Genesis bus and would arrive in Baler, Aurora, anytime of the day wearing a red and white striped T-shirt. The team members then posted themselves. A Genesis bus arrived in Baler. Having alighted from the bus, appellant stood near the highway and waited for a tricycle that would bring him to his final destination. As appellant was about to board a tricycle, the team approached him and invited him to the police station on suspicion of carrying shabu. Appellant immediately denied the accusation, but as he pulled out his hands from his pants pocket, a white envelope slipped therefrom which, when opened, yielded a small sachet containing the suspected drug. The team then brought appellant to the police station for investigation. The confiscated specimen was turned over to a Police Inspector who marked it with his initials and with appellants name. The field test and laboratory examinations on the contents of the confiscated sachet yielded positive results for methamphetamine hydrochloride. Appellant was charged in two separate Informations, one for violation of Section 5 of R.A. 9165, for transporting or delivering; and the second, of Section 11 of the same law for possessing, dangerous drugs. During the arraignment, appellant pleaded "Not Guilty" to both charges. At the trial, appellant denied liability and claimed that he went to Baler, Aurora to visit his brother to inform him about their ailing father. As to the circumstances of his arrest, he explained that the police officers, through their van, blocked the tricycle he was riding in; forced him to alight; brought him to Sea Breeze Lodge; stripped his clothes and underwear; then brought him to the police station for investigation. The RTC rendered a Joint Judgment convicting appellant and sentencing him to suffer the penalty of life imprisonment and to pay a fine of P500,000.00; but acquitted him of the charge of Violation of Section 11, Article II, R.A. 9165. On appeal, the CA affirmed the RTC decision. Hence, the present appeal.

15

ISSUE: Whether or not jurisdiction over the person o the accused has been acquired. HELD: The records show that appellant never objected to the irregularity of his arrest before his arraignment. In fact, this is the first time that he raises the issue. Considering this lapse, coupled with his active participation in the trial of the case, we must abide with jurisprudence which dictates that appellant, having voluntarily submitted to the jurisdiction of the trial court, is deemed to have waived his right to question the validity of his arrest, thus curing whatever defect may have attended his arrest. The legality of the arrest affects only the jurisdiction of the court over his person. Appellants warrantless arrest therefore cannot, in itself, be the basis of his acquittal. Appellant Jack Raquero Racho is ACQUITTED for insufficiency of evidence.
MIAQUE VS. PATAG G.R. NOS. 170609-13 JANUARY 30, 2009

FACTS: Five Informations for libel were filed in the RTC against petitioner Bernie G. Miaque and three others. These Informations were quashed for lack of jurisdiction over the offenses charged. Specifically, said Informations failed to allege either that private respondent (therein private complainant) Vicente Aragona actually held office in Iloilo City at the time of the commission of the offenses or that the alleged libelous remarks were printed or first published in Iloilo City. The Asst. Provincial Prosecutor issued a resolution recommending the filing of Informations which was accordingly filed in the RTC presided by Judge Patag. The new Informations were similarly worded as those previously quashed but with these added allegations: (1) Aragona, Regional State Prosecutor VI of the Department of Justice, held office at the Hall of Justice, Iloilo City or (2) the alleged libelous remarks were written, printed and published in Iloilo City (on the pertinent dates thereof). Said Informations were likewise signed and filed by Assistant Provincial Prosecutor Maraon. Petitioner filed his motions not to issue warrants of arrest and, if already issued, to recall them and remand the Informations to the Provincial Prosecutors Office for preliminary investigation. Respondent judge denied petitioners motions on the ground that petitioner was beyond the courts jurisdiction as he was not under the custody of the court. Petitioners motion for reconsideration was denied. Hence, this petition. ISSUE: Whether or not the Iloilo Provincial Prosecutors Office has the authority to file and sign the new Informations against petitioner. HELD: The authority to sign and file the new Informations is properly lodged with the Iloilo City Prosecutors Office. The Iloilo Provincial Prosecutors Office was clearly bereft of authority to file the new Informations against petitioner. An Information, when required by law to be filed by a public prosecuting officer, cannot be filed by another. The court does not acquire jurisdiction over the case because there is a defect in the Information.

16

Questions relating to lack of jurisdiction may be raised at any stage of the proceeding. An infirmity in the information, such as lack of authority of the officer signing it, cannot be cured by silence, acquiescence, or even by express consent. The foregoing considered, the Informations corresponding to Criminal Case Nos. 05-61407 to 05-61411 were fatally defective. The common infirmity in the Informations constituted a jurisdictional defect that could not be cured. There was no point in proceeding under a defective Information that could never be the basis of a valid conviction. The orders of the Regional Trial Court of Iloilo City, are hereby REVERSED AND SET ASIDE. Criminal Case Nos. 05-61407 to 05-61411 are DISMISSED WITHOUT PREJUDICE to the filing of new Informations by an authorized officer. The warrants of arrest issued are likewise QUASHED.
BALTAZAR VS. CHUA G.R. NO. 177583

FACTS: Jaime and Jovito were charged before the RTC Manila with the crimes of homicide and frustrated homicide for the death of Ildefonso Baltazar and the wounding of Edison Baltazar. Petitioners Lourdes Baltazar (Lourdes) and Edison Baltazar (Edison), through counsel, filed a motion for reinvestigation of the cases, praying that Jaime and Jovito be charged with the crimes of murder and frustrated murder, instead of homicide and frustrated homicide. The City Prosecutors Office, upon reinvestigation, found that the appropriate charges against Jaime and Jovito were murder and frustrated murder. Jaime and Jovito appealed to the DOJ. The Secretary of the DOJ, modified the resolution of the City Prosecutor by directing the latter to amend the Informations for Murder and Frustrated Murder to Homicide and Frustrated Homicide against Jovito and to drop Jaime from the charges. Lourdes and Edison filed a motion for reconsideration but was denied. In obedience to the directive of the Secretary of the DOJ, the City Prosecutor filed with the RTC a Manifestation and Motion for the Withdrawal of the Informations for Murder and Frustrated Murder and for the Admission of New Informations for Homicide and Frustrated Homicide. Judge Cruz granted the said manifestation. Lourdes and Edison moved for reconsideration. Lourdes and Edison filed before Judge Cruz a Motion to Maintain the Amended Informations for Murder and Frustrated Murder which was denied on the ground that the same was, in effect, a second motion for reconsideration. Judge Hidalgo, after making his own assessment granted the motion and ordered the reinstatement of the informations for murder and frustrated murder. Jaime and Jovito filed a motion for reconsideration. Judge Hidalgo denied the said motion. Jaime then filed a petition for certiorari and prohibition with the Court of Appeals. The Court of Appeals granted Jaimes petition and nullified the Order of Judge Hidalgo, ruling that the same were issued in grave abuse of discretion amounting to excess of jurisdiction. Aggrieved, Lourdes and Edison filed the instant petition. ISSUE: Whether Judge Hidalgo may review the finding of the Secretary of Justice on the existence or nonexistence of probable cause sufficient to hold Jaime for trial and substitute his judgment for that of the Secretary of Justice.

17

HELD: YES. The rule is that once an information is filed in court, any disposition of the case, be it dismissal, conviction, or acquittal of the accused, rests on the sound discretion of the court. Reliance on the resolution of the Secretary of Justice alone is considered an abdication of the trial courts duty and jurisdiction to determine a prima facie case. While the ruling of the Justice Secretary is persuasive, it is not binding on courts. The trial court is not bound by the Resolution of the Justice Secretary, but must evaluate it before proceeding with the trial. Considering that the trial court has the power and duty to look into the propriety of the prosecutions motion to dismiss, with much more reason is it for the trial court to evaluate and to make its own appreciation and conclusion, whether the modification of the charges and the dropping of one of the accused in the information, as recommended by the Justice Secretary, is substantiated by evidence. This should be the state of affairs, since the disposition of the case -- such as its continuation or dismissal or exclusion of an accused -- is reposed in the sound discretion of the trial court.
PEOPLE VS. TEODORO G.R. NO. 172372 DECEMBER 4, 2009

FACTS: The prosecution charged the accused Romar Teodoro before the RTC of Batangas of the crime of rape under three separate Informations. First, Criminal Case No. 8538, which states that on or about the 18th day of June, 1995 the accused by means of force and intimidation, willfully, unlawfully and feloniously lie with and have carnal knowledge with the said [AAA] who is below twelve (12) years old. Second, Criminal Case No. 8539 which states that the carnal knowledge was committed sometime in the first week of July, 1995 and Third, Criminal Case No. 8540 which states that the carnal knowledge happened on the 30th day of March 1996. The appellant pleaded not guilty. Upon trial, AAA declared on the witness stand that she knew the appellant since 1993 because Teodoro was an employee of her parents. AAA recalled that on June 18, 1995, Teodoro went to the bathroom and kissed her on the face and neck. He then removed her clothes, pants and panty. Thereafter, the appellant took off his pants and inserted his penis into her vagina. AAA likewise recalled that during the first week of July 1995, the appellant again "raped" her in the bathroom as well as in morning of March 30, 1996. On the other hand, Teodoro claimed that AAA had been his sweetheart since June 22, 1996 and denied using force against her. The RTC convicted the appellant of two (2) counts of statutory rape. The accused, however, is acquitted in Criminal Case No. 8540, as this Court finds him innocent of the crime charged. CA affirmed the RTC decision which dismissed Teodoros argument that the Information in Criminal Case No. 8539 was vague and insufficient because the exact date of the crime was not stated. ISSUE: Whether or not the Information in Criminal Case No. 8539 is defective because it failed to state the exact date of the commission of the crime HELD: NO. An information, under Section 6, Rule 110 of the 2000 Revised Rules on Criminal Procedure, is deemed sufficient if it states the name of the accused; the designation of the offense given by the statute; the acts or omissions complained of as constituting the offense; the name of the offended party; the approximate date of the commission of the offense; and the place where the offense was committed. Section 11 of the same Rule also provides that it is not necessary to state in the complaint or 18

information the precise date the offense was committed, except when the date of commission is a material element of the offense. The offense may thus be alleged to have been committed on a date as near as possible to the actual date of its commission. At the minimum, an indictment must contain all the essential elements of the offense charged to enable the accused to properly meet the charge and duly prepare for his defense.
LEVISTE VS. ALAMEDA G.R. NO. 182677 AUGUST 3, 2010

FACTS: Jose Antonio C. Leviste (petitioner) was charged with with homicide for the death of Rafael de las Alas on January 12, 2007 before the RTC of Makati City. The private complainants-heirs of De las Alas filed, an Urgent Omnibus Motion praying, for the deferment of the proceedings to allow the public prosecutor to re-examine the evidence on record or to conduct a reinvestigation to determine the proper offense. The RTC thereafter issued the Order deferring petitioners arraignment and allowing the prosecution to conduct a reinvestigation to determine the proper offense. The trial court nonetheless issued the other order that admitted the Amended Information for murder and directed the issuance of a warrant of arrest. Petitioner questioned these two orders before the appellate court. Upon arraignment, petitioner refused to plead, drawing the trial court to enter a plea of "not guilty" for him. The trial court went on to try petitioner under the Amended Information. The trial court found petitioner guilty of homicide sentencing him to suffer an indeterminate penalty of six years and one day of prision mayor as minimum to 12 years and one day of reclusion temporal as maximum. From the Decision, petitioner filed an appeal to the appellate court which was denied by the CA. Hence, appeal. ISSUE: Whether or not the amendment of the Information from homicide to murder is considered a substantial amendment, which would make it not just a right but a duty of the prosecution to ask for a preliminary investigation. HELD: YES. A substantial amendment consists of the recital of facts constituting the offense charged and determinative of the jurisdiction of the court. All other matters are merely of form. The test as to whether a defendant is prejudiced by the amendment is whether a defense under the information as it originally stood would be available after the amendment is made, and whether any evidence defendant might have would be equally applicable to the information in the one form as in the other. There is no substantial distinction between a preliminary investigation and a reinvestigation since both are conducted in the same manner and for the same objective of determining whether there exists sufficient ground to engender a well-founded belief that a crime has been committed and the respondent is probably guilty thereof and should be held for trial. What is essential is that petitioner was placed on guard to defend himself from the charge of murder after the claimed circumstances were made known to him as early as the first motion. Petitioner did not, however, make much of the opportunity to present countervailing evidence on the proposed amended charge. Despite notice of hearing, petitioner opted to merely observe the proceedings and declined to actively participate, even with extreme caution, in the reinvestigation.

19

CAPISTRANO VS. LIMCUANDO G.R. NO. 152413 FEBRUARY 13, 2009

FACTS: Barceliza P. Capistrano owned a parcel of land located at Barangay Talaga, Rizal, Laguna. She sold the land with a right of repurchase in favor of Spouses Felimon Zuasola and Anita. On February 1, 1989, petitioner sold half of the same parcel of land to respondents for the price of P75,000.00 on the understanding that respondents shall pay the amount of P10,000.00 as partial payment and the balance to be paid by monthly instalments. Petitioner received the partial payment of P10,000.00 but signed a deed of absolute sale, denominated as "Kasulatan ng Bilihang Tuluyan," disposing half of the property in favor of respondents. Respondents learned afterwards that the disputed land had been previously sold by the petitioner to the spouses Zuasola and Subida which led respondents to file a criminal complaint for estafa against petitioner. Petitioner repurchased the parcel of land from the spouses Zuasola and Subida. She also offered to repurchase from respondents the portion of the disputed land which she sold to them but the latter refused. Petitioner filed a complaint for the annulment of the subject deed of sale alleging that the sale was a nullity from the beginning and that respondents even assailed its validity in the previously mentioned criminal case for estafa against petitioner. In their Answer with Counterclaim, respondents claim that they never assailed the validity of the subject deed of sale in the estafa case. RTC ruled the filing of Information for Estafa against plaintiff is a criminal action which cannot properly be considered as a basis for the annulment of a Deed of Absolute Sale. The annulment of the Deed of Absolute Sale should be ventilated in a separate civil action that needs preponderance of evidence for the purpose. CA affirmed the judgment of the RTC. Hence, this petition ISSUE: Whether or not an action for annulment of deed of sale is deemed impliedly instituted with criminal action for estafa HELD: The theory of petitioner that the respondents should be deemed to have themselves assailed the validity of the subject deed of sale, since the civil aspect of the criminal case for estafa was impliedly instituted with the filing of said criminal action, is bereft of legal basis. The civil action which is deemed impliedly instituted with the criminal action is the recovery of indemnity or damages under the Revised Penal Code and specifically enumerated articles of the Civil Code. The action to annul the subject deed of sale is obviously not among the civil actions that are deemed impliedly instituted with the criminal action. Thus, respondents active participation in the prosecution of petitioner for the crime of estafa, as well as their concession that fraud attended the execution of the said deed of sale, would have significance only as to the recovery of civil indemnity arising from the said crime. The trial court did not err when it held that the action to annul the deed of sale should be ventilated in a separate civil action, notwithstanding petitioners conviction in the criminal action.
ALBERT VS SANDIGANBAYAN 580 SCRA 279

FACTS: Special Prosecution Officer II of the Office of the Ombudsman for Mindanao charged the accused in violation of Section 3(e) R.A. 3019. It alleged in the information that RAMON A. ALBERT, a public officer, the President of the National Home Mortgage and Finance Corporation with salary grade of above 27 acted with evident bad faith and manifest partiality and or gross neglect of duty , enter and make it 20

appear in Tax Declaration that two parcels of real property are residential lands when in truth and in fact, the two pieces of real property agricultural land, and by reason of accuseds misrepresentation, the NHMFC released the amount which is higher than the loanable amount the land could command being agricultural, thus causing undue injury to the government. Sandiganbayan issued a Hold Departure Order against petitioner and his co-accused. Petitioner filed a Motion to Dismiss. Pending the resolution of the Motion to Dismiss, petitioner filed a Motion to Lift Hold Departure Order and to be Allowed to Travel. The prosecution did not object to the latter motion on the condition that petitioner would be "provisionally" arraigned. Sandiganbayan arraigned petitioner who entered a plea of "not guilty." Sandiganbayan denied petitioners Motion to Dismiss and ordered the prosecution to conduct a reinvestigation. On 7 October 2003, the prosecution filed a Motion for Leave to Admit Amended Information which replaced gross neglect of duty" with "gross inexcusable negligence. Petitioner opposed the motion, alleging that the amendment made on the information is substantial and, therefore, not allowed after arraignment. Sandiganbayan granted the prosecutions Motion to Admit Amended Information. It ruled that even granting that the amendment of the information be formal or substantial, the prosecution could still effect the same in the event that the accused had not yet undergone a permanent arraignment. Petitioner filed a Motion for Reconsideration, which was denied by the Sandiganbayan. Hence, this petition. ISSUE: Whether or not the replacement of gross neglect of duty" with "gross inexcusable negligence" is a substantial amendment of the Information which is prejudicial to his rights HELD: NO. The test as to when the rights of an accused are prejudiced by the amendment of a complaint or information is when a defense under the complaint or information, as it originally stood, would no longer be available after the amendment is made, and when any evidence the accused might have, would be inapplicable to the complaint or information as amended. On the other hand, an amendment which merely states with additional precision something which is already contained in the original information and which, therefore, adds nothing essential for conviction for the crime charged is an amendment to form that can be made at anytime. In this case, the amendment entails the deletion of the phrase "gross neglect of duty" from the Information. Although this may be considered a substantial amendment, the same is allowable even after arraignment and plea being beneficial to the accused. As a replacement, "gross inexcusable negligence" would be included in the Information as a modality in the commission of the offense.
YAP VS. CABALES G.R. NO. 159186 JUNE 5, 2009

FACTS: Petitioner Jesse Y. Yap and his spouse Bessie Yap are engaged in the real estate business through their company Primetown Property Group. Yap purchased several real properties from a certain Evelyn Te. In consideration of said purchases, petitioner issued several BPI postdated checks to Evelyn. Thereafter, spouses Orlando and Mergyl Mirabueno and spouses Charlie and Jovita Dimalanta, rediscounted the checks from Evelyn. Some of the checks were dishonour by reason of account closed. Despite of the demand, Yap failed to pay the amounts represented by the said checks. Spouses Mirabueno filed a civil action for collection of sum of money against Yap. Subsequently, the Office of the City Prosecutor of General Santos City filed several informations for violation of BP 22 against the petitioner. In the 21

criminal cases, Yap filed separate motions to suspend proceedings on account of the existence of a prejudicial question. The MCTC denied the motions for lack of merit. On appeal, the RTC likewise denied the petition. CA rendered a Decision dismissing the petition for lack of merit. The CA opined that Civil Case Nos. 6231 and 6238 did not pose a prejudicial question to the prosecution of the petitioner for violation of B.P. Blg. 22. Hence, this appeal. ISSUE: Whether or not there exists a prejudicial question that necessitates the suspension of the proceedings in the MTCC. HELD: None. A prejudicial question generally exists in a situation where a civil action and a criminal action are both pending, and there exists in the former an issue that must be preemptively resolved before the latter may proceed, because howsoever the issue raised in the civil action is resolved would be determinative juris et de jure of the guilt or innocence of the accused in the criminal case. The rationale behind the principle of prejudicial question is to avoid two conflicting decisions. It has two essential elements: (i) the civil action involves an issue similar or intimately related to the issue raised in the criminal action; and (ii) the resolution of such issue determines whether or not the criminal action may proceed. If both civil and criminal cases have similar issues, or the issue in one is intimately related to the issues raised in the other, then a prejudicial question would likely exist, provided the other element or characteristic is satisfied. It must appear not only that the civil case involves the same facts upon which the criminal prosecution would be based, but also that the resolution of the issues raised in the civil action would be necessarily determinative of the guilt or innocence of the accused. If the resolution of the issue in the civil action will not determine the criminal responsibility of the accused in the criminal action based on the same facts, or if there is no necessity that the civil case be determined first before taking up the criminal case, the civil case does not involve a prejudicial question. Neither is there a prejudicial question if the civil and the criminal action can, according to law, proceed independently of each other.
PIMENTEL V. PIMENTEL G.R. NO. 172060

Facts: This is a petition for review assailing the Decision of the Court of Appeals, promulgated on 20 March 2006, in CA-G.R. SP No. 91867. On 25 October 2004, Maria Chrysantine Pimentel y Lacap (private respondent) filed an action for frustrated parricide against Joselito R. Pimentel (petitioner), docketed as Criminal Case No. Q-04130415, before the Regional Trial Court of Quezon City, which was raffled to Branch 223 (RTC Quezon City). On 7 February 2005, petitioner received summons to appear before the Regional Trial Court of Antipolo City, Branch 72 (RTC Antipolo) for the pre-trial and trial of Civil Case No. 04-7392 (Maria Chrysantine Lorenza L. Pimentel v. Joselito Pimentel) for Declaration of Nullity of Marriage under Section 36 of the Family Code on the ground of psychological incapacity.

22

Petitioner asserted that since the relationship between the offender and the victim is a key element in parricide, the outcome of Civil Case No. 04-7392 would have a bearing in the criminal case filed against him before the RTC Quezon City. The court stated that the pendency of the case before the RTC Antipolo is not a prejudicial question that warrants the suspension of the criminal case before it Petitioner herein appealed the case before the Court of Appeals which affirmed the decision of the lower court Issue: Whether or not the resolution of the action for annulment of marriage is a prejudicial question that warrants the suspension of the criminal case for frustrated parricide against the petitioner. Held. NO. The rule is clear that the civil action must be instituted first before the filing of the criminal action. As in the case before, the civil action for annulment of marriage was filed after the filing of the criminal complaint for frustrated parricide. Also, there is a prejudicial question when a civil action and a criminal action are both pending, and there exists in the civil action an issue which must be preemptively resolved before the criminal action may proceed because howsoever the issue raised in the civil action is resolved would be determinative of the guilt or innocence of the accused in the criminal case. A prejudicial question is defined as: x x x one that arises in a case the resolution of which is a logical antecedent of the issue involved therein, and the cognizance of which pertains to another tribunal. It is a question based on a fact distinct and separate from the crime but so intimately connected with it that it determines the guilt or innocence of the accused, and for it to suspend the criminal action, it must appear not only that said case involves facts intimately related to those upon which the criminal prosecution would be based but also that in the resolution of the issue or issues raised in the civil case, the guilt or innocence of the accused would necessarily be determined.

HEIRS OF SIMON V. CHAN G.R. NO. 157547

Facts: There is no independent civil action to recover the civil liability arising from the issuance of an unfunded check prohibited and punished under Batas Pambansa Bilang 22 (BP 22). Respondent herein, filed a case at the City Prosecutors Office of Manila a case of B.P. 22 against the petitioner, by issuing a LandBank check No. 0007280 worth 366,000.00. After sometime, respondent herein filed another case against petitioner for civil damages arising from the same transactions at Pasay City. Petitioner herein filed in Pasay a Motion to Dismiss due to litis pendencia, at which instance, the Pasay Court dismissed the case against the petitioner. Appeal was likewise denied the the lower court. The Court of Appeals upheld the ruling of the lower court.

23

Issue: Whether or not the institution of a criminal case (B.P. 22), the complainant can reserved his/her right to file a civil case. Held. NO. Regardless, therefore, of whether or not a special law so provides, indemnification of the offended party may be had on account of the damage, loss or injury directly suffered as a consequence of the wrongful act of another Section 1. Institution of criminal and civil actions. - (a) When a criminal action is instituted, the civil action for the recovery of civil liability arising from the offense charged shall be deemed instituted with the criminal action unless the offended party waives the civil action, reserves the right to institute it separately or institutes the civil action prior to the criminal action. The reservation of the right to institute separately the civil action shall be made before the prosecution starts presenting its evidence and under circumstances affording the offended party a reasonable opportunity to make such reservation. Except as otherwise provided in these Rules, no filing fees shall be required for actual damages. No counterclaim, cross-claim or third-party complaint may be filed by the accused in the criminal case, but any cause of action which could have been the subject thereof may be litigated in a separate civil action. (1a) (b) The criminal action for violation of Batas Pambansa Blg. 22 shall be deemed to include the corresponding civil action. No reservation to file such civil action separately shall be allowed. To repeat, Chans separate civil action to recover the amount of the check involved in the prosecution for the violation of BP 22 could not be independently maintained under both Supreme Court Circular 5797 and the provisions of Rule 111 of the Rules of Court, notwithstanding the allegations of fraud and deceit
METROBANK V. YAO G.R. NO. 180165

Facts: This is a petition for review on certiorari under Rule 45 of the Rules of Court, assailing the decision of the Secretary of Justice and of the Court of Appeals in dismissing the complaint file herein by the petitioner as against Oliver Yao and Diana Yao. Private respondents obtained from herein petitioner 24 letters of credit in the amount of P68,749,487.96. Upon maturity of the said amount, respondents were not able to pay as such, a complaint was filed. In the said complaint, herein petitioner insisted that the loan was a trust receipt transaction, which was denied by respondents, as they argued that it was a mortgaged loan.

24

Upon approval of the City Fiscal that the case be filed against respondents, they appeal ed the decision of the City Prosecutor to the Secretary of Justice, which in turn ordered that the Prosecutor file a motion to dismiss the case as there was no probable cause to prosecute private respondent for estafa in relation to P.D. 115 RTC then granted the withdrawal of the case. The Motion for Reconsideration was likewise denied by the lower court. The Court of Appeals likewise dismiss the petition by herein appellant as it stated that the appellate court recognized the authority of the Secretary of Justice to control and supervise the prosecutors, which includes the power to reverse or modify their decisions without committing grave abuse of discretion. Issue: Whether or not the Secretary of Justice can order the dismissal of the complaint after the court has already acquired jurisdiction over it. Held: No. The conduct of preliminary investigation is executive in nature. The Court may not be compelled to pass upon the correctness of the exercise of the public prosecutors function unless there i s a showing of grave abuse of discretion or manifest error in his findings. In the present case, the abuse of discretion is patent in the act of the Secretary of Justice holding that the contractual relationship forged by the parties was a simple loan, for in so doing, the Secretary of Justice assumed the function of the trial judge of calibrating the evidence on record, done only after a full-blown trial on the merits. The fact of existence or non-existence of a trust receipt transaction is evidentiary in nature, the veracity of which can best be passed upon after trial on the merits, for it is virtually impossible to ascertain the real nature of the transaction involved based solely on the selfserving allegations contained in the opposing parties pleadi ngs. The public prosecutor merely determines whether there is probable cause or sufficient ground to engender a well-founded belief that a crime has been committed, and that the respondent is probably guilty thereof and should be held for trial. A trial is needed to allow the both parties to present their evidences and to substantiate their respective claims.
ALAWIYA VS. DATUMANONG G.R. NO. 164170 APRIL 16, 2009

Facts: Petitioners charged Policemen Martin, Medina, Asis, Gutierrez, Paz and Berida, Jr. with kidnapping for ransom. In their sworn statements, petitioners alleged that on September 11 2001, while they were cruising on board a vehicle along United Nations Avenue, a blue Toyota Sedan bumped their vehicle from behind; that when they went out of their vehicle to assess the damage, several armed men alighted from the Toyota Sedan, poked guns at, blindfolded, and forced them to ride in the Toyota Sedan; that they were brought to an office where P10,000,000 and two vehicles were demanded from 25

them in exchange for their freedom; that, after haggling, the amount was reduced to P700,000 plus the two vehicles; that the money and vehicles were delivered in the late evening of 11 September 2001; that they were released in the early morning of 12 September 2001 in Quiapo after they handed the Deed of Sale and registration papers of the two vehicles. The Philippine National Police Intelligence Group in Camp Crame (PNP-NCR-RID) recommended that accused be charged with violation of Article 267 of the Revised Penal Code, as amended by Republic Act No. 7659. State Prosecutor Emmanuel Y. Velasco, who conducted the preliminary investigation, issued a Resolution recommending that the accused be indicted for the crime of kidnapping for ransom. Subsequently, an Information was filed against the policemen. In a Resolution dated 24 September 2002, then Secretary of Justice Hernando B. Perez reversed the ruling of Velasco and ordered the latter to cause the withdrawal or dismissal of the Information for kidnapping for ransom. He ruled that there was no prior approval by the Office of the Ombudsman before the Information for kidnapping was filed with the trial court. He also found that the incident complained of was a bungled buy-bust operation, not kidnapping for ransom. On 11 October 2002, petitioners filed a Motion for Reconsideration, which was denied by then Secretary of Justice Simeon A. Datumanong in a Resolution promulgated on 17 February 2003. The Court of Appeals upheld the Secretary of Justices ruling that prior approval by the Office of the Ombudsman for the Military was needed for the filing of the Information before the RTC, pursuant to OMB-DOJ Joint Circular No. 95-001. Hence, this petition. Issue: Whether the prior approval by the Office of the Ombudsman for the Military is required for the investigation and prosecution of the instant case against the accused. Held: NO. The Office of the Solicitor General (OSG), which is representing the Secretary of Justice, agrees with petitioners that prior approval by the Ombudsman is not required for the investigation and prosecution of the criminal case against the accused policemen. The OSG correctly cites the case of Honasan II v. The Panel of Investigating Prosecutors of the Department of Justice, where the Court held that the power of the Ombudsman to investigate offenses involving public officers or employees is not exclusive but is concurrent with other similarly authorized agencies of the government such as the provincial, city and state prosecutors. In view of the foregoing, both the Court of Appeals and the Secretary of Justice clearly erred in ruling that prior approval by the Ombudsman is required for the investigation and prosecution of the criminal case against the accused policemen. Case remanded to the RTC for independent evaluation or assessment of the merits of the case to determine whether probable cause exists to hold the accused for trial.

26

ENRIQUE V. VIUDEZ II vs.THE COURT OF APPEALS G.R. No. 152889 June 5, 2009

Facts: Two Informations for murder were filed against Enrique Viudez II for the killing of Mayor Honorato Galvez with the RTC of Malolos, Bulacan, which then issued warrants of arrest on the same day. Viudez filed a Motion to Suspend Proceedings and to Suspend the Implementation of the Warrant of Arrest, Pursuant to Department Circular No. 70 of the Department of Justice (DOJ) arguing that he had filed a timely petition for review with the Secretary of Justice. Pursuant to Section 9 of Department Circular No. 70, the implementation of the warrant of arrest against petitioner should be suspended and/or recalled pending resolution of the said petition for review. The RTC denied Viudezs Motion stating that, insofar as the implementation of the warrant of arrest against petitioner was concerned, said warrant had already been issued for his apprehension. The court also added that there was no way for it to recall the same in the absence of any compelling reason, and that jurisdiction over his person had not yet been acquired by it; hence, petitioner had no personality to file any pleading in court relative to the case until he was arrested or voluntarily surrendered himself to the court. Viudezs filed with the CA a petition for certiorari with prayer for the issuance of a temporary restraining order (TRO) and/or writ of preliminary injuction which was dismissed for lack of merit. Issue: Whether a pending resolution of a petition for review filed with the Secretary of Justice concerning a finding of probable cause will suspend the proceedings in the trial court, including the implementation of a warrant of arrest. Held: No. It is well to remember that there is a distinction between the preliminary inquiry, which determines probable cause for the issuance of a warrant of arrest and the preliminary investigation proper, which ascertains whether the offender should be held for trial or be released. The determination of probable cause for purposes of issuing a warrant of arrest is made by the judge. The preliminary investigation proper whether or not there is reasonable ground to believe that the accused is guilty of the offense charged is the function of the investigating prosecutor. The function of the judge to issue a warrant of arrest upon the determination of probable cause is exclusive. Thus, the consequent implementation of a warrant of arrest cannot be deferred pending the resolution of a petition for review by the Secretary of Justice as to the finding of probable cause, a function that is executive in nature. To defer the implementation of the warrant of arrest would be an encroachment on the exclusive prerogative of the judge. The DOJ Circular No. 70 is directed specifically at the appellant and the trial prosecutor, giving them latitude in choosing a remedy to ensure that the proceedings in court are held in abeyance. Nowhere in the said provision does it state that the court must hold the proceedings in abeyance. Therefore, the discretion of the court whether or not to suspend the proceedings or the implementation of the warrant of arrest, upon the motion of the appellant or the trial prosecutor, remains unhindered.

27

PEOPLE VS. CASTILLO G.R. No. 171188 June 19, 2009

FACTS: A task force from the Bacoor Municipal Hall effected the closure of the stalls through the installation of galvanized iron fences. Lessees of the stalls thereafter filed before the Office of the Ombudsman a complaint against respondent Jessie B. Castillo, in his capacity as Bacoor Municipal Mayor, respondent Mejia. The Ombudsman ruled that respondents acted in good faith in closing the stalls. Sarino filed a Complaint against respondents Castillo and Mejia before the Office of the Ombudsman charging them criminally and administratively for oppression, grave misconduct and for committing acts contrary to law. Respondents countered that Sarinos complaint was anchored on the same set of facts that was dismissed by the Ombudsman. Ombudsman dismissed the administrative complaint for being moot and academic. The Office of the Ombudsman, through the Office of the Special Prosecutor, filed an Information against respondents before the Sandiganbayan. The Sandiganbayan declared that probable cause exists against and it directed the issuance of the corresponding warrants of arrest and hold departure orders against respondents. Respondents voluntarily surrendered to the Sandiganbayan and posted their respective bonds for their provisional liberty. Respondents moved for the reinvestigation of the case which the Sandiganbayan gave due course. An Amended Information was admitted. Sandiganbayan denied the respondents Motion for Judicial Determination of Probable Cause. The Sandiganbayan, upon motion for reconsideration filed by respondents, reversed its Resolution and dismissed the case. The Sandiganbayan likewise set aside the arrest warrants it previously issued. It held that the instant criminal case is a mere rehash of the previously dismissed criminal case filed by complainants lessees against respondents. It also ruled that there was no evident bad faith, manifest partiality or inexcusable negligence that can be attributed to respondents. Neither did complainants claim of undue injury have any leg to stand on. The Office of the Special Prosecutor filed a motion for reconsideration, but it was denied ISSUE: WON SB erred in overturning the Ombudsmans determination of probable cause resulting in the dismissal of the case against respondents. HELD: YES. There are two kinds of determination of probable cause: executive and judicial. The executive determination of probable cause is one made during preliminary investigation. It is a function that properly pertains to the public prosecutor who is given a broad discretion to determine whether probable cause exists and to charge those whom he believes to have committed the crime as defined by law and thus should be held for trial. Otherwise stated, such official has the quasi-judicial authority to determine whether or not a criminal case must be filed in court. Whether or not that function has been correctly discharged by the public prosecutor, i.e., whether or not he has made a correct ascertainment of the existence of probable cause in a case, is a matter that the trial court itself does not and may not be compelled to pass upon.

28

The judicial determination of probable cause, on the other hand, is one made by the judge to ascertain whether a warrant of arrest should be issued against the accused. The judge must satisfy himself that based on the evidence submitted, there is necessity for placing the accused under custody in order not to frustrate the ends of justice. If the judge finds no probable cause, the judge cannot be forced to issue the arrest warrant. Corollary to the principle that a judge cannot be compelled to issue a warrant of arrest if he or she deems that there is no probable cause for doing so, the judge in turn should not override the public prosecutors determination of probable cause to hold an accused for trial on the ground that the evidence presented to substantiate the issuance of an arrest warrant was insufficient. It must be stressed that in our criminal justice system, the public prosecutor exercises a wide latitude of discretion in determining whether a criminal case should be filed in court, and that courts must respect the exercise of such discretion when the information filed against the person charged is valid on its face, and that no manifest error or grave abuse of discretion can be imputed to the public prosecutor. Thus, absent a finding that an information is invalid on its face or that the prosecutor committed manifest error or grave abuse of discretion, a judges determination of probable cause is limited only to the judicial kind or for the purpose of deciding whether the arrest warrants should be issued against the accused. It was clearly premature on the part of the Sandiganbayan to make a determinative finding prior to the parties presentation of their respective evidence that there was no bad faith and manifest partiality on the respondents part and undue injury on the part of the complainant. In Go v. Fifth Division, Sandiganbayan, we held that "it is well established that the presence or absence of the elements of the crime is evidentiary in nature and is a matter of defense that may be best passed upon after a full-blown trial on the merits." Also, it would be unfair to expect the prosecution to present all the evidence needed to secure the conviction of the accused upon the filing of the information against the latter. The reason is found in the nature and objective of a preliminary investigation. Here, the public prosecutors do not decide whether there is evidence beyond reasonable doubt of the guilt of the person charged; they merely determine whether there is sufficient ground to engender a well-founded belief that a crime has been committed and that respondent is probably guilty thereof, and should be held for trial.
HEIRS OF THE LATE NESTOR TRIA V. ATTY. EPIFANIA OBIAS G.R. NO. 175887 NOV. 24, 2010

FACTS: Engr. Nestor Tria, RD of DPWH Region V was shot by a gunman while waiting to board his flight to Manila. He was brought to a hospital but died the following day from the lone gunshot wound on his nape. NBI Regional Director Alejandro R. Tenerife, Chairman of Task Force Tria, recommended to the Provincial Prosecutor of Camarines Sur the indictment of Obet Aclan, Totoy Ona and Atty. Epifania Fanny Gonzales-Obias, for the murder of Engr. Tria. During the preliminary investigation respondent filed her Counter-Affidavit denying that she was in anyway involved with the killing of Engr. Tria and further asserted that from the totality of evidence gathered by the NBI, it has not established prima facie the existence of conspiracy as to implicate her in the death of Engr. Tria. The Prosecutor issued a resolution directing the filing of an information for murder against Aclan and Ona but dismissed the case for insufficiency of evidence as against Obias. Justice Secretary Cuevas issued a Resolution directing the Prosecutor to include Obias in the information. The DOJ was convinced

29

that the sequence of events and respondents conduct before, during and after the killing of Engr. Tria undeniably points to her complicity with Aclan and Ona. The DOJ denied respondents motion for reconsideration stating that the proper procedure is the filing of an appeal or petition for review with the OP and not before the DOJ. Hence, the case was considered closed and terminated. OP dismissed the murder charge, CA affirmed OPs decision. ISSUE: Whether or not the CA gravely abused uts discretion in affirming the OPs dismissal of the murder charge. HELD: NO. In arguing that the CA gravely abused its discretion when it affirmed the OPs dismissal of th e murder charge against respondent, petitioner invoked our ruling in Crespo v. Mogul that any disposition of the case rests on the sound discretion of the court once an information has been filed with it. A refinement of petitioners understanding of the Crespo ruling is in order. In Crespo, we ruled that after the information has already been filed in court, the courts permission must be secured should the fiscal find it proper that reinvestigation be made. Thereafter, the court shall consider and act upon the findings and recommendations of the fiscal. In Ledesma v. Court of Appeal we clarified that the justice secretary is not precluded from exercising his power of review over the investigating prosecutor even after the information has already been filed in court. However, the justice secretarys subsequent resolution withdrawing the information or dismissing the case does not cause the court to lose jurisdiction over the case. In fact, the court is dutybound to exercise judicial discretion and its own independent judgment in assessing the merits of the resulting motion to dismiss filed by the prosecution. In resolving the issue of whether the CA gravely abused its discretion in affirming the OPs reversal of the ruling of the Secretary of Justice, it is necessary to determine whether probable cause exists to charge the respondent for conspiracy in the murder of Engr. Tria. A prosecutor, by the nature of his office, is under no compulsion to file a particular criminal information where he is not convinced that he has evidence to prop up its averments, or that the evidence at hand points to a different conclusion. The decision whether or not to dismiss the criminal complaint against respondent is necessarily dependent on the sound discretion of the investigating prosecutor and ultimately, that of the Secretary of Justice. The findings of the prosecutor with respect to the existence or non-existence of probable cause is subject to the power of review by the DOJ. Indeed, the Secretary of Justice may reverse or modify the resolution of the prosecutor, after which he shall direct the prosecutor concerned either to file the corresponding information without conducting another preliminary investigation, or to dismiss or move for dismissal of the complaint or information with notice to the parties. Ordinarily, the determination of probable cause is not lodged with this Court. Its duty in an appropriate case is confined to the issue of whether the executive or judicial determination, as the case may be, of probable cause was done without or in excess of jurisdiction or with abuse of discretion amounting to want of jurisdiction.

30

However, this Court may ultimately resolve the existence or non-existence of probable cause by examining the records of the preliminary investigation when necessary for the orderly administration of justice, or to avoid oppression or multiplicity of action. To begin with, whether or not respondent actually conspired with Aclan and Ona need not be fully resolved during the preliminary investigation. The absence or presence of conspiracy is factual in nature and involves evidentiary matters. The same is better left ventilated before the trial court during trial, where the parties can adduce evidence to prove or disprove its presence. Preliminary investigation is executive in character. It does not contemplate a judicial function. It is essentially an inquisitorial proceeding, and often, the only means of ascertaining who may be reasonably charged with a crime. Prosecutors control and direct the prosecution of criminal offenses, including the conduct of preliminary investigation, subject to review by the Secretary of Justice. The duty of the Court in appropriate cases is merely to determine whether the executive determination was done without or in excess of jurisdiction or with grave abuse of discretion. Resolutions of the Secretary of Justice are not subject to review unless made with grave abuse.
BORLONGAN, JR VS PENA GR NO. 143591

Facts: Respondent Magdaleno Pea instituted a civil case for recovery of agents compensation and expenses, damages, and attorneys fees, against Urban Bank and the petitioners. Respondent anchored his claim for compensation on the contract of agency, allegedly entered into with the petitioners wherein the former undertook to perform such acts necessary to prevent any intruder and squatter from unlawfully occupying Urban Banks property located along Roxas Boulevard, Pasay City. Petitioners filed a Motion to Dismiss arguing that they never appointed the respondent as agent or counsel. Attached to the MD were the following documents: 1. A letter signed by Herman Ponce and Julie Abad on behalf of ISCI, the original owner of the subject property; 2. An unsigned letter addressed to Corazon Bejasa from MarilynG. Ong; 3. A letter addressed to Teodoro Borlongan and signed byMarilyn G. Ong; and 4. A Memorandum from Enrique Montilla III.- The above stated documents were presented in an attempt to show that the respondent was appointed as agent by ISCI and not by Urban Bank or by the petitioners. Respondent Pea filed his Complaint-Affidavit with the Office of the City Prosecutor, Bago City. He claimed that said documents were falsified because the alleged signatories did not actually affix their signatures, and the signatories were neither stockholders nor officers and employees of ISCI. Worse, petitioners introduced said documents as evidence before the RTC knowing that they were falsified. The Prosecutor concluded that the petitioners were probably guilty of 4 counts of the crime of Introducing Falsified Documents penalized by the second paragraph of Article 172 of the RPC. The City Prosecutor concluded that the documents were falsified because the alleged signatories untruthfully stated that ISCI was the principal of the respondent; that petitioners knew that the documents were falsified considering that the signatories were mere dummies; and that the documents formed part of the record of Civil Case No. 754 where they were used by petitioners as evidence in support of their motion to dismiss, adopted in their answer and later, in their Pre- Trial Brief. Subsequently, the corresponding Informations were filed with the MTCC, Bago City. Thereafter, Judge Primitivo Blanca issued the warrants for the arrest of the petitioners. 31

Petitioners filed an Omnibus MQ : They insist that they were denied due process because of the nonobservance of a proper procedure on preliminary investigation prescribed inthe Rules of Court; since no such counter-affidavit and supporting documents were submitted by the petitioners, the trial judge merely relied on the complaint-affidavit and attachments of the respondent in issuing the warrants of arrest, also in contravention of the Rules. Moreover they claim that the respondents affidavit was not based on the latters personal knowledge and therefore should not have been used by the court in determining probable cause. On the same day that the Omnibus MQ was filed, the petitioners posted bail. Their bail bonds expressly provided that they do not intend to waive their right to question the validity of their arrest. On the date of arraignment, the petitioners refused to enter their plea, for the obvious reason that the legality of their information and their arrest was yet to be settled by the court. MTCCs upheld the validity of the warrant of arrest, saying that it was issued in accordance with the Rules. Besides, petitioners could no longer question the validity of the warrant since they already posted bail. Issue: WON petitioners were deprived of their right to due process of law because of the denial of their right to preliminary investigation and to submit their counter-affidavit and WON the Informations charging the petitioners were validly filed and the warrants for their arrest were properly issued; Held: YES, Records show that the prosecutor relied merely on the affidavits submitted by the complainant and did not require the petitioners to submit their answer. He should not be faulted for doing such as this is sanctioned by the rules. Moreover, he is not mandated to require the submission of counter-affidavits. Probable cause may then be determined on the basis alone of the affidavits and supporting documents of the complainant, without infringing on the constitutional rights of the petitioners Petitioners contend that the warrants were illegally issued as they were solely based on the affidavits of the complainant. Section 2 of Article III of the Constitution underscores the exclusive and personal responsibility of the issuing judge to satisfy himself of the existence of probable cause. But the judge is not required to personally examine the complainant and his witnesses. Following established doctrine and procedure, he shall (1) personally evaluate the report and the supporting documents submitted by the prosecutor regarding the existence of probable cause, and on the basis thereof, he may already make a personal determination of the existence of probable cause; and (2) if he is not satisfied that probable cause exists, he may disregard the prosecutors report and require the submission of supporting affidavits of witnesses to aid him in arriving at a conclusion as to the existence of probable cause. There is no provision or procedural rule which makes the submission of counter-affidavits mandatory before the judge could determine probable cause. What he is never allowed to do is to follow blindly the prosecutor's bare certification as to the existence of probable cause. Much more is required by the constitutional provision. Judges have to go over the report, the affidavits, the transcript of stenographic notes if any, and other documents supporting the prosecutor's certification. Although the extent of the judge's personal examination depends on the circumstances of each case, to be sure, he cannot just rely on the bare certification alone but must go beyond it. This is because the warrant of arrest issues not on the strength of the certification standing 32

alone but because of the records which sustain it. He should even call for the complainant and the witnesses to answer the court's probing questions when the circumstances warrant. For the issuance of a warrant of arrest, probable cause has been defined as the existence of such facts and circumstances that would lead a reasonably discreet and prudent person to believe that an offense has been committed by the person sought to be arrested. It is one of the requisites for a warrant of arrest to be valid. An arrest without a probable cause is an unreasonable seizure of a person, and violates the privacy of persons, which ought not to be intruded by the State.

PEOPLE VS. GREY G.R. NO. 180109 JULY 26, 2010

FACTS: Mayor Jojo Grey, his son Francis Grey and the others were charge of the crime Murder which resulted in a filing of a counter-charge of Perjury for the death of Rolando Diocton. Judge Bandal was the the presiding judge who denied the motion for issuance of warrant of arrest due to insufficiency but later on inhibit herself and denied the motion or reconsideration. Respondent then filed his own petition for the change of venue alleging that the one who took over the case was a pawn. Judge Natividad who was then the presiding judge proceeded with the preliminary investigation and issued warrant of arrest against the respondent. Consequently, respondent filed a petition alleging that petitioner gravely abuse his discretion for issuing order and for seeking a TRO. CA then issued a permanent TRO, ordering a warrant of arrest but it was set aside and the case was dismissed. Petitioner argued that respondent committed forum shopping which would warrant the outright dismissal of their petition. ISSUE: Whether or not Judge Natividad gravely abuse his discretion for issuing the order and warrant of arrest. HELD: NO, Judge Natividad did not gravely abuse his discretion in issuing a warrant of arrest. His personal determination revealed no improper motive on the part of the prosecution and no circumstance which would overwhelm the presumption of regularity in the performance of official functions. Judge Natividad complied with constitution to personally determine the complainant and witnesses for probable cause before issuing the warrant of arrest. The Court ruled that personal determination of complainant and witnesses is not mandatory and indispensable in determination of probable cause for the issuance of warrant of arrest. The necessity arises only when there is an utter failure of the evidence to show the existence of probable cause. Otherwise, the judge may rely on the report of the investigating prosecutor, provided that he likewise evaluates the documentary evidence in support thereof.
PEOPLE OF THE PHILIPPINES VS. GADIANA G.R. NO. 184761 SEPTEMBER 8, 2010

FACTS: RTC Branch 15 Judge Fortunato de Gracia sentenced Gadiana to suffer imprisonment of up to 12 years and one day and a fine of P300,000 after he was found guilty beyond reasonable doubt for possession of 33

two small packs of shabu when he was arrested last February 7, 2004. When the accused appealed the decision before the Court of Appeals, the appellate court affirmed the RTC decision and modified the maximum penalty by increasing it to 14 years imprisonment. The arresting officers namely PO1 Julius Busico, PO3 Joseph Dinauanao and PO2 Erwin Ferrer claimed that they personally saw Gadiana holding the two packs of shabu preparing to pocket it while they were patrolling at Sitio Tromar. Busico testified that it was he who recovered the shabu from Gadiana and it was Ferrer who prepared the letter request for examination of the confiscated white crystalline powder to the PNP Crime Laboratory Service.During the trial, Busico said that Ferrer was the one who took the shabu packs to the PNP Crime Laboratory where it was found positive of shabu, the street name of methamphetamine hydrochloride. Appellant maintains that his guilt was not proven beyond reasonable doubt. ISSUE: Whether or not the warrantless arrest was valid. HELD: NO. Parenthetically, appellants arrest, not to mention resulting confiscation of the alleged confiscation of the plastic sachets of crystalline substances in his possession, leaves nagging doubts on its validity in light of the fact that what PO1 Busico merely saw was appellants placing of the plas tic sachets in his pocket does not justify his warrantless arrest under the Rules. Section 5 of Rule 113 of the Rules of Court provide: A peace officer or a private person may, without a warrant, arrest a person: (a) When, in his presence, the person to be arrested has committed, is actually committing, or is attempting to commit an offense; (b) When an offense has just been committed and he has probable cause to believe based on personal knowledge of facts or circumstances that the person to be arrested has committed it; and (c) When the person to be arrested is a prisoner who has escaped from a penal establishment or place where he is serving final judgment or is temporarily confined while his case is pending, or has escaped while being transferred from one confinement to another. In cases falling under paragraphs (a) and (b) above, the person arrested without a warrant shall be forthwith delivered to the nearest police station or jail and shall be proceeded against in accordance with Section 7 of Rule 112.

PEOPLE VS. NG YIK BUN 639 SCRA 639 SCRA

Facts: On August 24, 2000, at around 9:00 p.m., Capt. Danilo Ibon of Task Force Aduana received information from an operative that there was an ongoing shipment of contraband in Barangay Bignay II, Sariaya, Quezon Province. Upon instructions from his superior, Major Carlo Magno Tabo, Capt. Ibon formed a team in coordination with a Philippine National Police detachment, and, along with the operative, the team then proceeded to Villa Vicenta Resort in Barangay Bignay II, Sariaya. The members of the team were able to observe the goings-on at the resort from a distance of around 50 meters. They spotted six Chinese-looking men loading bags containing a white substance into a white van. Having been noticed, Capt. Ibon identified his team and asked accused-appellant Chua Shilou Hwan (Hwan) what they were loading on the van. Hwan replied that it was shabu and pointed, when probed 34

further, to accused-appellant Raymond Tan as the leader. A total of 172 bags of suspected shabu were then confiscated. Bundles of noodles (bihon) were also found on the premises. Based on the aforementioned, an information was filed against Hwan, Tan, Ng Yik Bun, Cheng, Shi and Min for violation of Section 16, Article III of Republic Act No. (RA) 6425 or the Dangerous Drugs Act of 1972. The trial court and the CA convicted all of the accused. The accused raised that there was no valid warrantless arrest Issue: Whether there was a valid warrantless arrest? Held: Yes. Sec. 5. Arrest without warrant; when lawful. A peace officer or a private person may, without a warrant, arrest a person: a) When, in his presence, the person to be arrested has committed, is actually committing, or is attempting to commit an offense; (Emphasis supplied.) The foregoing proviso refers to arrest in flagrante delicto. In the instant case, contrary to accusedappellants contention, there was indeed a valid warrantless arrest in flagrante delicto. Consider the circumstances immediately prior to and surrounding the arrest of accused-appellants: (1) the police officers received information from an operative about an ongoing shipment of contraband; (2) the police officers, with the operative, proceeded to Villa Vicenta Resort in Barangay Bignay II, Sariaya, Quezon; (3) they observed the goings-on at the resort from a distance of around 50 meters; and (4) they spotted the six accused-appellants loading transparent bags containing a white substance into a white L300 van. In People v. Alunday, we held that when a police officer sees the offense, although at a distance, or hears the disturbances created thereby, and proceeds at once to the scene, he may effect an arrest without a warrant on the basis of Sec. 5(a), Rule 113 of the Rules of Court, as the offense is deemed committed in his presence or within his view. In the instant case, it can plausibly be argued that accused-appellants were committing the offense of possessing shabu and were in the act of loading them in a white van when the police officers arrested them. As aptly noted by the appellate court, the crime was committed in the presence of the police officers with the contraband, inside transparent plastic containers, in plain view and duly observed by the arresting officers. And to write finis to the issue of any irregularity in their warrantless arrest, the Court notes, as it has consistently held, that accused-appellants are deemed to have waived their objections to their arrest for not raising the issue before entering their plea. PEOPLE VS. UYBOCO 640 SCRA Facts: Accused along with 4 others were charged with kidnapping with ransom. Accused allegedly kidnapped and detained JESON KEVIN DICHAVES, JESON KIRBY DICHAVES and NIMFA CELIZ against their will and without their consent. The incident allegedly took place while the abovementioned were riding the familys Isuzu car along Claro M. Recto Ave. While waiting for Yusan Dichaves, Acon, their driver, drove along Bilibid Viejo, Sampaloc. When the vehicle passed by in front of San Sebastian Church, a stainless jeep with two men and one woman described as a tomboy on board, suddenly blocked its way. One of the men, who was in police uniform accosted Acon and accused him of hitting the son of a Presidential 35

Security Group (PSG) General apparently with a stone when the vehicle ran over it. Acon denied the charges but he was transferred to the stainless jeep while the man in police uniform drove the Isuzu car. The tomboy sat next to Nimfa who then had Jeson Kirby sit on her lap while Jeson Kevin was sitting on the tomboys lap. They were brought to a house in Merville Subdivision, Paraaque. Accused allegedly communicated with Jepson, father of the siblings and employer of Nimfa, via phone call, asking for money. He initially demanded 26m but Jepson explained that he did not have such amount. They ultimately agreed for the amount of 1.3m with jewelry and a pistol. Accused allegedly instructed Jepson that the drop off would at Magallanes Commercial Center. Jepson was told to leave the trunk of his car unlocked and to place the money, jewelry and pistol on the same. Unknown to the accused, Jepson already asked the help of then VP Joseph Estrada for help. He then instructed Gen. Panfilo Lacson to rescue the victims and to arrest the kidnappers. On the day of the supposed pay off, P/Insp. Escandor was assigned to proceed to Magallanes Commercial Center, together with two other police officers. They reached the place at 3:30 p.m. and positioned themselves in front of the Maranao Arcade located at Magallanes Commercial Center. He brought a camera to cover the supposed pay-off. He took a total of 24 shots. Also, P/Supt. Chan was one of the team leaders dispatched also at Magallanes Commercial Center, took a video coverage on the supposed pay-off. He witnessed the pay-off and identified appellant as the one who took the bag containing the ransom money from the car trunk of Jepson. The accused were later apprehended by the task force assigned in Fort Bonifacio. The policemen who apprehended the accused were told that the accused were going towards their direction while riding a red Nissan sentra. The policemen were able to block off the vehicle of the accused and when they tried to approached, accused brought a .38 caliber pistol. A scuffle allegedly took place where the accused was pacified and he was ordered to open the trunk of their car, the money, jewelry and pistol were found. The trial court and CA found the accused guilty beyond reasonable doubt. Issue: Whether the arrest was valid? Held: Yes. SEC. 5. Arrest without warrant; when lawful. A peace officer or a private person may, without a warrant, arrest a person: (a) When, in his presence, the person to be arrested has committed, is actually committing, or is attempting to commit an offense; (b) When an offense has in fact been committed and he has personal knowledge of facts indicating that the person to be arrested has committed it; and, (c) When the person to be arrested is a prisoner who has escaped from a penal establishment or place where he is serving final judgment or temporarily confined while his case is pending, or has escaped while being transferred from one confinement to another. (Emphasis supplied) The second instance of lawful warrantless arrest covered by paragraph (b) cited above necessitates two stringent requirements before a warrantless arrest can be effected: (1) an offense has just been committed; and (2) the person making the arrest has personal knowledge of facts indicating that the person to be arrested has committed it.56

36

Records show that both requirements are present in the instant case. The police officers present in Magallanes Commercial Center were able to witness the pay-off which effectively consummates the crime of kidnapping. They all saw appellant take the money from the car trunk of Jepson. Such knowledge was then relayed to the other police officers stationed in Fort Bonifacio where appellant was expected to pass by. Personal knowledge of facts must be based on probable cause, which means an actual belief or reasonable grounds of suspicion. The grounds of suspicion are reasonable when, in the absence of actual belief of the arresting officers, the suspicion that the person to be arrested is probably guilty of committing the offense is based on actual facts, i.e., supported by circumstances sufficiently strong in themselves to create the probable cause of guilt of the person to be arrested. A reasonable suspicion, therefore, must be founded on probable cause, coupled with good faith on the part of the peace officers making the arrest. Section 5, Rule 113 of the 1985 Rules on Criminal Procedure does not require the arresting officers to personally witness the commission of the offense with their own eyes.57 It is sufficient for the arresting team that they were monitoring the pay-off for a number of hours long enough for them to be informed that it was indeed appellant, who was the kidnapper. This is equivalent to personal knowledge based on probable cause.

PICO VS. COMBONG 215 SCRA 421

Facts: An Information charging one Eddie Villegas with the murder of Father Narciso M. Pico. The Provincial Fiscal recommended that no bail be granted. A warrant was issued by respondent Judge. In this warrant, the words "no bail recommended" were typed in on the appropriate space. Complainant Pico, brother of the deceased, went to the RTC Carlota and discovered, to his surprise, that accused Villegas had been granted bail and had been released. Complainant charged the respondent Combong with serious and grave abuse of discretion for having granted bail to the accused, who had been charged with an offense punishable by reclusion perpetua, without notice and hearing and even before the accused had been arrested or detained. In his Comment, Respondent Judge explained, denied that he had granted the application for bail even prior to the arrest or detention of the accused. He, however, admits having failed to hold a hearing on the application for bail. Respondent Judge contends that on the day the motion for bail was filed, he as Judge had jurisdiction over the same, even if the accused had not personally surrendered to the court and even if the return of the warrant of arrest had not yet been filed in court. He claims that his failure to require defense counsel to show proof that the accused had been taken into custody by the police authorities, was due to oversight on his part, and that he had not been motivated by any illegal or immoral consideration when he granted bail and ordered the release of accused Villegas. He had deliberarely omitted holding a hearing because he had been fully convinced that the possibility of the accused jumping bail "was practically nil." Issue: Whether the grant of bail was proper? Held: 37

No. A person applying for admission to bail must be in the custody of the law or otherwise deprived of his liberty. A person who has not submitted himself to the jurisdiction of the court has no right to invoke the processes of that court. Respondent Judge should have diligently ascertained the whereabouts of the applicant and that he indeed had jurisdiction over the body of the accused before considering the application for bail. It is well-settled that an application for bail from a person charged with a capital offense (now an offense punishable by reclusion perpetua) must be set for hearing at which both the prosecution and the defense must be given a reasonable opportunity to prove (in the case of the prosecution) that evidence of guilt of the applicant is strong, or (in the case of the defense) that such evidence of guilt was not strong. In the instant case, where the offense charged is murder and punishable by reclusion perpetua or death, respondent Judge's deliberate failure to set the application for bail hearing effectively deprived the People of its right to due process. Granting the application for bail and fixing the amount thereof, absent any taking of evidence as to whether or not the guilt of the accused was strong, constitutes arbitrary, capricious and whimsical action. Such inexcusable conduct reflects either gross ignorance of the law or a cavalier disregard of its requirements. Respondent Judge's alleged impression that the probability of flight on the part of the accused was "practically nil," was obviously not based on evidence of record; he had no right to act on the basis of such merely personal impression. He himself had issued the warrant of arrest stating that no bail was recommended and then, inexplicably, without any evidence being presented to support the application for bail, released the accused on bail of P50,000.00.
GOVT. OF THE USA VS. PURGANAN 389 SCRA 623

Facts: The Government of the USA represented by the DOJ filed a Petition for Exrtradition against Mark Jimenez. the RTC could act on the Petition, Respondent Jimenez filed before it an "Urgent Manifestation/Ex-Parte Motion," which prayed that petitioners application for an arrest warrant be set for hearing. The RTC granted the motion. After the hearing, the court a quo required the parties to submit their respective memoranda. In his Memorandum, Jimenez sought an alternative prayer: that in case a warrant should issue, he be allowed to post bail in the amount of P100,000. Thereafter, the court below issued the assailed order, directing the issuance of a warrant for his arrest and fixing bail for his temporary liberty at one million pesos in cash. Said order gave rise to this Petition. Issue: Whether or not he is entitled to bail and to provisional liberty while the extradition proceedings are pending. Held: No. We agree with petitioner. As suggested by the use of the word "conviction," the constitutional provision on bail quoted above, as well as Section 4 of Rule 114 of the Rules of Court, applies only when a person has been arrested and detained for violation of Philippine criminal laws. It does not apply to extradition proceedings, because extradition courts do not render judgments of conviction or acquittal. Moreover, the constitutional right to bail "flows from the presumption of innocence in favor of every accused who should not be subjected to the loss of freedom as thereafter he would be entitled to 38

acquittal, unless his guilt be proved beyond reasonable doubt." 60 It follows that the constitutional provision on bail will not apply to a case like extradition, where the presumption of innocence is not at issue. The provision in the Constitution stating that the "right to bail shall not be impaired even when the privilege of the writ of habeas corpus is suspended" does not detract from the rule that the constitutional right to bail is available only in criminal proceedings. It must be noted that the suspension of the privilege of the writ of habeas corpus finds application "only to persons judicially charged for rebellion or offenses inherent in or directly connected with invasion." 61 Hence, the second sentence in the constitutional provision on bail merely emphasizes the right to bail in criminal proceedings for the aforementioned offenses. It cannot be taken to mean that the right is available even in extradition proceedings that are not criminal in nature. That the offenses for which Jimenez is sought to be extradited are bailable in the United States is not an argument to grant him one in the present case. To stress, extradition proceedings are separate and distinct from the trial for the offenses for which he is charged. He should apply for bail before the courts trying the criminal cases against him, not before the extradition court.
ZUO VS. CABEBE 444 SCRA 382

Facts: Petitioner is a Chief State Prosecutor. He filed an information against several policemen for illegal possession of prohibited or regulated drugs was filed. The accused in said case all pleaded not guilty. The accused filed a motion to dismiss invoking as ground the right of the accused to a speedy trial. The respondent judge, instead of acting on the motion, motu propio issued an Order granting bail to the accused, fixing the bail for each at P70,000.00 in cash or property bond at P120,000.00, except for accused Evelyn Manuel whose bail was fixed at P20,000.00 in cash. Respondent judge issued the Order without the accused's application or motion for bail. The prosecution then filed a motion for reconsideration. Instead of acting thereon, respondent judge issued an order inhibiting himself from further proceeding with the case, realizing that what he did was patently irregular. Complainant thus prays that respondent judge be dismissed from the service with forfeiture of all benefits and be disbarred from the practice of law. Respondent Judge, in his Comment, while admitting that he issued the Order dated November 5, 2002 granting bail to the accused without any hearing, "the same was premised on the constitutional right of the accused to a speedy trial." There was delay in the proceedings due to complainant's frequent absences and failure of the witnesses for the prosecution to appear in court, resulting in the cancellation of the hearings. Issue: Whether the bail was granted properly? Held: No. In Cortes vs. Catral, we laid down the following rules outlining the duties of the judge in case an application for bail is filed:

39

1. In all cases whether bail is a matter of right or discretion, notify the prosecutor of the hearing of the application for bail or require him to submit his recommendation (Section 18, Rule 114 of the Revised Rules of Criminal Procedure); 2. Where bail is a matter of discretion, conduct a hearing of the application for bail regardless of whether or not the prosecution refuses to present evidence to show that the guilt of the accused is strong for the purpose of enabling the court to exercise its sound discretion (Section 7 and 8, id.); 3. Decide whether the guilt of the accused is strong based on the summary of evidence of the prosecution; 4. If the guilt of the accused is not strong, discharge the accused upon the approval of the bail bond (Section 19, id.); otherwise the petition should be denied. Based on the above-cited procedure, after the hearing, the court's order granting or refusing bail must contain a summary of the evidence of the prosecution and based thereon, the judge should formulate his own conclusion as to whether the evidence so presented is strong enough to indicate the guilt of the accused. Respondent judge did not follow the above Rules and procedure enumerated in Cortes. He did not conduct a hearing before he granted bail to the accused, thus depriving the prosecution of an opportunity to interpose objections to the grant of bail. Irrespective of his opinion on the strength or weakness of evidence to prove the guilt of the accused, he should have conducted a hearing and thereafter made a summary of the evidence of the prosecution. The importance of a bail hearing and a summary of evidence cannot be downplayed, these are considered aspects of procedural due process for both the prosecution and the defense; its absence will invalidate the grant or denial of bail.
LEVISTE VS. CA G.R. NO. 189122 MARCH 17, 2010

FACTS: Charged with the murder of Rafael de las Alas, petitioner Jose Antonio Leviste was convicted by the Regional Trial Court of Makati City for the lesser crime of homicide and sentenced to suffer an indeterminate penalty of six years and one day of prision mayor as minimum to 12 years and one day of reclusion temporal as maximum. He appealed his conviction to the Court of Appeals. Pending appeal, he filed an urgent application for admission to bail pending appeal, citing his advanced age and health condition, and claiming the absence of any risk or possibility of flight on his part. The Court of Appeals denied petitioners application for bail. Petitioner now questions as grave abuse of discretion the denial of his application for bail, considering that none of the conditions justifying denial of bail under the third paragraph of Section 5, Rule 114 of the Rules of Court was present. Petitioners theory is that, where the penalty imposed by the trial court is more than six years but not more than 20 years and the circumstances mentioned in the third paragraph of Section 5 are absent, bail must be granted to an appellant pending appeal. ISSUE: In an application for bail pending appeal by an appellant sentenced by the trial court to a penalty of imprisonment for more than six years, does the discretionary nature of the grant of bail pending appeal mean that bail should automatically be granted absent any of the circumstances mentioned in the third paragraph of Section 5, Rule 114 of the Rules of Court? 40

HELD: No. the appellate courts stringent discretion requires that the exercise ther eof be primarily focused on the determination of the proof of the presence of any of the circumstances that are prejudicial to the allowance of bail. This is so because the existence of any of those circumstances is by itself sufficient to deny or revoke bail. Nonetheless, a finding that none of the said circumstances is present will not automatically result in the grant of bail. Such finding will simply authorize the court to use the less stringent sound discretion approach. Our rules authorize the proper courts to exercise discretion in the grant of bail pending appeal to those convicted by the Regional Trial Court of an offense not punishable by death, reclusion perpetua or life imprisonment. In the exercise of that discretion, the proper courts are to be guided by the fundamental principle that the allowance of bail pending appeal should be exercised not with laxity but with grave caution and only for strong reasons, considering that the accused has been in fact convicted by the trial court.
IVLER VS. MODESTO-SAN PEDRO G.R. NO. 172716 NOVEMBER 17, 2010

FACTS: Following a vehicular collision in August 2004, petitioner Jason Ivler (petitioner) was charged before the Metropolitan Trial Court of Pasig City, Branch 71 (MeTC), with two separate offenses: (1) Reckless Imprudence Resulting in Slight Physical Injuries (Criminal Case No. 82367) for injuries sustained by respondent Evangeline L. Ponce (respondent Ponce); and (2) Reckless Imprudence Resulting in Homicide and Damage to Property (Criminal Case No. 82366) for the death of respondent Ponces husband Nestor C. Ponce and damage to the spouses Ponces vehicle. Petitioner posted bail for his temporary release in both cases. Ivler did not attend the arraignment. ISSUE: Whether petitioner forfeited his standing to seek relief in S.C.A. 2803 when the MeTC ordered his arrest following his non-appearance at the arraignment in Criminal Case No. 82366 HELD: Under Section 21, Rule 114 of the Revised Rules of Criminal Procedure, the defendants absence merely renders his bondsman potentially liable on its bond (subject to cancellation should the bondsman fail to produce the accused within 30 days); the defendant retains his standing and, should he fail to surrender, will be tried in absentia and could be convicted or acquitted. Indeed, the 30-day period granted to the bondsman to produce the accused underscores the fact that mere non-appearance does not ipso facto convert the accuseds status to that of a fugitive without standing.

GACAL VS. INFANTE [FORMERLY A.M. NO. IPI NO. 03-1831-RTJ] A.M. NO. RTJ- 04-1845 OCTOBER 5, 2011

FACTS: Atty. Franklin Gacal, the private prosecutor in Criminal Case No. 1136-03 of the Regional Trial Court (RTC) in Alabel, Sarangani entitled People v. Faustino Ancheta, a prosecution for murder arising from the killing of Felomino O. Occasion, charges Judge Jaime I. Infante, Presiding Judge of Branch 38 of the RTC to whose Branch Criminal Case No. 1136-03 was raffled for arraignment and trial, with gross ignorance 41

of the law, gross incompetence, and evident partiality, for the latters failure to set a hearing before granting bail to the accused and for releasing him immediately after allowing bail. Judge Infante to rely on the mere representation of the public prosecutor that his grant of bail upon the public prosecutors recommendation had been proper, and that his (public prosecutor) recommendation of bail had in effect waived the need for a bail hearing ISSUE: Whether the judge erred in not conducting a hearing before allowing bail. HELD: Yes. Bail hearing is mandatory. The willingness of Judge Infante to rely on the mere representation of the public prosecutor that his grant of bail upon the public prosecutors recommendation had been proper, and that his (public prosecutor) recommendation of bail had in effect waived the need for a bail hearing perplexes the Court. He thereby betrayed an uncommon readiness to trust more in the public prosecutors judgment than in his own judicious discretion as a trial judge. He should not do so. It is axiomatic that bail cannot be allowed to a person charged with a capital offense, or an offense punishable with reclusion perpetua or life imprisonment, without a hearing upon notice to the Prosecution. Any judge who so allows bail is guilty of gross ignorance of the law and the rules, and is subject to appropriate administrative sanctions.
OKABE VS. GUTIERREZ G.R. NO. 150185 MAY 27, 2004

FACTS: Cecilia Maruyama executed a fifteen-page affidavit-complaint2 and filed the same with the Office of the City Prosecutor of Pasay City, on December 29, 1999, charging Lorna Tanghal and petitioner Teresita Tanghal Okabe, a.k.a. Shiela Okabe, with estafa. In her affidavit, Maruyama alleged, inter alia, that on December 11, 1998, she entrusted Y11,410,000 with the peso equivalent of P3,993,500 to the petitioner, who was engaged in the business of "door-to-door delivery" from Japan to the Philippines. It was alleged that the petitioner failed to deliver the money as agreed upon, and, at first, denied receiving the said amount but later returned only US$1,000 through Lorna Tanghal. On June 15, 2000, the petitioner posted a personal bail bond in the said amount, duly approved by Judge Demetrio B. Macapagal, the Presiding Judge of Branch 79 of the RTC of Quezon City, who forthwith recalled the said warrant. The approved personal bail bond of the petitioner was transmitted to the RTC of Pasig City on June 21, 2000. Upon her request, the petitioner was furnished with a certified copy of the Information, the resolution and the criminal complaint which formed part of the records of the said case. The petitioner left the Philippines for Japan on June 17, 2000 witho ut the trial courts permission, and returned to the Philippines on June 28, 2000. She left the Philippines anew on July 1, 2000, and returned on July 12, 2000. On July 14, 2000, the trial court issued an Order setting the petitioners arraignment and pre -trial at 2:00 p.m. of July 16, 2000. On the same day, the private prosecutor filed an urgent ex parte motion for the issuance of the hold departure order. ISSUE: 42

Whether the posting of a personal bail bond for her provisional liability resulted to voluntary submission to the jurisdiction of the trial court and waiver of right to assail the infirmities that infected the trial courts issuance of the warrant for her arrest HELD: NO. It bears stressing that Section 26, Rule 114 of the Revised Rules on Criminal Procedure is a new one, intended to modify previous rulings of this Court that an application for bail or the admission to bail by the accused shall be considered as a waiver of his right to assail the warrant issued for his arrest on the legalities or irregularities thereon.
RE: ANONYMOUS LETTER-COMPLAINT AGAINST HON. MARILOU RUNES-TAMANG, PRESIDING JUDGE A.M. MTJ-04-1558 APRIL 7, 2010(FORMERLY OCA IPI NO. 04-1594-MTJ)

FACTS: An anonymous "Concerned Filipino Citizen" sent to then Chief Justice Hilario G. Davide, Jr. a letter dated October 22, 2003 requesting the investigation of Judge Marilou D. Runes-Tamang, Presiding Judge of the Metropolitan Trial Court (MeTC) in Pateros and Acting Presiding Judge of the MeTC in San Juan, Metro Manila. The letter-sender complained that Judge Tamang, through the connivance of the arresting officer and court employees of MeTC at San Juan, had been indiscriminately approving fake bonds for a fee of P1,000.00 "per count ng kaso. The office of DCA Lock conducted a discreet investigation on the reported impropriety. The investigation revealed that Judge Tamang had approved bail bonds issued by Covenant Assurance Company, Inc (Covenant), despite Covenant having been blacklisted since December 20, 2002 in the RTC in Pasig City. Maintaining her innocence of the charges, Judge Tamang submitted her answer/comment dated September 30, 2003,13 in which she related the circumstances surrounding the approval of the bail bonds. Insisting that the court personnel had taken advantage of her leniency and kindness, Judge Tamang declared that she "has never transgressed the Code of Judicial Conduct with malicious intention and orchestrated plans of compromising the integrity of the judiciary." ISSUE: Whether Judge Tamang is guilty of neglect of duty. HELD: YES. Judge Tamang approved bail bonds issued by Covenant although they manifestly lacked the required clearance from the Supreme Court indicating that Covenant was qualified to transact business with the courts. As earlier stated, Covenant was a blacklisted company at the time of issuance of the bail bonds. She was thereby guilty of a neglect of duty, for, according to Judicial Audit and Physical Inventory of Confiscated Cash, Surety and Property Bonds at RTC, Tarlac City, Brs. 63, 64 & 65, the judge is still bound to review the supporting documents before approving the bail bonds, even if it is the Clerk of Court who has the duty to ascertain that the bail bonds are in order, and that all requisites for approval have been complied with. The Court concurred with the OCAs following observation submitted in said case, to wit:

43

Although the duty to ensure compliance with the requisites of the bail bond application rests mainly with the Clerk of Court or his duly authorized personnel and the task of the Judge is only to approve the same, said task has an accompanying responsibility on the part of the approving Judge to review or determine its validity. Understandably, he should be employing the minimum standard the rules require the clerks of court to observe. Considering the seriousness of the purpose in the posting of bail bond, approval thereof should pass through strict scrutiny and with utmost caution on the part of both the Clerk of Court (or his duly authorized personnel) and the approving Judge
PEOPLE vs. HON. JUDGE AYSON G.R. NO. 85215

FACTS: Felipe Ramos was a ticket freight clerk of PAL assigned in Baguio City Station. He was alleged to b e involved in some irregularities in the sales of plane tickets. The PAL mgt. notified and invited him for an administrative investigation in accordance with PALs Code of Conduct and Discipline and the Collective Bargaining signed by PALEA (PAL Employees Association). During the investigation, Ramos gave his superiors a handwritten note stating that he is willing to settle the irregularities allegedly charges against him. On arraignment, Ramos pleaded not guilty. On trial, the prosecutors presented a written offer of evidence which included the statement of Felipe together with his handwritten admission. The defense questioned the evidence presented on the ground that such statement was taken without the accused being represented by a lawyer. ISSUE: Whether the right against self- incrimination and those during custodial investigation apply to persons under preliminary investigation or already charged in court for a crime. HELD: Yes and No. Self-incrimination applies to him while those rights during custodial investigation do not apply in this case. Included in the conduct of custodial investigation is the right to have an attorney present for the evidence adduced from it be admissible. In this case, Felipe Ramos was not under custodial investigation as custodial investigation is defined by the court as questioning initiated by law enforcement officers after a person has been taken into custody or otherwise deprived of his freedom of act on in any significant way.
PEOPLE VS. MORIAL, ET. AL. G.R. NO. 129295

FACTS: The appellants were sentenced for Robbery with Homicide. Upon arraignment, the 3 accused pleaded not guilty. During trial, they interposed denial and alibi as their defense denying that they were together at the time of the incident. After trial, the RTC rendered a decision convicting the 3 accused. The appellants conviction rested on 2 vital pieces of evidence: the extra-judicial confession of one of the appellant Leonardo Morial and the eyewitness account of Gabriel Guilao. Ca found the extra-judicial confession as invalid since he was effectively deprived of his right to counsel during custodial investigation on the ground that his counsel, Atty. Aguilar was not present throughout the investigation, instead he left after the material points were asked as stated by the said counsel. 44

ISSUE: Whether the extra-judicial confession was admissible as evidence HELD: No. In a custodial investigation, the counsel afforded to the accused should be present the en t ire investigation. If for instance, the said counsel should go someplace else, he could either terminate the investigation until such time that he is again available or he could designate a new counsel so as not to delay the process.
PEOPLE VS. BASADA, ET. AL. G.R. NO. 185840

FACTS: The Assistant Provincial Prosecutor of Marikina charged the accused Pedro, Ricardo alias Carding, Reynaldo alias Rene, Crisanto alias Totoy, and Buyo, all surnamed Basada, and Elmer Apelado before the Regional Trial Court (RTC) of San Mateo, Rizal, in Criminal Case 2929 with the crime of murder. Upon arraignment, all of the accused pleaded not guilty to the charge, except Reynaldo and Buyo, who were then both at large. The CA decided that the offense proved was homicide instead of murder because of the absence of the circumstance of treachery. ISSUE: Whether the CA is correct in its ruling. HELD: YES. As a general rule, a trial courts assessment of the credibility of a witness is entitled to great weight. But this is true only if the trial court had not overlooked some fact or circumstance of great weight and persuasiveness, which if taken into account, could affect the outcome of the case such as when the witnesses are not telling the truth as in this case. The prosecution has the burden of proving the guilt of the accused beyond reasonable doubt.[5] The overriding consideration is not whether the court doubts the innocence of the accused but whether it entertains a reasonable doubt as to his guilt.[6] Here, the prosecution amply proved that Reynaldo stabbed Jill but utterly failed to show the involvement of the others in the offense.
PEOPLE V. SIONGCO G.R. NO. 186472 JULY 5, 2010

FACTS: Appellants Siongco, Boton and Enriquez, induced 11-year old Nikko Satimbre, a resident of Balanga, Bataan, to board a bus bound for Pilar, Bataan and promised the latter a Gameboy. He was then bought to Dinalupihan, Bataan where he was kept for the night. Two days after, Siongco called Elvira Satimbre, Nikkos mother, and demanded P400,000.00, in exchange for the release of her son. Siongco further threatened that Nikko would be killed if Elvira failed to give the ransom money. Nikko was moved to Taguig City and was cautioned not to tell anybody that he was kidnapped. Appellants were finally arrested in an entrapment operation conducted by the PAOCTF four days after Nikko was kidnapped. The RTC convicted appellants of kidnapping with serious illegal detention, then punishable 45

by death, with the exception of Boton, on the ground of reasonable doubt. The CA affirmed the conviction byt modified the penalty to reclusion perpetua. On review, the appellants claimed that they were deprived of their right to an independent and competent counsel when the RTC appointed Atty. Michael Moralde (Atty. Moralde) as their counsel de oficio during the pre-trial conference, direct examination and cross-examination of the prosecutions principal witness, Nikko. This was so, despite Atty. Moraldes manifestation during Nikkos cross examination that the defense of his actual client, accused Boton, conflicts with that of the other accused. ISSUE: Whether Appellants were deprived of their right to an independent and competent counsel by the appointment of Atty.Moralde. HELD: NO, A scrutiny of the records shows that Atty. Moralde was appointed as appellants counsel de oficio in six (6) hearings, because their regular counsel de oficio, Atty. Antoniano from the Public Attorneys Office (PAO), was inexplicably absent. There is no denial of the right to counsel where a counsel de oficio is appointed during the absence of the accused's counsel de parte, or in this case the regular counsel de oficio, pursuant to the court's desire to finish the case as early as practicable under the continuous trial system. The choice of counsel by the accused in a criminal prosecution is not a plenary one. If the chosen counsel deliberately makes himself scarce, the court is not precluded from appointing a de oficio counsel, which it considers competent and independent, to enable the trial to proceed until the counsel of choice enters his appearance. Otherwise, the pace of a criminal prosecution will be entirely dictated by the accused, to the detriment of the eventual resolution of the case.
PEOPLE VS. FRANCISCO G.R. NO. 192818 NOVEMBER 17, 2010

FACTS: Appellant was indicted for murder under Article 248 of the Revised Penal Code (RPC). The accused allegedly attacked, assaulted and stabbed the Ramil Tablate, with the use of a bladed instrument (kitchen knife) wounding mortally his chest, abdomen and different parts of his body which wounds were necessarily mortal causing the direct and immediate death. During arraignment, appellant pleaded not guilty to the crime charged. However, during the pre-trial on March 4, 2003, he withdrew his former plea. Consequently, on the same hearing, he was re-arraigned and he pleaded guilty to the crime charged. Through the March 4, 2003 Order from the pre-trial proceeding, it was shown that the RTC conducted searching questions to determine that appellant voluntarily entered his guilty plea and that he understood its consequences.The prosecution rested its case and made its formal offer of exhibits without any objection from the defense.After admitting the death of Ramil resulting from appellant s assault, the defense, however, did not present any witnesses, but simply argued that the offense of appellant is only homicide and not murder. Contending that no treachery attended the assault, the defense asserted that appellant did not attack Ramil from behind. RTC convicted appellant which the on appeal the CA affirmed. Appellant maintains that he was not given opportunity to present evidence and that the case was submitted for decision immediately after the prosecution filed its offer of evidence. 46

ISSUE: Whether appellant was deprived of his right to present evience? HELD: NO. The defense chose not to present any witnesses which amounts to a waiver to present evidence. This was not objected to by appellant. Thus, there was an implied acquiescence on the part of appellant not to present himself or other witnesses even though he was entitled to present evidence to prove, inter alia, mitigating circumstances under Sec. 3 of Rule 116. Appellant is, consequently, estopped from questioning the rendition of the trial courts disposition of the case without the presentation of any evidence by the defense, unless there are exceptional reasons justifying the additional reception of evidence for the defense. Appellant has not shown any cogent justification to set aside the defenses waiver of right to present evidence. Moreover, the records show that appellant filed neither comment nor objection to the prosecutions Formal Offer of Exhibits. We also take note that under Sec. 3, Rule 116, the accused may present evidence in his behalfit is, therefore, not mandatory for the defense to present evidence but is only accorded an opportunity to do so, which, in the instant case, was waived by the defense.
IMPERIAL VS JOSON

Facts: At or about 2:00 oclock in the morning of 11 May 2001, along the portion of the National Highway in Barangay Concepcion, Sariaya, Quezon, an Isuzu ten-wheeler truck collided with a Fuso six-wheeler truck. Owned by petitioner Nelson Imperial, the Isuzu ten-wheeler truck was then being driven by petitioner Santos Francisco, while the Fuso six-wheeler truck was driven by respondent Santiago Giganto, Jr. who was, at the time, accompanied by a helper or pahinante, respondent Samuel Cubeta. After colliding with the Fuso six-wheeler truck, the Isuzu ten-wheeler truck further rammed into a Kia Besta Van which was, in turn, being driven by respondent Arnel Lazo. The KIA Besta Van was owned by Noel Tagle who was then on board said vehicle, together with 8 passengers. As a consequence of the collisions which resulted to the death of the driver of a Kia Besta van and and its 8 passengers, a criminal complaint for Reckless Imprudence Resulting to Multiple Homicide, Multiple Serious Physical Injuries and Damage to Property was filed against petitioners Santos Francisco and Noel Imperial. The case was docketed as Criminal Case No. 01-99 before the Municipal Trial Court (MTC) of Sariaya, Quezon. (Many action have been brought to different courts by one party against the other. Nobela ito! But for purposes of the issue on rights of the accused, we'll take the one filed in the MTC Sariaya). Issue: Whether or not the fact that the nine postponements of the pre-trial conference of the case amounted to a violation of Francisco's constitutional right to a speedy trial. Held: In determining whether the accused has been deprived of his right to a speedy disposition of the case and to a speedy trial, four factors must be considered: (a) length of delay; (b) the reason for the delay; (c) the defendants assertion of his right; and (d) prejudice to the defendant. xxxx

47

Petitioner Francisco claims that his right to a speedy trial was violated when the Public Prosecutors assigned to the case failed to attend the nine hearings scheduled by the Sariaya MTC. Far from being vexatious, capricious and oppressive, however, the delays entailed by the postponements of the aforesaid hearings were, to a great extent, attributable to petitioner Franciscos own pursuit of extraordinary remedies against the interlocutory orders issued by the Sariaya MTC and the assignment of at least three public prosecutors to the case. Although the Revised Rules of Criminal Procedure concededly mandates commencement of the trial within 30 days from receipt of the pre-trial order and the continuous conduct thereof for a period not exceeding 180 days, Section 3 a (1), Rule 119 provides that delays resulting from extraordinary remedies against interlocutory orders shall be excluded in computing the time within which trial must commence. In determining the right of an accused to speedy trial, moreover, courts are "required to do more than a mathematical computation of the number of postponements of the scheduled hearings of the case" and to give particular regard to the facts and circumstances peculiar to each case.

PEOPLE VS ESTOMACA

Facts: On May 24, 1994, consequent to five separate complaints, Criminal Cases Nos. 43567,43568,43569,43570 and 43571 were filed in the Regional Trial Court, Branch 38, Iloilo City charging herein appellant, an illiterate laborer, with rape committed on five separate occasions against his own daughter, complainant Estelita Estomaca. The arraignment appears to have consisted merely of the bare reading of the five complaints, synthetically and cryptically reported in the transcript, thus: (Reading the information/complaint to the accused in Ilonggo/local dialect). Since what was supposed to have been read was stated in the singular, but there were five criminal complaints against appellant, this Court is then left to speculate on whether all five criminal complaints were actually read, translated or explained to appellant on a level within his comprehension, considering his limited education. There is some inconsistency in the statements on record as to what actually took place on June 14, 1994 during the arraignment of appellant, assisted by his government counsel de oficio, Atty. Rogelio Antiquiera. The decision of the court below, dated July 15, 1994, declares that he entered a plea of guilty to Criminal Cases Nos. 43568 and 43571, and a plea of not guilty to Criminal Cases Nos. 43567, 43569 and 43570. Issue: Whether or not the arraignment was in accordance with the prescribed rules. Held: It will be readily observed, if one would analyze appellants responses during his irregular arraignment, that his low intelligence quotient and lack of education combined to deprive him of fully understanding what obviously appeared to him as mysterious rituals and unfamiliar jargons. The significance of this distinction is found right in the provisions of Section 1(a) of Rule 116 which, cognizant of the aforestated linguistic variations, deliberately required that the complaint or information be read to the accused in the language or the dialect known to him, to ensure his comprehension of the 48

charges. The Court takes judicial notice, because it is either of public knowledge or readily capable of unquestionable demonstration, that in the central and northwestern part of Iloilo province and all the way up to and throughout Antique, including necessarily San Joaquin where the offenses were committed and of which appellant and his family are natives, the local dialect is known as kinaray -a. Section 3 of Rule 116 which the trial court violated is not a new rule for it merely incorporated the decision of this Court in People vs. Apduhan Jr. and reiterated in an unbroken line of cases. The bottom line of the rule is that a plea of guilt must be based on a free and informed judgment. Thus, the searching inquiry of the trial court must be focused on: (1) the voluntariness of the plea; and (2) the full comprehension of the consequences of the plea. The questions of the trial court failed to show the voluntariness of the plea of guilt of the appellant nor did the questions demonstrate appellants full comprehension of the consequences of the plea. The records do not reveal any information about the personality profile of the appellant which can serve as a trustworthy index of his capacity to give a free and informed plea of guilt. The age, socio-economic status, and educational background of the appellant were not plumbed by the trial court

SAMSON VS DAWAY

Facts: Two informations for unfair competition under Section 168.3 (a), in relation to Section 170, of the Intellectual Property Code (Republic Act No. 8293), similarly worded save for the dates and places of commission, were filed against petitioner Manolo P. Samson, the registered owner of ITTI Shoes distribute, sell and/or offer for sale CATERPILLAR products such as footwear, garments, clothing, bags, accessories and paraphernalia which are closely identical to and/or colorable imitations of the authentic Caterpillar products and likewise using trademarks, symbols and/or designs as would cause confusion, mistake or deception on the part of the buying public. On April 19, 2002, petitioner filed a motion to suspend arraignment and other proceedings in view of the existence of an alleged prejudicial question involved in Civil Case No. Q-00-41446 for unfair competition pending with the same branch; and also in view of the pendency of a petition for review filed with the Secretary of Justice assailing the Chief State Prosecutors resolution finding probable cause to charge petitioner with unfair competition. In an Order dated August 9, 2002, the trial court denied the motion to suspend arraignment and other proceedings. Issue: Did the respondent Judge gravely abuse his discretion in refusing to suspend the arraignment and other proceedings in Criminal Case Nos. Q-02-108043-44 on the ground of (a) the existence of a prejudicial question; and (b) the pendency of a petition for review with the Secretary of Justice on the finding of probable cause for unfair competition?

Held: Section 11 (c), Rule 116 of the Revised Rules on Criminal Procedure provides SEC. 11. Suspension of arraignment. Upon motion by the proper party, the arraignment shall be suspended in the following cases xxx xxx xxx (c) A petition for review of the resolution of the prosecutor is pending at either the Department of Justice, or the Office of the President; Provided, that the period of suspension shall not exceed sixty (60) days counted from the filing of the petition with the reviewing office. 49

While the pendency of a petition for review is a ground for suspension of the arraignment, the aforecited provision limits the deferment of the arraignment to a period of 60 days reckoned from the filing of the petition with the reviewing office. It follows, therefore, that after the expiration of said period, the trial court is bound to arraign the accused or to deny the motion to defer arraignment. In the instant case, petitioner failed to establish that respondent Judge abused his discretion in denying his motion to suspend. His pleadings and annexes submitted before the Court do not show the date of filing of the petition for review with the Secretary of Justice.15 Moreover, the Order dated August 9, 2002 denying his motion to suspend was not appended to the petition. He thus failed to discharge the burden of proving that he was entitled to a suspension of his arraignment and that the questioned orders are contrary to Section 11 (c), Rule 116 of the Revised Rules on Criminal Procedure. Indeed, the age-old but familiar rule is that he who alleges must prove his allegations.
PEOPLE VS PANGILINAN

Facts: Accused-appellant Nomer Velasco y Pangilinan together with Reynaldo Endrina y Roa and Ernesto Figueroa y Santos were charged with the crime of Murder in an Information filed on March 2, 1994. Upon arraignment on March 16, 1994, all three accused duly assisted by their counsel de parte pleaded not guilty to the offense charged in the Information. During the trial, he was positively identified by an eyewitness, Leonardo Lucaban. After the presentation of evidence, the trial court in its decision promulgated on February 19, 1996 found the accused-appellant to be guilty beyond reasonable doubt and ordered the acquittal of Reynaldo Endrina and Ernesto Figueroa. Issue: Whether or not the court a quo should favorably consider the defense of alibi. Held: It is well-settled rule that the defense of alibi, admittedly, the weakest defense, cannot prevail over the positive identification of the accused by prosecution witnesses. Accused-appellant contends that he was sleeping at his house at the time of the incident. His wife corroborates this. However, in this instance, we have to take the word of his wife with a grain of salt for witnesses who are either wives or mothers of the accused, in almost all instances, would freely perjure themselves for the sake of their loved ones. In the light of the positive identification made by an eyewitness who admittedly has no grudge against the accused-appellant the defense of alibi put up by the latter does not hold water.
PEOPLE VS AGUILAR

Facts: Accused-appellant Noel Aguilar y Amistuso was charged with murder in two separate Information for the killing of Helen Revilla and Angelaida Pascua. Upon arraignment, accused-appellant pleaded not guilty. The trial court was not persuaded by accused-appellants version that he acted in self-defense and instead found him guilty as charged. Accused-appellant now assails the trial courts Decision. Issue: Whether or not Aguilars plea of self-defense be given credence.

50

Held: Where an accused charged with the killing of a person admits having caused that death but invokes selfdefense to escape criminal liability, it becomes incumbent upon him to prove by clear and convincing evidence the positiveness of that justifying circumstance; otherwise, having admitted the killing, conviction is inescapable. Accused-appellant failed in his attempt to show the element of unlawful aggression. That one of the victims supposedly went on top of him and poked a sharp pointed instrument near his armpit while another tried to get his wallet is nothing but a self-serving statement which did not, in any way, meet the required quantum of proof for unlawful aggression. Neither did accused-appellant establish the reasonableness of the means employed to prevent or repel the so-called attack. Helen and Angelaida suffered multiple stab wounds while accused -appellant did not even have a single wound to present before the lower court. That he had a wound on his forefinger can hardly be compared to the extent of wounds inflicted upon his victims. Besides, his allegation that he had a wound on one of his fingers is again self-serving as he did not present a medical certificate to corroborate his testimony. For failure to prove unlawful aggression and the reasonableness and necessity of the means employed to prevent or repel the attack, accused-appellants plea of self-defense must fail.
DAAN VS. SANDIGANBAYAN G.R. NOS. 163972-77

Facts: Joselito Daan together with co-accused Benedicto Kuizon were charged for three counts of malversation of public funds which they purportedly tried to conceal by falsifying the time book and payrolls for given period making it appear that some laborers worked on the construction of the new municipal hall building of Bato, Leyte and collected their respective salaries thereon when, in truth and in fact, they did not. Thus, in addition to the charge for malversation, the accused were also indicted for three counts of falsification of public document by a public officer or employee. The accused offered withdraw their plea of "not guilty" and substitute the same with a plea of "guilty", provided, the mitigating circumstances of confession or plea of guilt and voluntary surrender will be appreciated in their favor. In the alternative, if such proposal is not acceptable, said accused proposed instead to substitute their plea of "not guilty" to the crime of falsification of public document by a public officer or employee with a plea of "guilty", but to the lesser crime of falsification of a public document by a private individual. On the other hand, in the malversation cases, the accused offered to substitute their plea of "not guilty" thereto with a plea of "guilty", but to the lesser crime of failure of an accountable officer to render accounts. The Sandiganbayan denied petitioners Motion to Plea Bargain, despite favorable recommendation by the prosecution, on the main ground that no cogent reason was presented to justify its approval. Hence, this appeal. Issue: Whether Sandiganbayan committed grave abuse of discretion in denying petitioners plea bargaining offer. Held:

51

Plea bargaining in criminal cases is a process whereby the accused and the prosecution work out a mutually satisfactory disposition of the case subject to court approval. It usually involves the defendant's pleading guilty to a lesser offense or to only one or some of the counts of a multi-count indictment in return for a lighter sentence than that for the graver charge. Records show that there was a favorable recommendation by the Office of the Special Prosecutor to approve petitioner's motion to plea bargain. With respect to the falsification cases earlier mentioned, it appears that the act of the accused in pleading guilty for a lesser offense of falsification by private individual defined and penalized under Article 172 of the Revised Penal Code will strengthen the cases against the principal accused, the Municipal Mayor Benedicto Kuizon, who appears to be the master mind of these criminal acts. After all, the movants herein JOSELITO RANIERO J. DAAN was merely designated as draftsman detailed as foreman/timekeeper of the Municipality of Bato, Leyte. In the cases at bar, there is no dispute that JOSELITO RANIERO J. DAAN has already restituted the total amount of P18,860.00 as per official receipt issued by the provincial government of Leyte dated February 26, 2002. In short, the damage caused to the government has already been restituted by the accused. There is also no dispute that accused DAAN voluntarily surrendered in the instant cases. Moreover, the accused is also willing to plead guilty to a lesser offense which to our mind, merits consideration. Petition granted.
BIENVENIDO DIO VS. PABLO OLIVAREZ G.R. NO. 170447

Facts: Petitioners Dio & Comparativo filed two (2) information against respondent Olivarez for allegedly buying votes to the people of Paranaque City, as herein respondent Olivarez was running for mayoralty position in the said city. Olivarez filed before the Law department of the Comelec an appeal of the Joint Resolution of the City Prosecutor of Paraaque City with Motion to Revoke Continuing Authority, arguing that the pendency of the appeal of the Joint Resolution before the COMELEC should prevent the filing of the Informations before the RTC as there could be no final finding of probable cause until the COMELEC had resolved the appeal. Moreover, he argued that the charges made against him were groundless. Consequently, respondent filed a Motion to Quash the 2 Information on the ground that more than one offense was charged therein, in violation of Section 3(f), Rule 117 of the Rules of Court, in relation to Section 13, Rule 110 of the Rules of Court. Judge Madrona denied the said Motion. Reconsideration of the denied motion was also rejected by Judge Madrona. The said judge reset the arraignment to 9 March 2005, with a warning that the arraignment would proceed without any more delay. On the arraignment day, respondent failed to appear in court, thus prompting Judge Madrona to order the arrest of respondent and the confiscation of the cash bond.

52

Issue: WON Judge Madrona erred in issuing orders for the arrest of respondent due to his failure to attend the arraignment. Held: No. The orders issued by the said judge are consistent with the Rules of Criminal Procedure. The filing of an information in the trial court initiates a criminal action. The trial court thereby acquires jurisdiction over the case. After the filing of the complaint or the information, a warrant for the arrest of the accused is issued by the trial court. When the accused voluntarily submits himself to the court or is duly arrested, the court then acquires jurisdiction over the person of the accused. In this case, the trial court acquired jurisdiction over the persons of the accused Carmelo Jaro, Remedios Malibaran, and the respondent, who posted bail bonds after the trial court issued a Warrant of Arrest on 4 October 2004. While it is true that the fiscal has the quasi-judicial discretion to determine whether or not a criminal case should be filed in court, once the case has been brought to court, whatever disposition the fiscal may feel is proper in the case should be addressed to the consideration of the trial court. Thereafter, arraignment shall follow as a matter of course. Section 11, Rule 116 of the Rules of Criminal Procedure.

PEOPLE VS. JANJALANI G.R. NO. 188314

JANUARY 10, 2011

FACTS: The Valentines day bombing was allegedly committed by herein accused. As stipulated during pretrial, accused Trinidad gave ABS-CBN News Network an exclusive interview some time after the incident, confessing his participation in the Valentines Day bombing incident. In another exclusive interview on the network, accused Baharan likewise admitted his role in the bombing incident. Finally, accused Asali gave a television interview, confessing that he had supplied the explosive devices for the 14 February 2005 bombing. The bus conductor identified the accused Baharan and Trinidad, and confirmed that they were the two men who had entered the RRCG bus on the evening of 14 February. Members of the Abu Sayyaf Group namely Khaddafy Janjalani, Gamal B. Baharan, Angelo Trinidad, Gappal Bannah Asali, Jainal Asali, Rohmat Abdurrohim a.k.a. Abu Jackie or Zaky, and other "John" and "Jane Does" were then charged with multiple murder and multiple frustrated murder. Only Baharan, Trinidad, Asali, and Rohmat were arrested, while the other accused remain at-large. On their arraignment for the multiple murder charge, Baharan, Trinidad, and Asali all entered a plea of guilty. On the other hand, upon arraignment for the multiple frustrated murder charge, accused Asali pled guilty. Accused Trinidad and Baharan pled not guilty. Rohmat pled not guilty to both charges. In the light of the pretrial stipulations, the trial court asked whether accused Baharan and Trinidad were amenable to changing their "not guilty" pleas to the charge of multiple frustrated murder, considering that they pled "guilty" to the heavier charge of multiple murder, creating an apparent inconsistency in their pleas. Defense counsel conferred with accused Baharan and Trinidad and explained to them the consequences of the pleas. The two accused acknowledged the inconsistencies and manifested their readiness for re-arraignment. After the Information was read to them, Baharan and Trinidad pled guilty to the charge of multiple frustrated murder. HOWEVER, Accused-appellants Baharan and Trinidad argue that the trial court did not conduct a searching inquiry after they had changed their plea from "not guilty" to "guilty." 53

ISSUES: Whether or not the trial court gravely erred in accepting accused-appellants plea of guilt despite insufficiency of searching inquiry into the voluntariness and full comprehension of the consequences of the said plea. HELD: NO. As early as in People v. Apduhan, the Supreme Court has ruled that "all trial judges must refrain from accepting with alacrity an accused's plea of guilty, for while justice demands a speedy administration, judges are duty bound to be extra solicitous in seeing to it that when an accused pleads guilty, he understands fully the meaning of his plea and the import of an inevitable conviction." We have reiterated in a long line of cases that the conduct of a searching inquiry remains the duty of judges, as they are mandated by the rules to satisfy themselves that the accused had not been under coercion or duress; mistaken impressions; or a misunderstanding of the significance, effects, and consequences of their guilty plea.This requirement is stringent and mandatory Nevertheless, we are not unmindful of the context under which the re-arraignment was conducted or of the factual milieu surrounding the finding of guilt against the accused. The Court observes that accused Baharan and Trinidad previously pled guilty to another charge multiple murder based on the same act relied upon in the multiple frustrated murder charge. The Court further notes that prior to the change of plea to one of guilt, accused Baharan and Trinidad made two other confessions of guilt one through an extrajudicial confession (exclusive television interviews, as stipulated by both accused during pretrial), and the other via judicial admission (pretrial stipulation). Considering the foregoing circumstances, we deem it unnecessary to rule on the sufficiency of the "searching inquiry" in this instance. Remanding the case for re-arraignment is not warranted, as the accuseds plea of guilt was not the sole basis of the condemnatory judgment under consideration.
PEOPLE VS. ABAD G.R. NO. L-55132 AUGUST 30, 1988

FACTS: On complaint by Felix de Castro (permittee to quarry), an Information was filed in the Court of First Instance of Ifugao, presided over by respondent Judge, charging private respondents with the crime of "Theft of Minerals" defined and penalized under Section 78 of Presidential Decree No. 483, as amended by Presidential Decree No. 1385. Respondents-accused filed a Motion to Quash on the ground that the facts charged do not constitute an offense inasmuch as they had paid "sand and gravel tax," as shown by three official receipts, to the Municipal Treasurer of Banawe, Ifugao, for the quarrying of sand and gravel. The taking, therefore, according to private respondents, was with the consent of the government. They also invoked LOI No. 243, which allows persons to extract sand and gravel even within the leased area for use in government infrastructures. On 28 January 1980, respondent Judge issued the assailed Order quashing the Information on the ground that violation of P.D. No. 463 is limited to an administrative violation and that the crime of Theft under the Revised Penal Code (Article 308) has not been committed since malice, which is an essential element in the commission of a crime, is lacking. The reconsideration prayed for by petitioner was denied by respondent on 18 July 1980. Hence, this certiorari Petition alleging grave abuse of discretion on the part of respondent Judge

54

ISSUE: The crucial issue for resolution is whether or not the facts charged in the Information constitute an offense, for if not is a ground to quash the information. HELD: It is basic that since respondents-accused invoked the ground "that the facts charged do not constitute an offense" (Rule 1 17, Sec. 2[a] Rules of Court), the sufficiency of the Information hinges on the question of whether the facts alleged, if hypothetically admitted, meet the essential elements of the offense as defined in the law (People vs. Segovia 103 Phil. 1162 [1958]). Based upon the facts alleged in the Information, the essential requisites of the Offense of "Theft of Minerals," as specified by substantive law, are present. Thus, respondent Judge, in considering as evidence the three receipts of tax payments issued by the Municipal Treasurer of Banawe, Ifugao, exceeded his jurisdiction amounting to grave abuse of discretion when he considered matters of defense extrinsic to the allegations in the Information and which should be substantiated during the trial. The rationalization by respondent Judge that the taking away of sand and gravel was without malice because it was done with the knowledge and participation of the Government since private respondents had paid taxes on the sand and gravel extracted is not well-taken. In crimes punished by special laws, the act alone, irrespective of its motives, constitutes the offense.
PEOPLE VS. LACSON G.R. NO. 149453 MAY 28, 2002

FACTS: On June 1, 1995, Chief Superintendent Job A. Mayo, PNP Director for Investigation, filed murder charges with the Office of the Ombudsman against ninety-seven (97) officers and personnel of ABRITFG. The next-of-kin of the slain KBG members also filed murder charges against the same officers and personnel. The Ombudsman filed before the Sandiganbayan eleven (11) Informations for MURDER, docketed as Criminal Cases Nos. 23047 to 23057, against respondent Panfilo M. Lacson and twenty-five (25) other accused. All twenty-six (26) of them were charged as principals. Upon motion of the respondent, the criminal cases were remanded to the Ombudsman for reinvestigation. On March 1, 1996, Amended Informations were filed against the same twenty-six (26) suspects but the participation of respondent Lacson was downgraded from principal to accessory. Arraignment then followed and respondent entered a plea of not guilty With the downgrading of charges against him, respondent Lacson questioned the jurisdiction of theSandiganbayan to hear the criminal cases as none of the "principal" accused in the Amended Informations was a government official with a Salary Grade (SG) 27 or higher, citing Section 2 of R. A. No. 7975 then prevailing. Accordingly, the Sandiganbayan ordered the cases transferred to the Regional Trial Court. In Lacson v. Executive Secretary, respondent Lacson challenged the constitutionality of the amendment and contended that the Sandiganbayan had no jurisdiction over the criminal cases. This Court, while dismissing the constitutional challenge, nonetheless ordered the transfer of the criminal cases to the Regional Trial Court on the ground that the Amended Informations for murder failed to indicate that the

55

offenses charged therein were committed in relation to, or in discharge of, the official functions of the respondent, as required by R. A. No. 8249. Before the accused could be arraigned, prosecution witnesses Eduardo de los Reyes, Corazon de la Cruz, Armando Capili and Jane Gomez recanted their affidavits which implicated respondent Lacson in the murder of the KBG members.On the other hand, private also executed their respective affidavits of desistance declaring that they were no longer interested to prosecute these cases THUS, Judge Agnir issued a Resolution dismissing Criminal Cases because the Informations in support thereof have been rendered meaningless, if not absurd, with the recantation of the principal prosecution witnesses and the desistance of the private complainants. There is no more evidence to show that a crime has been committed and that the accused are probably guilty thereof. On March 27, 2001, PNP Director Leandro R. Mendoza indorsed to the Department of Justice the new affidavits of P/Insp. Ysmael S. Yu and P/S Insp. Abelardo Ramos regarding the Kuratong Baleleng incident for preliminary investigation. On the strength of this indorsement, Secretary of Justice Hernando B. Perez formed a panel to investigate the matter. On April 17, 2001, the respondent was subpoenaed to attend the investigation of Criminal Cases Nos. Q-99-81679 to Q-99-81689. On May 28, 2001, respondent Lacson, et al., invoking, among others, their constitutional right against double jeopardy, filed a petition for prohibition with application for temporary restraining order and/or writ of preliminary injunction with the Regional Trial Court of Manila, primarily to enjoin the State prosecutors from conducting the preliminary investigation. However, the same was denied by Judge Pasamba. On June 6, 2001, eleven (11) Informations for murder involving the killing of the same members of theKuratong Baleleng gang were filed before the Regional Trial Court of Quezon City and were docketed as Criminal Cases Nos. 01-101102 to 01-101112. The new Informations charged as principals thirty-four (34) people, including respondent Lacson and his twenty-five (25) other co-accused. On the same day, respondent Lacson filed before the Court of Appeals a petition for certiorari31 against Judge Pasamba, the Secretary of Justice, the PNP Chief, State Prosecutors Ong and Zacarias, 2nd Assistant City Prosecutor Jamolin, and the People of the Philippines. The said petition was amended to implead as additional party-respondents State Prosecutor Claro Arellano and the RTC, Quezon City, Branch 81 in which the Informations in Criminal Cases Nos. 01-101102 to. . CA found Judge Pasamba committed grave abuse of discretion. The Court of Appeals (Special Third Division), rendered the now assailed Decision. It characterized the termination of Criminal Cases Nos. Q-99-81679 to Q-99-81689 as "provisional dismissal," and considered Criminal Cases Nos. 01-101102 to 01-101112 as mere revivals of the same. Applying Section 8, Rule 117 of the 2000 Revised Rules of Criminal Procedure, it dismissed the criminal cases against the respondent. ISSUE: Whether Section 8, Rule 117 bars the filing of the eleven (11) informations against the respondent Lacson involving the killing of some members of the Kuratong Baleleng gang. HELD:

56

Like any other favorable procedural rule, this new rule can be given retroactive effect. However, this Court cannot rule on this jugular issue due to the lack of sufficient factual bases. Thus, there is need of proof of the following facts, viz: (1) whether the provisional dismissal of the cases had the express consent of the accused; (2) whether it was ordered by the court after notice to the offended party, (3) whether the 2-year period to revive has already lapsed, and (4) whether there is any justification for the filing of the cases beyond the 2-year period. The records of the case, however, do not reveal with equal clarity and conclusiveness whether notices to the offended parties were given before the cases against the respondent Lacson were dismissed by then Judge Agnir. It appears from the resolution of then Judge Agnir that the relatives of the victims who desisted did not appear during the hearing to affirm their affidavits. Their affidavits of desistance were only presented by Atty. Godwin Valdez who testified that he assisted the private complainants in preparing their affidavits and he signed them as a witness. The fact of notice to the offended parties was not raised either in the petition for prohibition with application for temporary restraining order or writ of preliminary injunction filed by respondent Lacson in the RTC of Manila, presided by Judge Pasamba, to enjoin the prosecutors from reinvestigating the said cases against him. Nor was the fact of notice to the offended parties the subject of proof after the eleven (11) informations for murder against respondent Lacson and company were revived in the RTC of Quezon City presided by Judge Yadao. This is not to be wondered at. The applicability of Section 8, Rule 117 was never considered in the trial court. It was in the Court of Appeals where respondent Lacson raised for the first time the argument that Section 8, Rule 117 bars the revival of the multiple murder cases against him. The reception of evidence on these various issues cannot be done in this Court but before the trial court. the case at bar is remanded to the RTC - Quezon City, Branch 81 so that the State prosecutors and the respondent Lacson can adduce evidence and be heard on whether the requirements of Section 8, Rule 117 have been complied with.
PEOPLE VS NAVARRO 414 SCRA 395 G. R. NO. 137597, OCTOBER 24, 2003

FACTS: On a certain night, Jason Navarro, Solomon Navarro and Roberto Olila, together with Reynante Olila, riding in a Tamaraw FX, approached Josefa Noel, a 16-year old college student, to ask for direction. As the group still seemed to be confused or lost with the direction given, Josefa offered to accompany them. When they reached the place, Josefa wanted to go down but Jason drove the vehicle so she may not go. The group bought some drinks and drank somewhere together with the girl until dawn. They left the place with Jason driving the car, she at the passenger seat, and the others on the backseats. Jason suddenly stopped the vehicle and kissed her while Solomon held her from the backseat. Jason then raped her. She escaped when Jason looked for a better position and so she was able to go out of the vehicle and then she happened to run into the direction of one passer-by named Nestor Igot. She was then brought to the police station and in the same morning, the accused were arrested. The trial court convicted the Jason and Solomon of the crime of rape. The two accused appealed to the Supreme Court and contended that the trial erred in finding them guilty when the information and the affidavit of the victim failed to allege force and intimidation in the complaint. ISSUE: 57

Whether or not the accused can be convicted under the information charging them of rape where the information failed to specify the acts which constituted the said crime. HELD: Generally no; but there are exceptions. While generally an accused cannot be convicted of an offense that is not clearly charged in the information, this rule is not without exception. The right to assail the sufficiency of the information or the admission of evidence may be waived by the accused. In People v. Torellos, this Court held: Appellant contends that the information failed to specify the acts which constituted the crime. It is too late in the day for him to assail the insufficiency of the allegations in the information. He should have raised this issue prior to his arraignment by filing a motion to quash. Failing to do so, he is deemed to have waived any objection on this ground pursuant to Rule 117, Section 9 (formerly Section 8) of the Revised Rules of Criminal Procedure, to wit: Failure to move to quash or to allege any ground therefore. The failure of the accused to assert any ground of a motion to quash before he pleads to the complaint or information, either because he did not file a motion to quash or failed to allege the same in said motion, shall be deemed a waiver of any objections based in the grounds provided for in paragraphs (a), (b), (g), and (i) of section 3 of this Rule. In the case at bar, while the information failed to specifically allege that the sexual intercourse was committed through force or intimidation, the prosecution presented evidence, no objection to which was interposed by appellants, that they committed rape through force. Besides, the information alleged that the sexual intercourse was against the victims will.
PANAGUITON VS DOJ - 571 SCRA 549 G.R. NO. 167571, NOVEMBER 25, 2008

FACTS: In 1992, Rodrigo Cawili borrowed various sums of money from Luis Panaguiton. On January 1993, Cawili and his business associate, Ramon C. Tongson jointly issued in favor of petitioner three checks which bear the signature of both in payment of the said loans. Upon presentment for payment, the checks were dishonored. Luis Panaguiton made demands but to no avail and so he filed a complaint against Cawili and Tongson for violating Batas Pambansa Bilang 22 before the Quezon City Prosecutor's Office. During the preliminary investigation, only Tongson appeared and filed his counter-affidavit. Tongson alleged that he himself filed some complaints against Cawili and they are not associates. Panaguiton showed documents proving the signatures of Tongson to strengthen his complaint against Tongson. In a resolution, City Prosecutor found probable cause only against Cawili and dismissed the charges against Tongson. A case was filed against Cawili before the proper court but the petitioner filed a partial appeal before the Department of Justice. The Chief State Prosecutor Jovencito R. Zuo directed the City Prosecutor of Quezon City to conduct a reinvestigation of the case against Tongson and to refer the questioned signatures to the National Bureau of Investigation. Assistant City Prosecutor Sampaga dismissed the complaint against Tongson since the offense had already prescribed. An appeal by Panaguiton to the Department of Justice thru Undersecretary Manuel A.J. Teehankee was dismissed. But on motion for reconsideration, Undersecretary Ma. Merceditas N. Gutierrez declared that the offense had not prescribed. On motion for reconsideration, this time by Tongson, DOJ reversed and held that the offense had already prescribed.

58

ISSUE: Whether or not that the offense had already prescribed as Act No. 3326 applies to violation of special acts and that Act No. 3326 states that prescription shall be interrupted when judicial proceedings are instituted. HELD: No, the offense had not prescribed. We agree that Act. No. 3326 applies to offenses under B.P. Blg. 22. An offense under B.P. Blg. 22 merits the penalty of imprisonment of not less than thirty (30) days but not more than one year or by a fine, hence, under Act No. 3326, a violation of B.P. Blg. 22 prescribes in four (4) years from the commission of the offense or, if the same be not known at the time, from the discovery thereof. Nevertheless, we cannot uphold the position that only the filing of a case in court can toll the running of the prescriptive period. It must be pointed out that when Act No. 3326 was passed on 4 December 1926, preliminary investigation of criminal offenses was conducted by justices of the peace, thus, the phraseology in the law, "institution of judicial proceedings for its investigation and punishment," and the prevailing rule at the time was that once a complaint is filed with the justice of the peace for preliminary investigation, the prescription of the offense is halted. In Ingco v. Sandiganbayan and Sanrio Company Limited v. Lim, which involved violations of the AntiGraft and Corrupt Practices Act (R.A. No. 3019) and the Intellectual Property Code (R.A. No. 8293), which are both special laws, the Court ruled that the prescriptive period is interrupted by the institution of proceedings for preliminary investigation against the accused. In the more recent case of Securities and Exchange Commission v. Interport Resources Corporation, et al., the Court ruled that the nature and purpose of the investigation conducted by the Securities and Exchange Commission on violations of the Revised Securities Act, another special law, is equivalent to the preliminary investigation conducted by the DOJ in criminal cases, and thus effectively interrupts the prescriptive period. The following disquisition in the Interport Resources case is instructive, thus: While it may be observed that the term "judicial proceedings" in Sec. 2 of Act No. 3326 appears before "investigation and punishment" in the old law, with the subsequent change in set-up whereby the investigation of the charge for purposes of prosecution has become the exclusive function of the executive branch, the term "proceedings" should now be understood either executive or judicial in character: executive when it involves the investigation phase and judicial when it refers to the trial and judgment stage. With this clarification, any kind of investigative proceeding instituted against the guilty person which may ultimately lead to his prosecution should be sufficient to toll prescription. Indeed, to rule otherwise would deprive the injured party the right to obtain vindication on account of delays that are not under his control. A clear example would be this case, wherein petitioner filed his complaint-affidavit on 24 August 1995, well within the four (4)-year prescriptive period. He likewise timely filed his appeals and his motions for reconsideration on the dismissal of the charges against Tongson. He went through the proper channels, within the prescribed periods. However, from the time petitioner filed his complaint-affidavit with the Office of the City Prosecutor (24 August 1995) up to the time the DOJ issued the assailed resolution, an aggregate period of nine (9) years had elapsed. Clearly, the delay was beyond petitioner's control. After all, he had already initiated the active prosecution of the case as early as 24 August 1995, only to suffer setbacks because of the DOJ's flip-flopping resolutions and its misapplication of Act No. 3326. Aggrieved parties, especially those who do not sleep on their rights and actively pursue their causes, should not be allowed to suffer unnecessarily further simply 59

because of circumstances beyond their control, like the accused's delaying tactics or the delay and inefficiency of the investigating agencies. We rule and so hold that the offense has not yet prescribed. Petitioner's filing of his complaint-affidavit before the Office of the City Prosecutor on 24 August 1995 signified the commencement of the proceedings for the prosecution of the accused and thus effectively interrupted the prescriptive period for the offenses they had been charged under B.P. Blg. 22. Moreover, since there is a definite finding of probable cause, with the debunking of the claim of prescription there is no longer any impediment to the filing of the information against petitioner.
JUMAQUIO VS VILLAROSA - 576 SCRA 204 G.R. NO. 165924, JANUARY 19, 2009

FACTS: Resty Jumaquio allegedly threatened and assaulted two young men, then ages 13 and 17. Resty, upon seeing the younger child, yelled at the child with some threats. That evening too, while the minors and their mother were traversing the road fronting another neighbors house, Resty, who was then having a drinking session, cursed them. The mother cursed him back and Resty then threw a stone towards the older child, but missed him. When the childrens father went out of their nearby house, Resty picked up another stone to fling towards the father, but the older child rushed to Resty to grab it. At that moment, Resty repeatedly punched the 17-year-old. The younger child came to the rescue, but he too received a blow on his left cheek. The family hurried home when Resty bellowed at his son for the latter to get a gun. Resty then pelted stones at the familys house while shouting and threatening. On account of that altercation, two separate informations were filed with the RTC; one with the crime of grave threat and the other one with physical injury. The trial court issued arrest warrant but the accused posted a bail. After posting bail and before the arraignment, Resty moved for the quashal of the informations for being duplicitous. He argued that, under the informations, he stood charged with several crimes - grave threats and violation of Republic Act (R.A.) No. 7610, and physical injuries and another violation of the aforesaid law. RTC denied the motion. Resty then filed directly before the Supreme Court the instant petition for certiorari under Rule 65. ISSUE: Whether or not the accused can seek relief by filing certiorari when his motion to quash was denied by the Regional Trial Court. HELD: No, petitioners remedy is not certiorari. As a rule, when a motion to quash in a criminal case is denied, petitioners remedy is not certiorari, but to go to trial without prejudice to reiterating the special defenses invoked in his motion to quash. In the event that an adverse decision is rendered after trial on the merits, an appeal therefrom is the next appropriate legal step.
PEOPLE VS DUMLAO - 580 SCRA 409 G.R. NO. 168918, MARCH 2, 2009

FACTS: An amended information was filed before the Sandiganbayan charging Dumlao and Lao, Aber P. Canlas, Jacobo C. Clave, Roman A. Cruz, Jr. and Fabian C. Ver with violation of Anti-Graft and Corrupt Practices 60

Act. The information alleged that Hermenegildo C. Dumlao, Aber Canlas, Jacobo C. Clave, Roman A. Cruz, Jr., and Fabian C. Ver, being then the members of the Board of Trustees of the Government Service Insurance System, conspired and confederated together and mutually helped one another, while in the performance of their official functions, entered into contract of lease-purchase with Emilio G. Lao, a private person whereby the GSIS agreed to sell to said Emilio G. Lao, a GSIS acquired property consisting of three parcels of land together with a 5-storey building situated in Ermita, Manila for the sum of P2,000,000.00 with a down payment of P200,000.00 with the balance payable in fifteen years at 12% interest per annum compounded yearly, with a yearly amortization of P264,278.37 including principal and interest granting Emilio G. Lao the right to sub-lease the ground floor for his own account during the period of lease, from which he collected yearly rentals in excess of the yearly amortization which contract is manifestly and grossly disadvantageous to the government. When arraigned, Dumlao pleaded not guilty. As agreed by the prosecution and Dumlao, a stipulation of facts and admission of exhibit was submitted to the court and on the basis of it, the court issued PreTrial Order. Dumlao then filed a motion to quash on the ground that the facts charged do not constitute an offense. Sandiganbayan dismissed the case.. ISSUE: Whether or not the Sandiganbayan erred in granting the motion to quash information by the accused after the pre-trial and before the petitioner could present its witnesses and formally offer its exhibits. HELD: Yes, Sandiganbayan should not have dismissed the case on the motion to quash by the accused. From the reasoning given by the Sandiganbayan, it is clear that it dismissed the case because of insufficiency of evidence. Insufficiency of evidence is not one of the grounds of a Motion to Quash. The grounds, as enumerated in Section 3, Rule 117 of the Revised Rules of Criminal Procedure, are as follows: (a) That the facts charged do not constitute an offense; (b) That the court trying the case has no jurisdiction over the offense charged; (c) That the court trying the case has no jurisdiction over the person of the accused; (d) That the officer who filed the information had no authority to do so; (e) That it does not conform substantially to the prescribed form; (f) That more than one offense is charged except when a single punishment for various offenses is prescribed by law; (g) That the criminal action or liability has been extinguished; (h) That it contains averments which, if true, would constitute a legal excuse or justification; and (i) That the accused has been previously convicted or acquitted of the offense charged, or the case against him was dismissed or otherwise terminated without his express consent. Insufficiency of evidence is a ground for dismissal of an action only after the prosecution rests its case. Section 23, Rule 119 of the Revised Rules of Criminal Procedure provides: Sec. 23. Demurrer to evidence. After the prosecution rests its case, the court may dismiss the action on the ground of insufficiency of evidence (1) on its own initiative after giving the prosecution the

61

opportunity to be heard or (2) upon demurrer to evidence filed by the accused with or without leave of court. In the case under consideration, the Sandiganbayan dismissed the case against respondent for insufficiency of evidence, even without giving the prosecution the opportunity to present its evidence. In so doing, it violated the prosecutions right to due process. It deprived the prosecution of its opportunity to prosecute its case and to prove the accuseds culpability. It was therefore erroneous for the Sandiganbayan to dismiss the case under the premises. Not only did it not consider the ground invoked by respondent Dumlao; it even dismissed the case on a ground not raised by him, and not at the appropriate time. The dismissal was thus without basis and untimely.
SORIANO VS PEOPLE - 591 SCRA 244 G.R. NO. 159517-18, JUNE 30, 2009

FACTS: Hilario P. Soriano and Rosalinda Ilagan were the President and General Manager, respectively, of the Rural Bank of San Miguel, Inc. (RBSM). During their incumbency as president and manager of the bank, they indirectly obtained loans from RBSM by falsifying loan applications and other bank records and made it appear that Virgilio J. Malang and Rogelio Maaol obtained loans of P15,000,000.00 each. The State Prosecutor charged Soriano in the Regional Trial Court for the violation of General Banking Act or for the violation of the Director, Officer, Stockholder or Related Interest (DOSRI) Rules. On the same date, an information for estafa thru falsification of commercial document was also filed against Soriano and Ilagan covering the amount of loan supposedly made by Virgilio J. Malang. Both informations were raffled to Branch 14 of the RTC. Another information for violation of Section 83 of R.A. No. 337, as amended, was filed against Soriano, this time, covering the loan obtained in the name of Rogelio Maaol. Soriano and Ilagan were also indicted for estafa thru falsification of commercial document for obtaining said loan. These information were raffled to Branch 77. The accused moved to quash the informations in the pending cases before the two branches of the RTC on grounds that more than one offense is charged and that the facts charged do not constitute an offense. The accused argued that the prosecutor charged more than one offense for a single act. Soriano was charged with violation of DOSRI rules and estafa thru falsification of commercial document for allegedly securing fictitious loans. They further argued that the facts as alleged in the information do not constitute an offense. Both denied the motion to quash. They appealed to the Court of Appeals via certiorari and their appeal was also denied. They went to the Supreme Court. ISSUE: Whether or not the accused can file a motion to quash information against them on the ground that more than one offense are charged against them. HELD: No, their motion will be denied as there are no grounds to quash the information. Petitioners assail the validity of the informations against them on the ground that more than one (1) offense is charged. They point that Soriano was charged with violation of DOSRI Rules and with estafa

62

thru falsification of commercial document for allegedly obtaining loans from RBSM. Thus, they claim that the informations were duplicitous; hence, they should be quashed. Indisputably, duplicity of offenses in a single information is a ground to quash the Information under Section 3(e), Rule 117 of the 1985 Rules of Criminal Procedure. The Rules prohibit the filing of a duplicitous information to avoid confusing the accused in preparing his defense. By duplicity of charges is meant a single complaint or information that charges more than one offense. Section 13 of Rule 110 of the 1985 Rules on Criminal Procedure clearly states: Duplicity of Offense. A complaint or information must charge but one offense, except only in those cases in which existing laws prescribe a single punishment for various offenses. Otherwise stated, there is duplicity (or multiplicity) of charges when a single Information charges more than one offense. In this case, however, Soriano was faced not with one information charging more than one offense, but with more than one information, each charging a different offense - violation of DOSRI rules in one, and estafa thru falsification of commercial documents in the others. Ilagan, on the other hand, was charged with estafa thru falsification of commercial documents in separate informations. Thus, petitioners erroneously invoke duplicity of charges as a ground to quash the Informations.
PEOPLE VS. TAN G.R. NO. 167526 JULY 26, 2010

Facts: Two informations were filed against Dante Tan, where he is being accused of being the beneficial owner of 84,030,000 Best World Resources Corporation shares where he did then and there willfully, unlawfully and criminally fail to file with the Securities and Exchange Commission and with the Philippine Stock Exchange a sworn statement of the amount of all BWRC shares of which he is the beneficial owner. The accused pleaded not guilty after he was arraigned and he began the formal presentation of evidence, soon after the RTC issued an order admitted some of evidence presented. The petitioner filed a Motion for Reconsideration, but it was denied by the RTC. In the meantime, respondent filed an Omnibus Motion for Leave to File Demurrer to Evidence and to admit the attached Demurrer to Evidence. The RTC issued another Order granting respondents Motion for Leave to File the Demurrer and admitted respondents attached Demurrer. The RTC also ordered petitioner to file an opposition. The petitioner filed its Opposition to the Demurrer to Evidence. Respondent then filed a Reply. Then after, the RTC issued an Order granting respondents Demurrer to Evidence. The lower couth then state that it would be futile to take further action on the petition, which is therefore DISMISSED outright for evident want of merit. In denying the petition, the CA ruled that the dismissal of a criminal action by the grant of a Demurrer to Evidence is one on the merits and operates as an acquittal, for which reason, the prosecution cannot appeal therefrom as it would place the accused in double jeopardy. The petitioner however, argues in the negative, that there is no double jeopardy and that the court acted with grave abuse of discretion in denying it.

63

Issue/s: 1) Whether there is double jeopardy to preclude the people from prosecuting the accused. 2) Whether the exception applies in the case at bar. Held / Ruling: 1) YES. The elements of double jeopardy are (1) the complaint or information was sufficient in form and substance to sustain a conviction; (2) the court had jurisdiction; (3) the accused had been arraigned and had pleaded; and (4) the accused was convicted or acquitted, or the case was dismissed without his express consent. These elements are present here: (1) the Informations filed against respondent were sufficient in form and substance to sustain a conviction; (2) the RTC had jurisdiction over the Criminal cases; (3) respondent was arraigned and entered a plea of not guilty; and (4) the RTC dismissed the cases on a demurrer to evidence on the ground of insufficiency of evidence which amounts to an acquittal from which no appeal can be had. 2) NO, This Court finds that the RTC did not abuse its discretion in the manner it conducted the proceedings of the trial, as well as its grant of respondents demurrer to evidence. The petitioner was given more than ample opportunity to present its case as gleaned from the factual antecedents which led to the grant of respondents demurrer. Hence his right to due process was properly observed.
JOSEPH C. CEREZO VS. PEOPLE G.R. NO. 185230, JUNE 01, 2011

Facts: The petitioner Joseph Cerezo filed a complaint for libel against the respondents. Finding probable cause to indict them, the Quezon City Prosecutors Office (OP-QC) filed the corresponding Information against before the RTC. Soon after they were arraigned and all of them entered a not guilty plea. The Court, therefore, after hearing and conferring with the fiscal, can dismiss the case if convinced that there is no reason to continue with the prosecution. As in this case, the Court finds merit in the motion of the Public Prosecutor. Aggrieved, petitioner moved for reconsideration of the said Order, arguing that the November 20, 2003 OP-QC resolution has not yet attained finality. Comes now The Secretary of Justice, which promulgated his resolution reversing and setting aside the OP-QCs November 20, 2003 resolution, and directing the latter to re -file the earlier Information for libel. Considering the findings of the Department of Justice reversing the resolution of the City Prosecutor, the Court gives favorable action to the Motion for Reconsideration. In the same manner as discussed in arriving at its assailed order dated 17 March 2004, the Court gives more leeway to the Public Prosecutor in determining whether it has to continue or stop prosecuting a case. While the City Prosecutor has previously decided not to pursue further the case, the Secretary of Justice, however, through its resolution on the Petition for Review did not agree with him. The Court disagrees with the argument raised by the accused that double jeopardy sets in to the picture. The order of dismissal as well as the withdrawal of the Information was not yet final because of the timely filing of the Motion for Reconsideration. The Court therefore, can still set aside its order. Moreover, there is no re-filing of the case nor the filing of a new one. The case filed remains the same 64

and the order of dismissal was merely vacated because the Court finds the Motion for Reconsideration meritorious. The case was elevated to the CA, which impugned RTC Orders, and ruled that all the elements of double jeopardy exist. Issue: Whether there was a valid termination of the case so as to usher in the impregnable wall of double jeopardy. Held / Ruling: In resolving a motion to dismiss a case or to withdraw an Information, the trial court should not rely solely on the findings of the public prosecutor or the Secretary of Justice. As to the order of the RTC, dismissing the criminal case, that the RTC judge failed to make his own determination of whether or not there was a prima facie case to hold respondents for trial. He failed to make an independent evaluation or assessment of the merits of the case. It is beyond cavil that double jeopardy did not set in. Double jeopardy exists when the following requisites are present: (1) a first jeopardy attached prior to the second; (2) the first jeopardy has been validly terminated; and (3) a second jeopardy is for the same offense as in the first. A first jeopardy attaches only (a) after a valid indictment; (b) before a competent court; (c) after arraignment; (d) when a valid plea has been entered; and (e) when the accused has been acquitted or convicted, or the case dismissed or otherwise terminated without his express consent. The respondents were not acquitted nor were there a valid and legal dismissal or termination of the case. Double Jeopardy has not set in.

LUGTU VS. COURT OF APPEALS

Facts: The Petitioner Domingo V. Lugtu, together with private respondent Rosa L. Cancio and Clodualdo F. Vitug were charged with estafa. The petitioner is a Bank Teller in Continental Bank. He claims that he was successfully convinced by the private respondent to participate in swindling the said bank. The case circulates around the decision of the court, which authorized the discharge of petitioner Domingo V. Lugtu from the information so that he could be utilized as witness for the government. At the arraignment, all the accused pleaded not guilty. After presenting three (3) witnesses, the Provincial Fiscal filed with the trial court a motion to discharge the accused Domingo V. Lugtu for the purpose of utilizing him as state witness against his co-accused. Alleging that the trial judge committed a grave abuse of discretion, or acted in excess of his jurisdiction in denying the discharge or Lugtu as a state witness against his co-accused. Issue/s:

65

Whether respondent appellate court erred in finding that the conditions required under Section 9, Rule 119 of the Rules of Court as to deny his discharge as a state witness. Held/Ruling: There was a valid ground for claiming the Sec. 9, Rule 119 of the Rules of Court; the court committed an error. In order to validly claim an accused as a state witness it requires that it must first comply with the laws requirements, Section 9, Rule 119 of the Rules reads as follows: (a) There is absolute necessity for the testimony of the defendant whose discharge is requested; (b) There is no other direct evidence available for the proper prosecution of the offense committed, except the testimony of said defendant; (c) The testimony of said defendant can be substantially corroborated in its material points; (d) Said defendant does not appear to be the most guilty; (e) Said defendant has not at any time been convicted of any offense involving moral turpitude. The respondent appellate court considered the sworn executed by accused Lugtu wherein he admitted his responsibility in the commission of the offense. The Court is of the opinion that there is no reasonable ground to set the former decision aside. The trial court after thoroughly and exhaustively examining and evaluating the facts and evidence on record, found Lugtu not to be the most guilty. Being a poor and ignorant man, he was easily convinced by Vitug and Cancio As for the testimony of the three (3) prosecution witnesses, petitioners assert that, as correctly ruled by the trial court, the said testimony can substantially corroborate the testimony of Lugtu in its material points. The testimony of the three (3) witnesses centered on the modus operandi of the swindle perpetrated by the three (3) accused. The finding of the trial court that the testimony of Lugtu would be the direct evidence to link the events starting from the opening of the checking account up to the time the checkbook in question found its way to the Philippine National Bank branch in Balanga. The discharge of an accused should be availed of only when there is absolute necessity for the testimony of said accused whose discharge is requested, as when he alone has knowledge of the crime, and not when his testimony would simply corroborate or otherwise strengthen the evidence in the hands of the prosecution.

SALVANERA VS. PEOPLE G.R. NO. 143093 MAY 21, 2007

Facts: The petitioner contests the decision of the court which discharged the accused Feliciano Abutin and Domingo Tampelix from the Information in Criminal Case for the Murder of Ruben Parane, pending before the Regional Trial Court of Trece Martires City, to become state witnesses. The appellate court likewise cancelled his bail bond. 66

The trial court granted bail of the petioner but denied the discharge of the accused Abutin and tampelix. The prosecution elevated the case to the CA and argued that the testimonies of the two accused are absolutely necessary to establish that petitioner masterminded the murder of Ruben Parane. The prosecution likewise claimed that it was premature to allow the petitioner bail as they have not even rested their case Issue/s: Whether there is sufficient ground to discharge the accused Abutin and Tampelix to be a state witness against the petitioner. Held/ruling: YES. There is sufficient ground.The court is satisfied that: a) There is absolute necessity for the testimony of the accused whose discharge is requested; b) There is no other direct evidence available for the proper prosecution of the offense committed, except the testimony of said accused; c) The testimony of said accused can be substantially corroborated in its material points; d) Said accused does not appear to be the most guilty; and, e) Said accused has not at any time been convicted of any offense involving moral turpitude. However, the petitioner argued that both Abutin and Tampelix will naturally seize the opportunity to be absolved of any liability by putting the blame on one of their co-accused. Petitioner argues that prosecution witnesses Parane and Salazar, who are not accused, do not have personal knowledge of the circumstances surrounding the alleged conspiracy. Thus, they could not testify to corroborate the statement of Abutin and Tampelix that petitioner is the mastermind or the principal by induction. The court dismissed their reasoning. What is needed is that the corroborative evidence required by the Rules does not have to consist of the very same evidence as will be testified on by the proposed state witnesses. We have ruled that "a conspiracy is more readily proved by the acts of a fellow criminal than by any other method. If it is shown that the statements of the conspirator are corroborated by other evidence, then we have convincing proof of veracity. Even if the confirmatory testimony only applies to some particulars, we can properly infer that the witness has told the truth in other respects."

MANGUERRA VS. RISOS G.R. NO. 152643, AUGUST 28, 2008

Facts: Respondents were charged with Estafa Through Falsification of Public Document, arising from a deed of real estate mortgage allegedly committed by respondents where they made it appear that Concepcion, the owner of the mortgaged property known as the Gorordo property, affixed her signature to the document. Concepcion, who was a resident of Cebu City, while on vacation in Manila, was unexpectedly confined at the Makati Medical Center due to upper gastro-intestinal bleeding; and was advised to stay in Manila for further treatment.

67

Respondents filed a Motion for Suspension of the Proceedings on the ground of prejudicial question. They argued that the Civil case, which was an action for declaration of nullity of the mortgage, should first be resolved. On May 11, 2000, the RTC granted the motion. Concepcion's motion for reconsideration was denied. Concepcions counsel filed a motion for deposition, which The Court then granted.. The court ratiocinated that procedural technicalities should be brushed aside because of the urgency of the situation, since Concepcion was already of advanced age. After several motions for change of venue of the deposition-taking, Concepcion's deposition was finally taken on March 9, 2001 at her residence. Aggrieved, respondents assailed the decision, and the CA rendered a decision in their favor which rendered the depositions void. On the outset, the CA observed that there was a defect in the respondents' petition by not impleading the People of the Philippines, an indispensable party. This notwithstanding, the appellate court resolved the matter on its merit, declaring that the examination of prosecution witnesses, as in the present case, is governed by Section 15, Rule 119 of the Revised Rules of Criminal Procedure and not Rule 23 of the Rules of Court. The latter provision, said the appellate court, only applies to civil cases. Pursuant to the specific provision of Section 15, Rule 119, Concepcion's deposition should have been taken before the judge or the court where the case is pending, which is the RTC of Cebu, and not before the Clerk of Court of Makati City; and thus, in issuing the assailed order, the RTC clearly committed grave abuse of discretion. Issue/s: 1) Whether rule 23 of the 1997 rules of civil procedure applies to the deposition of petitioner. 2) Whether failure to implead the "people of the Philippines" in a petition for certiorari arising from a criminal case a quo constitutes a waivable defect in the petition for certiorari. Held / Ruling: 1) NO. In criminal proceedings, Sections 12, 13 and 15, Rule 119 of the Revised Rules of Criminal Procedure allows the conditional examination of both the defense and prosecution witnesses. As exceptions, Rules 23 to 28 of the Rules of Court provide for the different modes of discovery that may be resorted to by a party to an action. These rules are adopted either to perpetuate the testimonies of witnesses or as modes of discovery. The conditional examination of a prosecution witness for the purpose of taking his deposition should be made before the court, or at least before the judge, where the case is pending. Such is the clear mandate of Section 15, Rule 119 of the Rules. The very reason offered by the petitioners to exempt Concepcion from the coverage of Rule 119 is at once the ground, which places her squarely within the coverage of the same provision. Rule 119 specifically states that a witness may be conditionally examined: 1) if the witness is too sick or infirm to appear at the trial; or 2) if the witness has to leave the Philippines with no definite date of returning. Thus, when Concepcion moved that her deposition be taken, had she not been too sick at that time, her motion would have been denied. Instead of conditionally examining her outside the trial court, she would have been compelled to appear before the court for examination during the trial proper. 68

It is thus required that the conditional examination be made before the court where the case is pending. It is also necessary that the accused be notified, so that he can attend the examination, subject to his right to waive the same after reasonable notice. As to the manner of examination, the Rules mandate that it be conducted in the same manner as an examination during trial, that is, through question and answer.

2) YES, it is a waivable defect. The court has repeatedly declared that the failure to implead an indispensable party is not a ground for the dismissal of an action. In such a case, the remedy is to implead the non-party claimed to be indispensable. Parties may be added by order of the court, on motion of the party or on its own initiative at any stage of the action and/or such times as are just. In this case, the CA disregarded the procedural flaw by allowing the petition to proceed, in the interest of substantial justice.
CABADOR VS. PEOPLE G.R. NO. 186001 OCTOBER 2, 2009

Facts: The petitioner was accused of murder before the RTC on June 23, 2000. In 2006, after five years of intermittent trial, only 5 witnesses were presented. The RTC required the prosecution to make a written or formal offer of its documentary evidence. But the public prosecutor asked for three extensions of time. Still, the prosecution did not make the required written offer. On August 1, 2006 petitioner Cabador filed a motion to dismiss the case invoking his right to a speedy trial. He also claimed that in the circumstances, the trial court could not consider any evidence against him that had not been formally offered. Unknown to petitioner Cabador, an offer was made by the prosecution on August 1, 2006, the day Cabador filed his motion to dismiss. On August 31, 2006 the RTC issued an Order treating petitioner Cabadors August 1, 2006 motion to dismiss as a demurrer to evidence. And, since he filed his motion without leave of court, the RTC declared him to have waived his right to present evidence in his defense. The trial court deemed the case submitted for decision. Cabador filed a motion for reconsideration of this Order but the RTC denied it. Cabador questioned the RTCs actio ns before the CA but the latter denied his petition and affirmed the lower courts actions. Issue: Whether or not petitioner Cabadors motion to dismiss before the trial court was in fact a demurrer to evidence filed without leave of court, with the result that he effectively waived his right to present evidence in his defense and submitted the case for decision insofar as he was concerned. Held: No. Demurrer to evidence assumes that the prosecution has already rested its case. Section 23, Rule 119 of the Revised Rules of Criminal Procedure, reads: Demurrer to evidence. After the prosecution rests its case, the court may dismiss the action on the ground of insufficiency of evidence (1) on its own initiative after giving the prosecution the opportunity to be heard or (2) upon demurrer to the evidence filed by the accused with or without leave of court. Here, after the prosecution filed its formal offer of exhibits on August 1, 2006, the same day Cabador filed his motion to dismiss, the trial court still needed to give him an opportunity to object to the admission of those exhibits. It also needed to rule on the formal offer. And only after such a ruling could the prosecution be deemed to have rested its case. Since Cabador filed his motion to dismiss 69

before he could object to the prosecutions formal offer, before the trial court could act on the offer, and before the prosecution could rest its case, it could not be said that he had intended his motion to dismiss to serve as a demurrer to evidence. In sum, the Court finds that petitioner Cabador filed a motion to dismiss on the ground of violation of his right to speedy trial, not a demurrer to evidence. He cannot be declared to have waived his right to present evidence in his defense.
PEOPLE VS. DE GRANO G.R. NO. 167710 JUNE 5, 2009

Facts: On November 28, 1991, an Information for murder was filed with the RTC against Joven de Grano (Joven), Armando de Grano (Armando), and Estanislao Lacaba (Estanislao), together with their coaccused Leonides Landicho (Leonides), Domingo Landicho (Domingo), and Leonardo Genil (Leonardo), who were at-large. Duly arraigned, Joven, Armando, and Estanislao pleaded not guilty to the crime as charged; while their co-accused Leonides, Leonardo, and Domingo remained at-large. Thereafter, respondents filed a motion for bail contending that the prosecutions evidence was not strong. RTC found the accused guilty of the offenses charged. In 2004 an order was issued that modified the previous decision, from murder the case was downgraded to homicide. However, Joven, Armando, and Domingo was not present during promulgation. They maintained that while they were not present during the promulgation of the RTC Decision, Estanislao, who was under police custody, attended the promulgation. Thus according to them, when they filed their Joint Motion for Reconsideration, which included that of Estanislao, the RTC was not deprived of its authority to resolve the joint motion. Issue: Whether or not RTC erred in taking cognizance of the joint motion for reconsideration despite the absence of the other accused during the promulgation of judgment? Held: Yes. Section 14(2),[59] Article III of the Constitution, authorizing trials in absentia, allows the accused to be absent at the trial but not at certain stages of the proceedings, to wit: (a) at arraignment and plea, whether of innocence or of guilt; (b) during trial, whenever necessary for identification purposes; and (c) at the promulgation of sentence, unless it is for a light offense, in which case, the accused may appear by counsel or representative. At such stages of the proceedings, his presence is required and cannot be waived. When the Decision dated April 25, 2002 was promulgated, only Estanislao Lacaba was present. Subsequently thereafter, without surrendering and explaining the reasons for their absence, Joven, Armando, and Domingo joined Estanislao in their Joint Motion for Reconsideration. In blatant disregard of the Rules, the RTC not only failed to cause the arrest of the respondents who were at large, it also took cognizance of the joint motion. The RTC clearly exceeded its jurisdiction when it entertained the joint Motion for Reconsideration with respect to the respondents who were at large. It should have considered the joint motion as a motion for reconsideration that was solely filed by Estanislao. Being at large, Joven and Domingo have not regained their standing in court. Once an accused jumps bail or flees to a foreign country, or escapes from prison or confinement, he loses his standing in court; and unless he surrenders or submits to the jurisdiction of the court, he is deemed to have waived any right to seek relief from the court. 70

IMPERIAL VS. JOSON G.R. NO. 160067 AND G.R. NO. 171622 NOVEMBER 17, 2010

Facts: A collision happened along the portion of the National Highway in Concepcion, Sariaya, Quezon. The Isuzu ten-wheeler truck collided with a Fuso six-wheeler truck. After colliding with the Fuso six-wheeler truck, the Isuzu ten-wheeler truck further rammed into a Kia Besta Van. There were multiple damages on the vehicles. Much more tragic than that, the accident resulted in one death, the owner of the KIA Besta Van, and seven of its passengers, all suffered serious physical injuries. A criminal complaint for Reckless Imprudence Resulting to Multiple Homicide, Multiple Serious Physical Injuries and Damage to Property was filed against petitioners Santos Francisco and Noel Imperial on 16 May 2001. During the course of the case, there have been 9 postponements. Petitioner claims that his right to speedy trial has been violated. Issue: Whether or not the postponements of the pre-trial conferences were violative of the Petitioners right to speedy trial? Held: No. Although the Revised Rules of Criminal Procedure concededly mandates commencement of the trial within 30 days from receipt of the pre-trial order and the continuous conduct thereof for a period not exceeding 180 days, Section 3 a (1), Rule 119 provides that delays resulting from extraordinary remedies against interlocutory orders shall be excluded in computing the time within which trial must commence. In determining the right of an accused to speedy trial, moreover, courts are "required to do more than a mathematical computation of the number of postponements of the scheduled hearings of the case" and to give particular regard to the facts and circumstances peculiar to each case. Viewed in the context of the above discussed procedural antecedents as well as the further reassignment of the case to Prosecutor Baligod as a consequence of Prosecutor Sias subsequent transfer to another government office, we find that the CA correctly brushed aside petitioner Francisco's claim that the postponements of the pre-trial conferences in the case before the Sariaya MTC were violative of his right to a speedy trial.
PP VS. SANDIGANBAYAN G.R. NO. 174504 MARCH 21, 2011

Facts: Manuel Barcenas, the Vice-Mayor of Toledo City was charged with violation of Section 89 of Presidential Decree (P.D.) No. 1445 before the Sandiganbayan. The said accused allegedly obtained cash advances from the City Government of Toledo in the total of (P61,765.00), which he received by reason of his office. He was to liquidate the same but failed to do so despite demands. He pleaded not guilty. The prosecution presented its lone witness, Manolo Tulibao Villad, Commission on Audit (COA) State Auditor. Thereafter, the prosecution filed its formal offer of evidence and rested its case. On April 20, 2006, private respondent filed a motion for leave to file demurrer to evidence. On June 16, 2006, the Sandiganbayan issued a Resolution granting the motion. On June 30, 2006, private respondent filed his demurrer to evidence. The said court granted the demurrer and ordered the case dismissed. Issue:

71

Whether the Sandiganbayan acted with grave abuse of discretion amounting to lack or excess of jurisdiction in giving due course to and eventually granting the demurrer to evidence. Held: An order of dismissal arising from the grant of a demurrer to evidence has the effect of an acquittal unless the order was issued with grave abuse of discretion amounting to lack or excess of jurisdiction. In criminal cases, the grant of a demurrer is tantamount to an acquittal and the dismissal order may not be appealed because this would place the accused in double jeopardy. Although the dismissal order is not subject to appeal, it is still reviewable but only through certiorari under Rule 65 of the Rules of Court. For the writ to issue, the trial court must be shown to have acted with grave abuse of discretion amounting to lack or excess of jurisdiction such as where the prosecution was denied the opportunity to present its case or where the trial was a sham thus rendering the assailed judgment void.The burden is on the petitioner to clearly demonstrate that the trial court blatantly abused its authority to a point so grave as to deprive it of its very power to dispense justice. In the case at bar, the Sandiganbayan granted the demurrer to evidence on the ground that the prosecution failed to prove that the government suffered any damage from private respondent's nonliquidation of the subject cash advance because it was later shown, as admitted by the prosecution's witness, that private respondent liquidated the same albeit belatedly.
PEOPLE VS. TAN G.R. NO. 167526 JULY 26, 2010

Facts: Two informations were filed against Dante Tan, the new beneficial owner of BWRC. According to the information he willfully, unlawfully and criminally fail to file with the Securities and Exchange Commission and with the Philippine Stock Exchange a sworn statement of the amount of all BWRC shares within ten (10) days after he became such beneficial owner, in violation of the Revised Securities Act and/or the rules and regulations prescribed and pursuant thereto. After arraignment, respondent pleaded not guilty to both charges and the trial ensued. Petitioner made its formal offer of evidence and in the meantime the respondent filed an Omnibus Motion for Leave to File Demurrer to Evidence and to admit the attached Demurrer to Evidence. RTC granted the same. Petitioner opposed but the CA ruled that the dismissal of a criminal action by the grant of a Demurrer to Evidence is one on the merits and operates as an acquittal, for which reason, the prosecution cannot appeal therefrom as it would place the accused in double jeopardy. Issue: Whether or not the dismissal of the criminal action was proper by the grant of the demurrer to evidence? Held: The general rule that the grant of a demurrer to evidence operates as an acquittal and is, thus, final and unappealable, to wit: is "filed after the prosecution had rested its case," and when the same is granted, it calls "for an appreciation of the evidence adduced by the prosecution and its sufficiency to warrant conviction beyond reasonable doubt, resulting in a dismissal of the case on the merits, tantamount to an acquittal of the accused." Such dismissal of a criminal case by the grant of demurrer to evidence may not be appealed, for to do so would be to place the accused in double jeopardy. The verdict being one of acquittal, the case ends there.

72

BANGAYAN JR. V. BANGAYAN

Facts: On March 7, 1982, Benjamin, Jr. married Sally Go in Pasig City and they had two children. Later, Sally Go learned that Benjamin, Jr. had taken Resally as his concubine whom he subsequently married on January 5, 2001 under the false name, "Benjamin Z. Sojayco." Benjamin, Jr. fathered two children with Resally. Furthermore, Sally Go discovered that on September 10, 1973, Benjamin, Jr. also married a certain Azucena Alegre (Azucena) in Caloocan City. The City Prosecutor of Caloocan City conducted a preliminary investigation and thereafter issued a Resolution dated June 5, 2002 recommending the filing of an information for bigamy against Benjamin, Jr. and Resally for having contracted a marriage despite knowing fully well that he was still legally married to Sally Go. The information was duly filed on November 15, 2002 and was raffled to the Regional Trial Court of Caloocan City. After the arraignment, during which petitioners both pleaded not guilty to the charge against them, the prosecution presented and offered its evidence. On September 8, 2003, Benjamin, Jr. and Resally separately filed their respective motions for leave to file a demurrer to evidence. This was granted by the RTC in its Order dated September 29, 2003. On October 20, 2003, Benjamin, Jr. filed his Demurrer to Evidence, praying that the criminal case for bigamy against him be dismissed for failure of the prosecution to present sufficient evidence of his guilt. His plea was anchored on two main arguments: (1) he was not legally married to Sally Go because of the existence of his prior marriage to Azucena; and (2) the prosecution was unable to show that he and the "Benjamin Z. Sojayco Jr.," who married Resally, were one and the same person. In its December 3, 2003 Order, the RTC dismissed the criminal case against Benjamin, Jr. and Resally for insufficiency of evidence. It reasoned out that the prosecution failed to prove beyond reasonable doubt that Benjamin, Jr. used the fictitious name, Benjamin Z. Sojayco Jr., in contracting his marriage with Resally. Corollarily, Resally cannot be convicted of bigamy because the prosecution failed to establish that Resally married Benjamin, Jr. Aggrieved, Sally Go elevated the case to the CA via a petition for certiorari. On March 14, 2006, the CA promulgated its Decision granting her petition and ordering the remand of the case to the RTC for further proceedings. The CA held that the following pieces of evidence presented by the prosecution were sufficient to deny the demurrer to evidence: (1) the existence of three marriages of Benjamin, Jr. to Azucena, Sally Go and Resally; (2) the letters and love notes from Resally to Benjamin, Jr.; (3) the admission of Benjamin, Jr. as regards his marriage to Sally Go and Azucena; and (4) Benjamin, Jr.'s admission that he and Resally were in some kind of a relationship. The CA further stated that Benjamin, Jr. was mistaken in claiming that he could not be guilty of bigamy because his marriage to Sally Go was null and void in light of the fact that he was already married to Azucena. A judicial declaration of nullity was required in order for him to be able to use the nullity of his marriage as a defense in a bigamy charge. Petitioners MR was denied, hence the present petition. Issue:

73

WON the demurrer to evidence granted by the RTC was proper, given the lack of conformity of the Solicitor General Ruling: YES, the demurrer to evidence granted was proper. It is well-settled that in criminal cases where the offended party is the State, the interest of the private complainant or the private offended party is limited to the civil liability. Thus, in the prosecution of the offense, the complainant's role is limited to that of a witness for the prosecution. If a criminal case is dismissed by the trial court or if there is an acquittal, an appeal therefrom on the criminal aspect may be undertaken only by the State through the Solicitor General. Only the Solicitor General may represent the People of the Philippines on appeal. The private offended party or complainant may not take such appeal. However, the said offended party or complainant may appeal the civil aspect despite the acquittal of the accused. In this case, however, neither the Solicitor General nor the City Prosecutor of Caloocan City joined the cause of Sally Go, much less consented to the filing of a petition for certiorari with the appellate court. Furthermore, she cannot claim to have been denied due process because the records show that the trial court heard all the evidence against the accused and that the prosecution had formally offered the evidence before the court granted the demurrer to evidence. Thus, the petitioners' acquittal was valid, entitling them to invoke their right against double jeopardy.

LLAMAS VS. CA Facts: Petitioners francisco and carmelita llamas were charged before the RTC of Makati of the crimeof "other forms of swindling". After trial on the merits, the RTC rendered a decision convicting the petitioners. The decision was affirmed by the CA. Assailing the aforesaid issuances of the appellate court, petitioners filed before the SC their petition for review. A warrant of arrest was issued for the petitioners to serve their sentence but only Carmelita served her 2 month sentence. Francisco moved for the lifting or recall of the warrant of arrest, raising for the first time the issue that the trial court had no jurisdiction over the offense charged. There being no action taken by the trial court on the said motion, petitioners instituted, the instant proceedings for the annulment of the trial and the appellate courts decisions. Issue: Whether the decision should be annulled? Held: No. Section 1, Rule 47 of the Rules of Court, limits the scope of the remedy of annulment of judgment to the following: Section 1. Coverage. This Rule shall govern the annulment by the Court of Appeals of judgments or final orders and resolutions in civil actions of Regional Trial Courts for which the ordinary remedies of

74

new trial, appeal, petition for relief or other appropriate remedies are no longer available through no fault of the petitioner. The remedy cannot be resorted to when the RTC judgment being questioned was rendered in a criminal case. The 2000 Revised Rules of Criminal Procedure itself does not permit such recourse, for it excluded Rule 47 from the enumeration of the provisions of the 1997 Revised Rules of Civil Procedure which have suppletory application to criminal cases. Section 18, Rule 124 thereof, provides: Sec. 18. Application of certain rules in civil procedure to criminal cases. The provisions of Rules 42, 44 to 46 and 48 to 56 relating to procedure in the Court of Appeals and in the Supreme Court in original and appealed civil cases shall be applied to criminal cases insofar as they are applicable and not inconsistent with the provisions of this Rule. There is no basis in law or the rules, therefore, to extend the scope of Rule 47 to criminal cases. As we explained in Macalalag v. Ombudsman, when there is no law or rule providing for this remedy, recourse to it cannot be allowed x x x. Here, petitioners are invoking the remedy under Rule 47 to assail a decision in a criminal case. Following Bitanga, this Court cannot allow such recourse, there being no basis in law or in the rules. In substance, the petition must likewise fail. The trial court which rendered the assailed decision had jurisdiction over the criminal case. Jurisdiction being a matter of substantive law, the established rule is that the statute in force at the time of the commencement of the action determines the jurisdiction of the court.15 In this case, at the time of the filing of the information, the applicable law was Batas Pambansa Bilang 129. The SC cited Sec. 20 and 32 of BP 129.
PEOPLE VS. BARTOLOME TAMPUS AND IDA MONTESCLAROS G.R. NO. 181084 JUNE 16, 2009

FACTS: The offended party, ABC, is the daughter of appellant Ida, and was 13 years old at the time of the incident. Ida worked as a waitress in Bayanihan Beer House in Mabini, Cebu City. On February 19, 1995, Ida and ABC started to rent a room in a house owned by Tampus, a barangay tanod. On April 1, 1995, about 4:30 p.m., ABC testified that she was in the house with Ida and Tampus who were both drinking beer at that time. They forced her to drink beer and after consuming three and one-half (3 ) glasses of beer, she became intoxicated and very sleepy. While ABC was lying on the floor of their room, she overheard Tampus requesting her mother, Ida, that he be allowed to "remedyo" or have sexual intercourse with her. Tampus succeeded in having carnal knowledge with abc. Moreover ABC testified that on April 4, 1995 around 1:00 a.m., she was left alone in the room since her mother was at work at the beer house. Tampus went inside their room and threatened to kill her if she would report the previous sexual assault to anyone. Tampus succeeded again in having carnal knowledge with abc. On September 22, 1995, ABC filed two Complaints.The trial court convicted Tampus of two counts of rape, as principal in Criminal Case No. 013324-L and Criminal Case No. 013325L. Appellant Ida was found guilty as an accomplice in Criminal Case No. 013324-L. The trial court appreciated in Idas favor the mitigating circumstance of illness which would diminish the exercise of will-power without depriving her of the consciousness of her acts, pursuant to Article 13(9) of the 75

Revised Penal Code. The dispositive portion of the trial court's decision ordered Tampus and Ida "jointly and severally, to indemnify the offended party, [ABC], the sum of P50,000.00 in Criminal Case No. 013324-L."The Court of Appeals, however, did not award any civil indemnity to ABC, and only awarded moral and exemplary damages. Tampus died pending appeal, hence his appeal was dismissed. ISSUE: Whether or not Ida is solidarity liable with Tampus in the payment of civil indemnity. HELD: No. Civil liability arising from the crime is shared by all the accused. Although, unlike criminal liabilityin which the Revised Penal Code specifically states the corresponding penalty imposed on the principal, accomplice and accessorythe share of each accused in the civil liability is not specified in the Revised Penal Code. The courts have the discretion to determine the apportionment of the civil indemnity which the principal, accomplice and accessory are respectively liable for, without guidelines with respect to the basis of the allotment. Article 109 of the Revised Penal Code provides that "[i]f there are two or more persons civilly liable for a felony, the courts shall determine the amount for which each must respond." Notwithstanding the determination of the respective liability of the principals, accomplices and accessories within their respective class, they shall also be subsidiarily liable for the amount of civil liability adjudged in the other classes. Article 110 of the Revised Penal Code provides that "[t]he principals, accomplices, and accessories, each within their respective class, shall be liable severally (in solidum) among themselves for their quotas, and subsidiarily for those of the other persons liable." We must stress, however, that the courts discretion should not be untrammelled and must be guided by the principle behind differing liabilities for persons with varying roles in the commission of the crime. The person with greater participation in the commission of the crime should have a greater share in the civil liability than those who played a minor role in the crime or those who had no participation in the crime but merely profited from its effects. Each principal should shoulder a greater share in the total amount of indemnity and damages than every accomplice, and each accomplice should also be liable for a greater amount as against every accessory. Care should also be taken in considering the number of principals versus that of accomplices and accessories. If for instance, there are four principals and only one accomplice and the total of the civil indemnity and damages is P6,000.00, the court cannot assign two-thirds (2/3) of the indemnity and damages to the principals and one-third (1/3) to the accomplice. Even though the principals, as a class, have a greater share in the liability as against the accomplice-since one-third (1/3) of P6,000.00 is P2,000.00, while two-thirds (2/3) of P6,000.00 is P4,000.00-- when the civil liability of every person is computed, the share of the accomplice ends up to be greater than that of each principal. This is so because the two-thirds (2/3) share of the principalsor P4,000.00is still divided among all the four principals, and thus every principal is liable for only P1,000.00.
HIPOS SR. V. BAY

Facts: On 15 December 2003, two Informations for the crime of rape and one Information for the crime of acts of lasciviousness were filed against petitioners Darryl Hipos, Jaycee Corsio, Arthur Villaruel and two others before the RTC of Quezon City, acting as a Family Court, presided by respondent Judge Bay.

76

On 23 February 2004, private complainants AAA1 and BBB filed a Motion for Reinvestigation asking Judge Bay to order the City Prosecutor of Quezon City to study if the proper Informations had been filed against petitioners and their co-accused. Judge Bay granted the Motion and ordered a reinvestigation of the cases. On 19 May 2004, petitioners filed their Joint Memorandum to Dismiss the Case[s] before the City Prosecutor. They claimed that there was no probable cause to hold them liable for the crimes charged. On 10 August 2004, the Office of the City Prosecutor issued a Resolution on the reinvestigation affirming the Informations filed against petitioners and their co-accused The Resolution was signed by Assistant City Prosecutor Raniel S. Cruz and approved by City Prosecutor Claro A. Arellano. On 3 March 2006, 2nd Assistant City Prosecutor Lamberto C. de Vera, treating the Joint Memorandum to Dismiss the Case as an appeal of the 10 August 2004 Resolution, reversed the Resolution dated 10 August 2004, holding that there was lack of probable cause. On the same date, the City Prosecutor filed a Motion to Withdraw Informations before Judge Bay. On 2 October 2006, Judge Bay denied the Motion to Withdraw Informations in an Order of even date. Said order states: WHEREFORE, finding no probable cause against the herein accused for the crimes of rapes and acts of lasciviousness, the motion to withdraw informations is DENIED. Without moving for a reconsideration of the above assailed Order, petitioners filed the present Petition for Mandamus, to compel the trial court to accept the Prosecutors Motion to Withdraw information. Issue: WON the fallo of the Order stating that there is no probable cause to hold the respondents for trial is sufficient to compel the trial court to grant the Motion to Withdraw information Ruling: NO. After a careful study of the sworn statements of the complainants and the resolution dated March 3, 2006 of 2nd Assistant City Prosecutor Lamberto C. de Vera, the Court finds that there was probable cause against the herein accused. As can be seen, the body of the assailed Order not only plainly stated that the court found probable cause against the petitioners, but likewise provided an adequate discussion of the reasons for such finding. Indeed, the general rule is that where there is a conflict between the dispositive portion or the fallo and the body of the decision, the fallo controls. However, where the inevitable conclusion from the body of the decision is so clear as to show that there was a mistake in the dispositive portion, the body of the decision will prevail.
PEOPLE OF THE PHILIPPINES VS. PATERNO LORENZO G.R. NO. 184760 APRIL 23, 2010

FACTS: Two (2) Informations were filed against accused-appellant Paterno Lorenzo y Casas (Lorenzo) charging him with violating Sections 5 and 11, Article II of Republic Act No. 9165. And One Conrado Estanislao y Javier (Estanislao) was similarly charged in a different Information. Estanislao was accused of possessing illegal drugs in violation of the provisions of Section 11, Article II of Republic Act No. 9165. After entering their pleas of not guilty, the cases were consolidated, joint trial on the merits ensued. And the prosecution presented as its lone witness, Police Officer 1 (PO1) Noel P. Pineda, who was a 77

member of the buy-bust team. Interposing the twin defenses of denial and frame-up, accused-appellant Lorenzo and Estanislao stood before the witness stand and presented their version of the facts. Interposing the twin defenses of denial and frame-up, accused-appellant Lorenzo and Estanislao stood before the witness stand and presented their version of the facts. On 5 October 2005, the RTC rendered a Decision convicting Lorenzo for illegal possession and sale of dangerous drugs, but acquitting Estanislao. Invoking his innocence, Lorenzo appealed his conviction to the Court of Appeals, questioning the procedure followed by the police operatives in the seizure and custody of the evidence against him. The CA affirmed the judgment of conviction rendered by the RTC. Hence, this petition. Lorenzo questions his conviction on the basis of reasonable doubt. The defense anchors its claim on the failure of the prosecution to adopt the required procedure under Section 21, Article II, Republic Act No. 9165, on the custody and disposition of confiscated, seized, or surrendered dangerous drugs. ISSUE: Whether or not the prosecution discharged its burden of proving Lorenzos guilt beyond reasonable doubt for the crime charged. HELD: NO. The presumption of innocence of an accused in a criminal case is a basic constitutional principle, fleshed out by procedural rules which place on the prosecution the burden of proving that an accused is guilty of the offense charged by proof beyond reasonable doubt. Corollary thereto, conviction must rest on the strength of the prosecutions evidence and not on the weakness of the defense. In both illegal sale and illegal possession of prohibited drugs, conviction cannot be sustained if there is a persistent doubt on the identity of the drug. The identity of the prohibited drug must be established with moral certainty. Apart from showing that the elements of possession or sale are present, the fact that the substance illegally possessed and sold in the first place is the same substance offered in court as exhibit must likewise be established with the same degree of certitude as that needed to sustain a guilty verdict. PO1 Pineda testified that it was their confidential agent who purchased the shabu from accusedappellant and that he only retrieved it from said informant. He further testified that he marked the retrieved sachet of shabu together with the two other sachets of shabu that were allegedly seized from the accused, but it was not certain when and where the said marking was done nor who had specifically received and had custody of the specimens thereafter. The Court also observes that the prosecution did not present the poseur-buyer who had personal knowledge of the transaction. The lone prosecution witness was at least four meters away from where accused-appellant and the poseur-buyer were. From this distance, it was impossible for him to hear the conversation between accused-appellant and the poseur-buyer.
PEOPLE VS. BARON G.R. NO. 185209, JUNE 28, 2010

FACTS: Rene Baron y Tangarocan (appellant), Rey Villatima (Villatima), and alias "Dedong" Bargo (Bargo) was charged with the special complex crime of robbery with homicide committed against Juanito Berallo (Berallo). 78

Only the appellant was arrested, while Villatima and Bargo remained at-large. Appellant entered a plea of "not guilty" and denied any participation in the crime. RTC rendered a Decision finding the appellant guilty beyond reasonable doubt of the complex crime of robbery with homicide. Before the appellate court, appellant alleged that the trial court erred in finding him guilty as charged and in not appreciating in his favor the exempting circumstance of irresistible force and/or uncontrollable fear of an equal or greater injury. However, the same was disregarded by the CA holding that all the requisites for said circumstances were lacking. The appellate court found that the alleged threat, if at all, was not real or imminent. Appellant had every opportunity to escape but did not take advantage of the same. ISSUES: 1. Whether or not the circumstantial evidence is sufficient to render a judgment of conviction against for the commission of the special complex crime of robbery with homicide 2. Whether or not the RTC gravely erred in failing to appreciate the exempting circumstances of irresistable force and/or force and/or uncontrollable fear of an equal or greater injury.

HELD: 1. YES. Although, there is no direct evidence proving that the appellant conspired and participated in committing the crime. However, his complicity may be proved by circumstantial evidence, which consists of proof of collateral facts and circumstances from which the existence of the main fact may be inferred according to reason and common experience. Circumstantial evidence is sufficient to sustain conviction if: (a) there is more than one circumstance; (b) the facts from which the inferences are derived have been established; (c) the combination of all circumstances is such as to warrant a finding of guilt beyond reasonable doubt. A judgment of conviction based on circumstantial evidence can be sustained when the circumstances proved form an unbroken chain that results to a fair and reasonable conclusion pointing to the accused, to the exclusion of all others, as the perpetrator. In this case, circumstantial evidence is sufficient to produce a conviction that the appellant conspired with his co-accused in committing the crime of robbery with homicide. The concerted manner in which the appellant and his companions perpetrated the crime showed beyond reasonable doubt the presence of conspiracy. 2. NO. The appellants attempt to evade criminal liability by insisting that he acted under the impulse of an uncontrollable fear of an equal or greater injury fails to impress. His claim that he acted under the impulse of uncontrollable fear of an equal or greater injury could not be sustained because there was no genuine, imminent, and reasonable threat, preventing his escape that compelled him to take part in the commission of the offense charged. To avail of this exempting circumstance, the evidence must establish: (1) the existence of an uncontrollable fear; (2) that the fear must be real and imminent; and (3) the fear of an injury is greater than or at least equal to that committed. A threat of future injury is insufficient. The compulsion must be of such a character as to leave no opportunity for the accused to escape. In this case, the appellant had every opportunity to escape but did not take advantage of the same. Instead, he waited inside the tricycle as if he was one of the malefactors
ABELLANA VS. PEOPLE G.R. NO. 174654, AUGUST 17, 2011

79

FACTS: In 1985, petitioner extended a loan to private respondents spouses Alonto, secured by a Deed of Real Estate Mortgage over Lot Nos. 6471 and 6472 located in Cebu City. Subsequently, or in 1987, petitioner prepared a Deed of Absolute Sale conveying said lots to him, which was signed by spouses Alonto in Manila. However, it was notarized in Cebu City allegedly without the spouses Alonto appearing before the notary public. Thereafter, petitioner caused the transfer of the titles to his name and sold the lots to third persons. On August 12, 1999, an Information was filed charging petitioner with Estafa through Falsification of Public Document. During arraignment, petitioner entered a plea of "not guilty". RTC concluded that petitioner can only be held guilty of Falsification of a Public Document by a private individual under Article 172(1) in relation to Article 171(2) of the RPC and not estafa through falsification of public document as charged in the Information. On appeal, CA held that petitioner who was charged with and arraigned for estafa through falsification of public document under Article 171(1) of the RPC could not be convicted of Falsification of Public Document by a Private Individual under Article 172(1) in relation to Article 171(2), as the falsification committed in Article 171(1) requires the counterfeiting of any handwriting, signature or rubric while the falsification in Article 171(2) occurs when the offender caused it to appear in a document that a person participated in an act or proceeding when in fact such person did not so participate. Thus, the CA opined that the conviction of the petitioner for an offense not alleged in the Information or one not necessarily included in the offense charged violated his constitutional right to be informed of the nature and cause of the accusation against him. Nonetheless, the CA affirmed the trial court's finding with respect to petitioner's civil liability. ISSUE: Whether or not petitioner Abellana could still be held civilly liable notwithstanding his acquittal. HELD: NO. It is an established rule in criminal procedure that a judgment of acquittal shall state whether the evidence of the prosecution absolutely failed to prove the guilt of the accused or merely failed to prove his guilt beyond reasonable doubt. In either case, the judgment shall determine if the act or omission from which the civil liability might arise did not exist. When the exoneration is merely due to the failure to prove the guilt of the accused beyond reasonable doubt, the court should award the civil liability in favor of the offended party in the same criminal action. In other words, the "extinction of the penal action does not carry with it the extinction of civil liability unless the extinction proceeds from a declaration in a final judgment that the fact from which the civil [liability] might arise did not exist." However, based on the records of the case, the acts allegedly committed by the petitioner did not cause any damage to spouses Alonto. Hence, there is therefore absolutely no basis for the trial court and the CA to hold petitioner civilly liable to restore ownership and possession of the subject properties to the spouses Alonto or to pay them P1,103,000.00 representing the value of the properties and to pay them nominal damages, exemplary damages, attorney's fees and litigation expenses.
PEOPLE VS. ASIS G.R. NO. 142531 OCTOBER 15, 2002

FACTS: 80

For automatic review before this Court is the March 8, 2000 Decision of the RTC- Manila in Criminal Case No. 98-163090, finding Danilo Asis y Fonperada and Gilbert Formento y Saricon guilty beyond reasonable doubt of robbery with homicide aggravated by abuse of confidence, superior strength and treachery. When arraigned on July 9, 1998, both appellants pleaded not guilty. Found to be deaf-mutes, they were assisted, not only by a counsel de oficio, but also by an interpreter from the Calvary Baptist Church. After due trial, appellants were found guilty and sentenced to death. The RTC held that the "crime charged and proved is robbery with homicide under Article 294, No. 1 of the Revised Penal Code." It ruled that "although no witnesses to the actual killing and robbery were presented, the circumstantial evidence including the recovery of bloodstained clothing from both accused definitely proved that the two (2) x x x committed the crime." Finally, the RTC also appreciated the aggravating circumstances of abuse of confidence, superior strength and treachery and thus sentenced both appellants to the supreme penalty of death. Hence, this automatic review. ISSUES: 1. Whether or not the trial court gravely erred in finding the accused-appellants guilty beyond reasonable doubt of the crime of robbery with homicide notwithstanding the insufficiency of the circumstantial evidence presented by the prosecution. 2. The trial court gravely erred in concluding that evident premeditation, treachery and conspiracy attended the killing of Roy Ching. 3. The trial court gravely erred in not considering the physical infirmities of the two accusedappellants who are deaf-mutes."16 HELD: YES. The prosecutions evidence does not prove the guilt of appellants beyond reasonable doubt; hence, their constitutional right to be presumed innocent remains and must be upheld. Circumstantial evidence that merely arouses suspicions or gives room for conjecture is not sufficient to convict. It must do more than just raise the possibility, or even the probability, of guilt. It must engender moral certainty. Otherwise, the constitutional presumption of innocence prevails, and the accused deserves acquittal.
ESTINO VS. PEOPLE G.R. NOS. 163957-58 APRIL 7, 2009

FACTS: For review before the Court under Rule 45 are the April 16, 2004 Decision and June 14, 2004 Resolution of the Sandiganbayan in the consolidated Criminal Case Nos. 26192 and 26193 entitled People of the Philippines v. Munib S. Estino and Ernesto G. Pescadera. In G.R. Nos. 163957-58, petitioners Munib S. Estino and Ernesto G. Pescadera appeal their conviction of violation of Section 3(e), Republic Act No. (RA) 3019 or the Anti-Graft and Corrupt Practices Act for failure to pay the Representation and Transportation Allowance (RATA) of the provincial government employees of Sulu. In G.R. Nos. 16400911, petitioner Pescadera alone appeals his conviction of malversation of public funds under Article 217 81

of the Revised Penal Code for failure to remit the Government Service Insurance System (GSIS) contributions of the provincial government employees amounting to PhP 4,820,365.30. In these consolidated appeals, petitioners pray for their acquittal. ISSUE: Whether or not a new trial is proper in the determination the guilt of the petitioners in non-payment of RATA in violation of Sec 3(e) of RA 3019. HELD: YES. Petitioners defense is anchored on their payment of RATA, and for this purpose, they submitted documents which allegedly show that they paid the RATA under the 1998 reenacted budget. They also claim that the COA Report did not sufficiently prove that they did not pay the RATA because the alleged disbursement vouchers, which were supposed to be annexed to the COA Report as proof of nonpayment of RATA, were not submitted with said report. Rule 121 of the Rules of Court allows the conduct of a new trial before a judgment of conviction becomes final when new and material evidence has been discovered which the accused could not with reasonable diligence have discovered and produced at the trial and which if introduced and admitted would probably change the judgment.20 Although the documents offered by petitioners are strictly not newly discovered, it appears to us that petitioners were mistaken in their belief that its production during trial was unnecessary.
BRIONES V. PEOPLE

FACTS: This is a Rule 45 petition for review on certiorari of the decision and resolution of the CA finding petitioner Rommel C. Briones (Briones) guilty of the crime of robbery. On January 6, 1998, at around 11:00 p.m., while S/G Molina and S/G George Gual (S/G Gual) were manning the northwest gate of BF Homes Northwest, Paraaque noticed Romulo Bersamina, a homeowner, being mauled by four (4) individuals, two (2) of whom were later identified as Briones and his brother, Vicente Briones (Vicente), who were both residents of BF Homes. S/G Molina and S/G Gual approached the group to stop the mauling; it was at this point that S/G Molina lost his firearm to Briones. The police arrested Briones after conducting an investigation. However, Briones denied any participation in the mauling and the firearm grabbing, and claimed that he was in his house when the incident happened. On January 8, 1998, a criminal information was filed against Briones before the RTC, Paraaque City, for robbery, where the accused pleaded not guilty to the charge. RTC found Briones guilty only of simple theft as the elements of violence and intimidation the attendant circumstances that must be present in the crime of robbery were not duly proven. On appeal, CA found Briones guilty of robbery under Article 293, in relation to paragraph 5 of Article 294, of the RPC, and not of theft. The CA ruled that force and intimidation attended the taking of S/G Molinas firearm, as Briones approached S/G Molina with the intent of taking his firearm away. Briones thereafter filed an Omnibus Motion for Reconsideration, Motion for New Trial and Motion to Dismiss, and Supplemental Omnibus Motion for Reconsideration, Motion for New Trial and Motion to Dismiss (collectively, Omnibus Motion) with the CA where he confessed his physical presence and

82

participation on the alleged robbery of the firearm, but claimed that he was merely protecting his brother, Vicente, when he took the firearm. The CA denied the Omnibus Motion; hence, this petition. ISSUES: 1. Whether or not there are factual and legal bases to support his conviction of the crime of robbery 2. Whether or not a new trial is justified under the circumstances.

HELD: 1. NO. The distinguishing element between the crimes of robbery and theft is the use of violence or intimidation as a means of taking the property belonging to another; the element is present in the crime of robbery and absent in the crime of theft. Hence, only the crime of theft was committed in the case as the witness - S/G Guals testimony does not show that violence or intimidation attended the taking of the firearm; S/G Gual only testified that Briones merely grabbed the firearm and ran away with it. Thus, Briones can only be convicted for the crime of theft for taking S/G Molinas firearm without his consent. Theft is produced the moment there is deprivation of personal property due to its taking with intent to gain. 2. NO. For new trial to be granted on the ground of newly discovered evidence, the concurrence of the following conditions must obtain: (a) the evidence must have been discovered after trial; (b) the evidence could not have been discovered at the trial even with the exercise of reasonable diligence; (c) the evidence is material, not merely cumulative, corroborative, or impeaching; and (d) the evidence must affect the merits of the case and produce a different result if admitted. In this case, although the firearm surfaced after the trial, the other conditions were not established. Evidence, to be considered newly discovered, must be one that could not, by the exercise of due diligence, have been discovered before the trial in the court below. The determinative test is the presence of due or reasonable diligence to locate the thing to be used as evidence in the trial. Under the circumstances, Briones failed to show that he had exerted reasonable diligence to locate the firearm; his allegation in his Omnibus Motion that he told his brothers and sisters to search for the firearm, which yielded negative results, is purely self-serving. He also now admits having taken the firearm and having immediately disposed of it at a nearby house, adjacent to the place of the incident. Hence, even before the case went to court, he already knew the location of the subject firearm, but did not do anything; he did not even declare this knowledge at the trial below. In any case, recovery of the firearm may not be considered material evidence that will affect the outcome of the case; the recovery of the subject firearm does not negate the commission of the crime charged.

SALUDAGA VS. GENIO G.R. NO. 184537

APRIL 23, 2010

Facts: Quintin Saludaga, municipal mayor of Lavesares, Northern Samar and SPO2 Fiel Genio were charged in the Sandiganbayan of violation of Sec. 3(e) of the Anti Graft and Corrupt Practices Act (R.A.3019) by 83

causing undue injury to the government. The Sandiganbayan dismissed the information "for failure of the prosecution to allege and prove the amount of actual damages caused the government, an essential element of the crime charged. The Ombudsman directed the Office of the Special Prosecutor to study the possibility of having the information amended and re-filed with the Sandiganbayan. The OSP re-filed the Information. Now, charging the petitioners for violation of Section 3(e) of R.A. No. 3019, by giving unwarranted benefit to a private person, to the prejudice of the government. Aside from arguing that the second Information constituted substituted Information and contained substantial amendments, the petitioners also highlighted that newly discovered evidence mandates due re-examination of the finding of prima facie cause to file the case which necessitates a new preliminary investigation. Issue: Whether or not the presence of newly discovered evidence necessitates a new preliminary investigation. Held: NO. Under Section 2 of Rule 121, the requisites for newly discovered evidence are: (a) the evidence was discovered after trial; (b) such evidence could not have been discovered and produced at the trial with reasonable diligence; and (c) that it is material, not merely cumulative, corroborative or impeaching, and is of such weight that, if admitted, will probably change the judgment. The piece of evidence sought to be considered by the Petitioners cannot be considered as newly found evidence because it was already in existence prior to the re-filing of the case. In fact, such sworn affidavit was among the documents considered during the preliminary investigation. It was the sole annexed document to petitioners Supplement to Motion for Reinvestigation, offered to dispute the charge that no public bidding was conducted prior to the execution of the subject project.
LUMANOG VS. PEOPLE G.R. NO. 182555 FEBRUARY 8, 2011

Facts: Lenido Lumanog and Augusto Santos, Cesar Fortuna and Rameses de Jesus were found guilty by the Supreme Court for the murder of Col. Rolando N. Abadilla. They filed separate motions for reconsideration. They seek to be admitted the affidavit of Orencio G. Jurado, Jr., one of the police officers initially assigned to investigate the case who claims that he was prevented to testify to the court that the movants are not the same persons arrested by them. The movants argued that the said belated statement would certainly cast doubt on the procedures undertaken by the police authorities in the apprehension of the likely perpetrators. Issue: Whether or not the affidavit of Jurado be admitted as newly discovered evidence. Held:

84

No. Fortuna seeks the introduction of additional evidence to support the defense argument that there was no positive identification of Abadillas killers. To justify a new trial or setting aside of the judgment of conviction on the basis of such evidence, it must be shown that the evidence was "newly discovered" pursuant to Section 2, Rule 121 of the Revised Rules of Criminal Procedure, as amended. Evidence, to be considered newly discovered, must be one that could not, by the exercise of due diligence, have been discovered before the trial in the court below. The movant failed to show that the defense exerted efforts during the trial to secure testimonies from police officers like Jurado, or other persons involved in the investigation, who questioned or objected to the apprehension of the accused in this case. Hence, the belatedly executed affidavit of Jurado does not qualify as newly discovered evidence that will justify re-opening of the trial and/or vacating the judgment. In any case, we have ruled that whatever flaw that may have initially attended the out-ofcourt identification of the accused, the same was cured when all the accused-appellants were positively identified by the prosecution eyewitness during the trial.
PAYUMO VS. SANDIGANBAYAN G.R. NO. 151911, JULY 25, 2011

Facts: A composite team of Philippine Constabulary and Integrated National Police units allegedly fired at a group of civilians instantly killing one civilian and wounding seven others, including Edgar Payumo. The accused pleaded not guilty to the offense charged. During the trial, the accused interposed the defenses of lawful performance of duty, self-defense, mistake of fact, and alibi. They insisted that the incident was a result of a military operation, and not an ambush as claimed by the prosecution. The Fifth Division promulgated its judgment dated November 27, 1998, convicting the accused of the crime of Murder with Multiple Attempted Murder. The accused filed their Supplemental Omnibus Motion to Set Aside Judgment and for New Trial because there was serious irregularity during the trial due to the erroneous admission of the testimonies of the witnesses of the petitioners, such should be taken anew and to afford the accused the opportunity to present in evidence the records of the Judge Advocate General Office(JAGO) relative to the shooting as to whether it was an ambush or the result of a military operation. The omnibus motion was granted. Ascribing grave abuse of discretion to the Sandiganbayan amounting to lack or excess of jurisdiction for nullifying the order of conviction and granting new trial, Edgar Payumo and et. al, filed a petition for certiorari and mandamus with prayer for the issuance of a temporary restraining order and/or injunction to enjoin the Sandiganbayan from proceeding with the scheduled hearings for a second new trial. Issue: Whether or not the Sandiganbayan acted in excess of its jurisdiction when it granted a new trial in favor of the accused. Held: Yes. Rule 121, Section 2 of the 2000 Rules on Criminal Procedure enumerates the grounds for a new trial, to wit: Sec. 2. Grounds for a new trial. The court shall grant a new trial on any of the following grounds:

85

That errors of law or irregularities prejudicial to the substantial rights of the accused have been commited during the trial; (b) That new and material evidence has been discovered which the accused could not with reasonable diligence have discovered and produced at the trial and which if introduced and admitted would probably change the judgment. It must be emphasized that an erroneous admission or rejection of evidence by the trial court is not a ground for a new trial or reversal of the decision if there are other independent evidence to sustain the decision, or if the rejected evidence, if it had been admitted; would not have changed the decision. The records of the JAGO relative to shooting incident do not meet the criteria for newly discovered evidence that would merit a new trial. A motion for new trial based on newly-discovered evidence may be granted only if the following requisites are met: 1. that the evidence was discovered after trial; 2. that said evidence could not have been discovered and produced at the trial even with the exercise of reasonable diligence; 3. that it is material, not merely cumulative, corroborative or impeaching; and 4. that the evidence is of such weight that, if admitted, would probably change the judgment. It is essential that the offering party exercised reasonable diligence in seeking to locate the evidence before or during trial but nonetheless failed to secure it. In this case, however, such records could have been easily obtained by the accused and could have been presented during the trial with the exercise of reasonable diligence.
PEOPLE V. MORALES G.R. NO. 172873 MARCH 19, 2010

Facts: Roldan Morales was charged in two separate Informations before the RTC with possession and sale of methylamphetamine hydrochloride (shabu). The trial court and the Court of Apelas found Morales guilty beyond reasonable doubt of illegal possession and illegal sale of dangerous drugs. Issue: What is the nature of appeal in criminal cases? Held: Appeal in criminal cases possess a unique nature. The appeal throws the whole case open for review and it is the duty of the appellate court to correct, cite and appreciate errors in the appealed judgment whether they are assigned or unassigned. On the basis of such review, we find the present appeal meritorious. Prevailing jurisprudence uniformly hold that the trial courts findings of fact, especially when affirmed by the CA, are, as a general rule, entitled to great weight and will not be disturbed on appeal. However, this rule admits of exceptions and does not apply where facts of weight and substance with direct and material bearing on the final outcome of the case have been overlooked, misapprehended or misapplied. After due consideration of the records of this case, evidence presented and relevant law and jurisprudence, we hold that this case falls under the exception. 86

The identity of the corpus delicti in this case was not proven beyond reasonable doubt. There was likewise a break in the chain of custody which proves fatal to the prosecutions case. Thus, since the prosecution has failed to establish the element of corpus delicti with the prescribed degree of proof required for successful prosecution of both possession and sale of prohibited drugs, we resolve to ACQUIT Roldan Morales.
QUIDET VS. PEOPLE G.R. NO. 170289 APRIL 8, 2010

Facts: Jimmy, Andrew, and two others visited a friend in Looc, Misamis Oriental. Along the way, they saw Taban, together with Quidet and Tubo, come out of the house of one Tomas Osep. Taban suddenly stabbed Andrew on the chest with a knife. Jimmy tried to pacify Andrew and Taban but the latter stabbed him in the abdomen. Taban immediately fled. After Jimmy fell down, Tubo threw a drinking glass at Andrews face while Quidet boxed Andrews jaw. Tubo stabbed Jimmy who was then lying face down on the ground twice on the back with an ice pick after which he fled. Quidet boxed Jimmys mouth. , Jimmy was declared dead by the attending physician while Andrew sustained minor injuries. The RTC found Quidet and Tubo guilty of homicide for the death of Jimmy. Quidet, Tubo and Taban were also pronounced guilty of frustrated homicide for the misfortune of Andrew by the same court. From this judgment, only petitioner appealed to the Court of Appeals. The Court of appeals dismissed the appeal but reduced the penalty imposed over Taban and Tubo because the crime committed against Andrew was attempted and not frustrated homicide. Issue: Whether or not Taban and Tubo may be benifited from the favorable result of an appeal even if it is only one of their co-accused filed it. Held: The rule is that an appeal taken by one or more of several accused shall not affect those who did not appeal except insofar as the judgment of the appellate court is favorable and applicable to the latter.
BALABALA V. PEOPLE GR NO.169519; JULY 17, 2009

FACTS: The Provincial Auditors Office conducted an examination of the cash and accounts of the accountable officers of the Municipality of Guindulman, Bohol. It was discovered that there were cash shortage, unaccounted cash tickets and an unrecorded check payable to Balaba. Thus, an Information for Malversation of Public Funds was filed against Balaba with the trial court. After trial, the trial court found Balaba guilty. Balaba hence filed a Notice of Appeal where he indicated that he will file his appeal with the Court of Appeals. Thereafter, he filed his appellants brief with the CA. The CA dismissed his appeal declaring that it has no jurisdiction over his appeal. Balaba thus filed a Motion for Reconsideration and asked that he be allowed to pursue his appeal with the Sandiganbayan. The CA denied his Motion for Reconsideration, thus, he filed his present petition with the Supreme Court. ISSUE: Whether or not the CA erred in dismissing his appeal instead of certifying the case to the Sandiganbayan. 87

HELD: Upon Balabas conviction by the trial court, his remedy should have been an appeal to the Sandiganbayan. Paragraph 3, Section 4(c) of Republic Act No. 8249 (RA 8249), which further defined the jurisdiction of the Sandiganbayan, reads: The Sandiganbayan shall exercise exclusive appellate jurisdiction over final judgments, resolutions or orders of the regional trial courts whether in the exercise of their own original jurisdiction or of their appellate jurisdiction as herein provided. An error in designating the appellate court is not fatal to the appeal. However, the correction in designating the proper appellate court should be made within the 15-day period to appeal. Once made within the said period, the designation of the correct appellate court may be allowed even if the records of the case are forwarded to the Court of Appeals. Otherwise, the second paragraph of Section 2, Rule 50 of the Rules of court would apply which reads: "An appeal erroneously taken to the Court of Appeals shall not be transferred to the appropriate court but shall be dismissed outright." In this case, Balaba sought the correction of the error in filing the appeal only after the expiration of the period to appeal. Therefore, the Court of Appeals did not commit any error when it dismissed Balabas appeal because of lack of jurisdiction.
GUASCH V. DELA CRUZ GR NO.176015; JUNE 16, 2009

FACTS: Respondent Arnaldo dela Cruz (respondent) alleged in his Complaint-Affidavit against petitioner Mercedita T. Guasch (petitioner) that petitioner transacted business with him by exchanging cash for checks of small amount without interest; that Petitioner requested him to exchange her check with cash of P3,300,000.00; that petitioner was able to convince him to give her P3,300,000.00 in cash in exchange for her Insular Savings Bank Check upon her assurance that she will have the funds and bank deposit to cover the said check by January 2000; that on the date of maturity and upon presentment the check was dishonored. Petitioner was charged with Estafa before the Regional Trial Court of Manila (RTC). After trial, the RTC granted the demurrer to evidence and dismissed the case against petitioner. Respondent filed a Motion to Amend Order to include a finding of civil liability on the petitioner. Respondent also filed a Petition for Certiorari with the Court of Appeals to set aside the order of demurrer. The Motion to Amend Order was denied thus, respondent filed a supplemental petition for certiorari with the CA. The CA affirmed the order of demurrer to evidence but ruled that the denial of the Motion to Amend Order was rendered with grave abuse of discretion. ISSUE: Whether or not the CA erred in holding that the trial court committed grave abuse of discretion when it denied respondents Motion to Amend. HELD: We affirm the rulings of the CA.

88

The statutory requirement that when no motion for reconsideration is filed within the reglementary period, the decision attains finality and becomes executory in due course must be strictly enforced, first, to avoid delay in the administration of justice and, second, to put an end to judicial controversies, at the risk of occasional errors. However, in exceptional cases, substantial justice and equity considerations warrant the giving of due course to an appeal by suspending the enforcement of statutory and mandatory rules of procedure. Certain elements are considered for the appeal to be given due course, such as: (1) the existence of special or compelling circumstances, (2) the merits of the case, (3) a cause not entirely attributable to the fault or negligence of the party favored by the suspension of the rules, (4) lack of any showing that the review sought is merely frivolous and dilatory, and (5) the other party will not be unduly prejudiced thereby. Several of these elements obtain in the case at bar. First, records show that petitioner admits her civil obligation to respondent for a total of P3,300,000.00. Second, it cannot be said that petitioner will be unduly prejudiced if respondents Motion to Amend for the sole purpose of including the civil liability of petitioner in the order of acquittal shall be allowed. Foremost, petitioner admits her civil obligation to respondent.
PEOPLE V. OLIVO, DANDA AND REYES GR NO.177768; JULY 27, 2009

FACTS: Charmen Olivo (Olivo), Nelson Danda (Danda), and Joey Zafra (Zafra) were charged for the crime of robbery with homicide of Mariano Constantino. After trial, the trial court found Olivo, Danda and Zafra, guilty. Only Olivo and Danda appealed to the Court of Appeals which affirmed the ruling of the trial court, thus, the instant appeal with the Supreme Court. The Supreme Court acquitted Olivo and Danda for the failure of the lone eyewitness of the prosecution to identify the accused-appellants. ISSUE: Whether or not the acquittal of Olivo and Danda would benift Zafra who did not join the former in their appeal.

HELD: The present rule is that an appeal taken by one or more several accused shall not affect those who did not appeal, except insofar as the judgment of the appellate court is favorable and applicable to the latter. Our pronouncements here with respect to the insufficiency of the prosecution evidence to convict appellants beyond reasonable doubt are definitely favorable and applicable to accused Joey Zafra. He should not therefore be treated as the odd man out and should benefit from the acquittal of his co-accused. In fact, under similar conditions and on the same ratiocination, Section 11(a), Rule 122 of the Rules of Court has justified the extension of our judgment of acquittal to the co-accused who failed to appeal from the judgment of the trial court which we subsequently reversed.

ISSUE: Whether or not the CA erred in holding that the trial court committed grave abuse of discretion when it denied respondents Motion to Amend.

89

HELD: We affirm the rulings of the CA. The statutory requirement that when no motion for reconsideration is filed within the reglementary period, the decision attains finality and becomes executory in due course must be strictly enforced, first, to avoid delay in the administration of justice and, second, to put an end to judicial controversies, at the risk of occasional errors. However, in exceptional cases, substantial justice and equity considerations warrant the giving of due course to an appeal by suspending the enforcement of statutory and mandatory rules of procedure. Certain elements are considered for the appeal to be given due course, such as: (1) the existence of special or compelling circumstances, (2) the merits of the case, (3) a cause not entirely attributable to the fault or negligence of the party favored by the suspension of the rules, (4) lack of any showing that the review sought is merely frivolous and dilatory, and (5) the other party will not be unduly prejudiced thereby. Several of these elements obtain in the case at bar. First, records show that petitioner admits her civil obligation to respondent for a total of P3,300,000.00. Second, it cannot be said that petitioner will be unduly prejudiced if respondents Motion to Amend for the sole purpose of including the civil liability of petitioner in the order of acquittal shall be allowed. Foremost, petitioner admits her civil obligation to respondent.
PEOPLE VS. TARUC GR NO. 185202; FEBRUARY 18, 2009

FACTS: Francisco Taruc (Taruc) was charged before the RTC of Bataan with the crime of murder in connection with the death of Emelito Sualog. Taruc pleaded not guilty. After trial on the merits, the RTC of Bataan found Taruc guilty for murder and sentenced him to suffer the death penalty. The case was brought to the Court of Appeals for automatic review in accordance with A.M. No. 00-5-03-SC. Thereafter, the CA was informed that Taruc escaped from prison. Then, the CA affirmed the Decision of the RTC. Taruc appealed the Decision of the CA to the Supreme Court. ISSUE: Whether or not Taruc has lost his right to appeal his conviction considering that he escaped from jail and eluded arrest. HELD: Once an accused escapes from prison or confinement or jumps bail or flees to a foreign country, he loses his standing in court and unless he surrenders or submits to the jurisdiction of the court he is deemed to have waived any right to seek relief from the court. Although Rule 124, Section 8 particularly applies to the Court of Appeals, it has been extended to the Supreme Court by Rule 125, Section 1 of the Revised Rules of Criminal Procedure. Notwithstanding, accused-appellant did not preclude the Court of Appeals from exercising its review jurisdiction, considering that what was involved was capital punishment. Automatic review being mandatory, it is not only a power of the court but a duty to review all death penalty cases.

90

But accused-appellant impliedly waived his right to appeal to the Supreme Court. There are certain fundamental rights which cannot be waived even by the accused himself but the right of appeal is not one of them. The accused cannot be accorded the right to appeal unless he voluntarily submits to the jurisdiction of the court or is otherwise arrested within 15 days from notice of the judgment against him. While at large, he cannot seek relief from the court, as he is deemed to have waived the appeal. Thus, having escaped from prison or confinement, he loses his standing in court; and unless he surrenders or submits to its jurisdiction, he is deemed to have waived any right to seek relief from the court. Thus, appeal is dismissed.
TIU VS. COURT OF APPEALS AND EDGARDO POSTANES GR NO. 16273; APRIL 21, 2009

FACTS: Respondent Edgardo Postanes (Postanes) charged Remigio Pasion (Pasion) for slight physical injuries. On the other hand, petitioner David Tiu (Tiu) filed a criminal charge for grave threats against Postanes. The Slight Physical Injuries was docketed as Criminal Case No. 96-412 while the Grave Threats was docketed as Criminal Case No. 96-413. Both were filed with the Metropolitan Trial Court (MeTC) of Pasay City. Postanes together with his witnesses testified as to his innocence in Criminal Case No. 96413. In Criminal Case No. 96-413, where he stood as the accused, Postanes adopted his testimony and his witnesses testimonies which were formally offered and admitted in Criminal Case No. 96-412. In its Decision, the MeTC dismissed the both criminal cases for insufficiency of evidence. Tiu then filed a Motion for Reconsideration of the Decision of the MeTC which was denied, thus, he filed a Petition for Certiorari with the RTC. The judgment of acquittal of Postanes in Criminal Case No. 96-413 was declared void by the RTC. Postanes moved for reconsideration of the decision and when he was denied, he filed Petition for Review with the Court of Appeals. The CA reversed the RTC decision and affirmed the dismissal of Criminal Case No. No. 96-413. Thus, Tui filed a Petition for Review assailing the Resolution of the CA. ISSUE: Whether or not there was double jeopardy when Tui filed a Petition for Review with the CA assailing the MeTC decision acquitting Postanes. HELD: The petition is defective since it was not filed by the Solicitor General. It was filed by Tiu, the private complainant through his counsel. Settled is the rule that only the Solicitor General may bring or defend actions on behalf of the Republic of the Philippines, or represent the People or State in criminal proceedings before this Court and the Court of Appeals. Tiu, the offended party is without legal personality to appeal the decision of the Court of Appeals before this Court. Further, the elements of double jeopardy were all present: (1) the complaint or information was sufficient in form and substance to sustain a conviction; (2) the court had jurisdiction; (3) the accused had been arraigned and had pleaded; and (4) the accused was convicted or acquitted or the case was dismissed without his express consent. The Information against Postanes was sufficient in form and substance to sustain a conviction; the MeTC had jurisdiction; Postanes was arraigned and entered a nonguilty plea; and the MeTC dismissed Criminal Case on the ground of insufficiency of evidence amounting to an acquittal from which no appeal can be had. The Petition is denied.

91

MICLAT, JR. VS. PEOPLE G.R. NO. 176077, 31 AUGUST 2011

Facts: On November 2002, P/Insp. Valencia called upon his subordinates after the receiving an INFOREP Memo from Camp Crame relative to the drug-trading activities being undertaken in Caloocan City involving Abe Miclat, Wily alias "Bokbok" and one Mic or Jojo. A surveillance team was formed headed by SPO4 Palting and is composed of five more operatives from the Drug Enforcement Unit, namely: PO3 Pagsolingan, PO2 Modina, PO2 De Ocampo, and herein witness PO3 Antonio. When the group of SPO4 Palting arrived at the site they were at once led by their informant to the house of one "Abe." PO3 Antonio then positioned himself at the perimeter of the house, while the rest of the members of the group deployed themselves nearby. Thru a small opening in the curtain-covered window, PO3 Antonio peeped inside and there at a distance of 1 meters, he saw "Abe" arranging several pieces of small plastic sachets which he believed to be containing shabu. Slowly, said operative inched his way in by gently pushing the door as well as the plywood covering the same. Upon gaining entrance, PO3 Antonio forthwith introduced himself as a police officer while "Abe," on the other hand, after being informed of such authority, voluntarily handed over to the former the four (4) pieces of small plastic sachets the latter was earlier sorting out. PO3 Antonio immediately placed the suspect under arrest and brought him and the four (4) pieces of plastic sachets containing white crystalline substance to their headquarters and turned them over to PO3 Fernando Moran for proper disposition. On 2004, the RTC rendered a Decision convicting Miclat Jr. of the crime of possession of a dangerous drugs. Aggrieved, petitioner sought recourse before the CA, who in turn affirmed in toto the decision of the RTC.In affirming the RTC, the CA ratiocinated that contrary to the contention of the Miclat Jr., the evidence presented by the prosecution were all admissible against him. Moreover, it was established that he was informed of his constitutional rights at the time of his arrest. Miclat Jr. raised the matter to the Supreme Court assailing the legality of his arrest and the subsequent seizure of the arresting officer of the suspected sachets of dangerous drugs from him. Petitioner insists that he was just watching television with his father and sister when police operatives suddenly barged into their home and arrested him for illegal possession of shabu. Moreover, being seen in the act of arranging several plastic sachets inside their house is not sufficient reason for the police authorities to enter his house without a valid warrant. In the same vein, peeping through a curtain-covered window cannot be contemplated as within the meaning of the plain view doctrine, rendering the warrantless arrest unlawful. Issue: Whether peeping through a window can be contemplated as within the meaning of the plain view doctrine, rendering the warrantless arrest lawful. Held: Yes. An exception to the right guaranteed against warrantless arrest is that of an arrest made during the commission of a crime. Such warrantless arrest is considered reasonable and valid under Section 5 (a), Rule 113 of the Revised Rules on Criminal Procedure, to wit: peace office of a private person may, without a warrant, arrest a person: (a) When, in his presence, the person to be arrested has committed, is actually committing, or is attempting to commit an offense; xxx For the exception in Section 5 (a), Rule 113 to operate, this Court has ruled that two (2) elements must be present: (1) the person to be arrested must execute an overt act indicating that he has just

92

committed, is actually committing, or is attempting to commit a crime; and (2) such overt act is done in the presence or within the view of the arresting officer. In the instant case, he was caught in flagrante delicto and the police authorities effectively made a valid warrantless arrest. Facts reveal that on the date of the arrest, agents of the Station Drug Enforcement Unit (SDEU) were conducting a surveillance operation in the area to verify the reported drug-related activities of several individuals, which included the Miclat Jr. During the operation, PO3 Antonio, through petitioners window, saw petitioner arranging several plastic sachets containing what appears to be shabu in the living room of their home. The plastic sachets and its suspicious contents were plainly exposed to the view of PO3 Antonio, who was only about one and one-half meters from where petitioner was seated. Upon gaining entrance, the operative introduced himself as a police officer. After which, Miclat Jr. voluntarily handed over to PO3 Antonio the small plastic sachets. Considering the circumstances immediately prior to and surrounding the arrest of the petitioner, petitioner was clearly arrested in flagrante delicto as he was then committing a crime, violation of the Dangerous Drugs Act, within the view of the arresting officer. As to the admissibility of the seized drugs in evidence, it too falls within the established exceptions. The right against warrantless searches and seizure, however, is subject to legal and judicial exceptions, and among them are warrantless search incidental to a lawful arrest and search of evidence in "plain view". The seizure made by PO3 Antonio of the four plastic sachets from the petitioner was not only incidental to a lawful arrest, but it also falls within the purview of the "plain view" doctrine. Objects falling in plain view of an officer who has a right to be in a position to have that view are subject to seizure even without a search warrant and may be introduced in evidence. The "plain view" doctrine applies when the following requisites concur: (a) the law enforcement officer in search of the evidence has a prior justification for an intrusion or is in a position from which he can view a particular area; (b) the discovery of evidence in plain view is inadvertent; (c) it is immediately apparent to the officer that the item he observes may be evidence of a crime, contraband or otherwise subject to seizure. The law enforcement officer must lawfully make an initial intrusion or properly be in a position from which he can particularly view the area. In the course of such lawful intrusion, he came inadvertently across a piece of evidence incriminating the accused. The object must be open to eye and hand and its discovery inadvertent. It is clear, therefore, that an object is in plain view if the object itself is plainly exposed to sight. Since Miclat Jr.s arrest is among the exceptions to the rule requiring a warrant before effecting an arrest and the evidence seized from the petitioner was the result of a warrantless search incidental to a lawful arrest, which incidentally was in plain view of the arresting officer, the results of the ensuing search and seizure were admissible in evidence to prove petitioners guilt of the offense charged. WHEREFORE, premises considered, the appeal is DENIED. The Decision dated October 13, 2006 of the Court of Appeals in CA-G.R. CR No. 28846 is AFFIRMED with MODIFICATION. Petitioner is sentenced to suffer the indeterminate sentence of twelve (12) years and one (1) day to fourteen (14) years and eight (8) months.
PEOPLE VS. MARIACOS G.R. NO. 188611, 16 JUNE 2010

Facts: On October 2005 the San Gabriel Police Station of La Union, conducted a checkpoint, composed of The Chief of Police, PO2 Pallayoc, and other policemen, near the police station at the poblacion to intercept 93

a suspected transportation of marijuana from Barangay Balbalayang, La Union. When the checkpoint did not yield any suspect or marijuana, the Chief of Police instructed PO2 Pallayoc to proceed to Barangay Balbalayang to conduct surveillance operation. There PO2 Pallayoc met with a secret agent of the Barangay Intelligence Network who informed him that a baggage of marijuana had been loaded on a passenger jeepney that was about to leave for the poblacion. The agent mentioned three (3) bags and one (1) blue plastic bag. Further, the agent described a backpack bag with an "O.K." marking. PO2 Pallayoc then boarded the said jeepney and positioned himself on top thereof. While the vehicle was in motion, he found the black backpack with an "O.K." marking and peeked inside its contents and found bricks of marijuana wrapped in newspapers. He then asked the other passengers on top of the jeepney about the owner of the bag, but no one knew. When the jeepney reached the poblacion, PO2 Pallayoc alighted together with the other passengers. Unfortunately, he did not notice who took the black backpack from atop the jeepney. He only realized a few moments later that the said bag and three (3) other bags, including a blue plastic bag, were already being carried away by two (2) women. He caught up with the women and introduced himself as a policeman. He told them that they were under arrest, but one of the women got away. PO2 Pallayoc brought the woman, who was later identified as accusedappellant Belen Mariacos, and the bags to the police station. The RTC promulgated a decision finding Mariacos guilty as charged. She appealed her conviction to the CA arguing that the search conducted on the bag, assuming it was hers, without a search warrant and with no permission from her, violates heer constitutional rights against warrantless search. The CA dismissed appellants appeal and affirmed the RTC decision in toto ruling that Mariacos was caught in flagrante delicto of "carrying and conveying" the bag that contained the illegal drugs, and thus held that appellants warrantless arrest was valid. Issue: Whether the warrantless search conducted on the bag of a moving vehicle was valid Held: YES. Mariacos main argument centered on the inadmissibility of the evidence used against her. Among the instances when a warrantless search is valid, is search of a moving vehicle. According to jurisprudence, this had been justified on the ground that the mobility of motor vehicles makes it possible for the vehicle to be searched to move out of the locality or jurisdiction in which the warrant must be sought. However, such a warrantless search has been held to be valid only as long as the officers conducting the search have reasonable or probable cause to believe before the search that they will find the instrumentality or evidence pertaining to a crime, in the vehicle to be searched. The essential requisite of probable cause must be satisfied before a warrantless search and seizure can be lawfully conducted. Probable cause is defined as a reasonable ground of suspicion supported by circumstances sufficiently strong in themselves to induce a cautious man to believe that the person accused is guilty of the offense charged. It refers to the existence of such facts and circumstances that can lead a reasonably discreet and prudent man to believe that an offense has been committed, and that the items, articles or objects sought in connection with said offense or subject to seizure and destruction by law are in the place to be searched. The grounds of suspicion are reasonable when, in the absence of actual belief of the arresting officers, the suspicion that the person to be arrested is probably guilty of committing the offense is based on actual facts, i.e., supported by circumstances sufficiently strong in themselves to create the probable 94

cause of guilt of the person to be arrested. A reasonable suspicion therefore must be founded on probable cause, coupled with good faith on the part of the peace officers making the arrest. In this case, the vehicle that carried the contraband or prohibited drugs was about to leave. PO2 Pallayoc had to make a quick decision and act fast. It would be unreasonable to require him to procure a warrant before conducting the search under the circumstances. Time was of the essence in this case. The searching officer had no time to obtain a warrant. Indeed, he only had enough time to board the vehicle before the same left for its destination. It is well to remember that on October 26, 2005, the night before appellants arrest, the police received information that marijuana was to be transported from Barangay Balbalayang, and had set up a checkpoint around the area to intercept the suspects. At dawn of October 27, 2005, PO2 Pallayoc met the secret agent from the Barangay Intelligence Network, who informed him that a baggage of marijuana was loaded on a passenger jeepney about to leave for the poblacion. Thus, PO2 Pallayoc had probable cause to search the packages allegedly containing illegal drugs. WHEREFORE, the foregoing premises considered, the appeal is DISMISSED. The Decision of the Court of Appeals in CA-G.R. CR-HC No. 02718 is AFFIRMED.
PEOPLE VS. TUAN G.R. NO. 176066, 11 AUGUST 2010

Facts: On January 2000, two informants namely, Tudlong and Lad-ing arrived at the office of CIDG (Criminal Investigation and Detention Group) in Baguio City, and reported to SPO2 Fernandez, Chief of the Station Drug Enforcement Unit (SDEU), that a certain "Estela Tuan" had been selling marijuana at Barangay Gabriela Silang, Baguio City. SPO2 Fernandez set out to verify the report of Tudlong and Lad-ing. On the afternoon of the same day, he gave Tudlong and Lad-ing P300.00 to buy marijuana, and accompanied the two informants to the accused Tuans house. Tudlong and Lad-ing entered the house, while SPO2 Fernandez waited at the adjacent house. Later, Tudlong and Lad-ing came out and showed SPO2 Fernandez the marijuana they bought. Upon returning to the CIDG office, SPO2 Fernandez requested a laboratory examination on the specimen and yielded positive results for marijuana. SPO2 Fernandez, together with the informants, filed the Application for a Search Warrant before Judge Iluminada Cabato-Cortes (Judge Cortes) of the Municipal Trial Court in Cities (MTCC), Baguio City on January 25, 2000. Two hours later, at around three oclock, Judge Cortes personally examined SPO2 Fernandez, Tudlong, and Lad-ing, after which, she issued a Search Warrant, which stated Tuans residence as the house of the accused Estela Tuan at Brgy. Gabriela Silang, Baguio City Even though accused Tuan was not around, the CIDG team was allowed entry into the house by Magno Baludda (Magno), accuseds father, after he was shown a copy of the Search Warrant. SPO2 Fernandez guarded the surroundings of the house, while SPO1 Carrera and PO2 Chavez searched inside. They saw, in the presence of Magno, a movable cabinet in Tuans room, below of which they found a brick of marijuana and a firearm. Later Tuan arrived and thereafter, the police officers asked Tuan to open a cabinet, in which they saw more bricks of marijuana. The defense, on the other hand, disclaimed ownership of the bricks and alleged that a Search Warrant was issued for her house because of a quarrel with her neighbor named Lourdes Estillore (Estillore). The RTC found accused guilty as charged. On appeal, the CA modified by acquitting Tuan of the charge for illegal possession of firearm but affirming her conviction for illegal possession of marijuana. Tuan raised the matter to the Supreme Court contending, among others, that the warrant failed to particularly describe the place because the house was a two-storey building composed of several rooms. 95

Issue: Whether there was probable cause for the judge to issue a Search Warrant and whether the search warrant particularly described the place to be searched. Held: Whether there was probable cause for the judge to issue a Search Warrant YES. The validity of the issuance of a search warrant rests upon the following factors: (1) it must be issued upon probable cause; (2) the probable cause must be determined by the judge himself and not by the applicant or any other person; (3) in the determination of probable cause, the judge must examine, under oath or affirmation, the complainant and such witnesses as the latter may produce; and (4) the warrant issued must particularly describe the place to be searched and persons or things to be seized. The only issue is compliance with the first and fourth factors, i.e., existence of probable cause; and particular description of the place to be searched and things to be seized. Probable cause generally signifies a reasonable ground of suspicion supported by circumstances sufficiently strong in themselves to warrant a cautious man to believe that the person accused is guilty of the offense with which he is charged. It likewise refers to the existence of such facts and circumstances which could lead a reasonably discreet and prudent man to believe that an offense has been committed and that the item(s), article(s) or object(s) sought in connection with said offense or subject to seizure and destruction by law is in the place to be searched. Before a search warrant can be issued, it must be shown by substantial evidence that the items sought are in fact seizable by virtue of being connected with criminal activity, and that the items will be found in the place to be searched. A magistrates determination of probable cause for the issuance of a search warrant is paid great deference by a reviewing court, as long as there was substantial basis for that determination. Substantial basis means that the questions of the examining judge brought out such facts and circumstances as would lead a reasonably discreet and prudent man to believe that an offense has been committed, and the objects in connection with the offense sought to be seized are in the place sought to be searched.Such substantial basis exists in this case. Judge Cortes found probable cause for the issuance of the Search Warrant for Tuans residence after said judges personal examination of SPO2 Fernandez, the applicant; and Lad-ing and Tudlong, the informants. SPO2 Fernandez based his Application for Search Warrant not only on the information relayed to him by Lad-ing and Tudlong. He also arranged for a test buy and conducted surveillance of Tuan Issue: Whether the search warrant particularly described the place to be searched. Held: YES. A description of the place to be searched is sufficient if the officer serving the warrant can, with reasonable effort, ascertain and identify the place intended and distinguish it from other places in the community. A designation or description that points out the place to be searched to the exclusion of all others, and on inquiry unerringly leads the peace officers to it, satisfies the constitutional requirement of definiteness. In the case at bar, the address and description of the place to be searched in the Search Warrant was specific enough. There was only one house located at the stated address, which was accused-appellants residence, consisting of a structure with two floors and composed of several rooms.

96

WHEREFORE, premises considered, the Decision dated September 21, 2006 of the Court of Appeals in CA-G.R. CR.-H.C. No. 00381, is hereby AFFIRMED in toto. No costs.
ESQUILLO VS. PEOPLE G.R. NO. 182010, AUGUST 25, 2010

Facts: On the basis of an informants tip, PO1 Cruzin, together with PO2 Aguas, proceeded to Bayanihan St., Pasay City to conduct surveillance on the activities of an alleged notorious snatcher operating in the area known only as "Ryan." As PO1 Cruzin alighted from the private vehicle that brought him and PO2 Aguas to the target area, he glanced in the direction of petitioner who was standing three meters away and seen placing inside a yellow cigarette case what appeared to be a small heat-sealed transparent plastic sachet containing white substance. While PO1 Cruz was not sure what the plastic sachet contained, he became suspicious when petitioner started acting strangely as he began to approach her. He then introduced himself as a police officer to petitioner and inquired about the plastic sachet she was placing inside her cigarette case. Instead of replying, however, petitioner attempted to flee to her house nearby but was timely restrained by PO1 Cruzin who then requested her to take out the transparent plastic sachet from the cigarette case. After apprising petitioner of her constitutional rights, PO1 Cruzin confiscated the plastic sachet. Repudiating the charges, petitioner averred that she was sick and resting home at the time of the incident and also claimed that the evidence against her was "planted," stemming from an all too obvious attempt by the police officers to extort money from her and her family. Trial Court found petitioner guilty of illegal possession of Methylamphetamine Hydrochloride or shabu which the CA affirmed. ISSUE: Whether the arrest of the petitioner is warranted. HELD: YES. Recall that the police officers were on a surveillance operation as part of their law enforcement efforts. When PO1 Cruzin saw petitioner placing a plastic sachet containing white crystalline substance into her cigarette case, it was in his plain view. Given his training as a law enforcement officer, it was instinctive on his part to be drawn to curiosity and to approach her. That petitioner reacted by attempting to flee after he introduced himself as a police officer and inquired about the contents of the plastic sachet all the more pricked his curiosity. That a search may be conducted by law enforcers only on the strength of a valid search warrant is settled. The same, however, admits of exceptions, viz: (1) consented searches; (2) as an incident to a lawful arrest; (3) searches of vessels and aircraft for violation of immigration, customs, and drug laws; (4) searches of moving vehicles; (5) searches of automobiles at borders or constructive borders; (6) where the prohibited articles are in "plain view;" (7) searches of buildings and premises to enforce fire, sanitary, and building regulations; and (8) "stop and frisk" operations. In stop and frisk operation what is, therefore, essential is that a genuine reason must exist, in light of the police officers experience and surrounding conditions, to warrant the belief that the person who manifests unusual suspicious conduct has weapons or contraband concealed about him. Such a "stopand-frisk" practice serves a dual purpose: (1) the general interest of effective crime prevention and 97

detection, which underlies the recognition that a police officer may, under appropriate circumstances and in an appropriate manner, approach a person for purposes of investigating possible criminal behavior even without probable cause; and (2) the more pressing interest of safety and selfpreservation which permit the police officer to take steps to assure himself that the person with whom he deals is not armed with a deadly weapon that could unexpectedly and fatally be used against the police officer. From these standards, the Court finds that the questioned act of the police officers constituted a valid "stop-and-frisk" operation. The search/seizure of the suspected shabu initially noticed in petitioners possession - later voluntarily exhibited to the police operative - was undertaken after she was interrogated on what she placed inside a cigarette case, and after PO1 Cruzin introduced himself to petitioner as a police officer. And, at the time of her arrest, petitioner was exhibiting suspicious behavior and in fact attempted to flee after the police officer had identified himself. Courts have tended to look with disfavor on claims of accused, such as those of petitioners, that they are victims of a frame-up. The defense of frame-up, like alibi, has been held as a shop-worn defense of the accused in drug-related cases, the allegation being easily concocted or contrived. For this claim to prosper, the defense must adduce clear and convincing evidence to overcome the presumption of regularity of official acts of government officials. This it failed to do. WHEREFORE, the assailed decision of the Court of Appeals is AFFIRMED, with the MODIFICATION that the penalty of imprisonment shall be twelve (12) years and one (1) day, as minimum, to fourteen (14) years, as maximum. In all other respects, the decision of the RTC in Criminal Case No. 02-2297 is AFFIRMED.

98

Vous aimerez peut-être aussi